Eufisky - The lost book

几个恒等式证明

1.证明:

\[\sum_{j=1}^{n-1}{\frac{1}{1-\exp \left\{ \frac{2\pi ij}{n} \right\}}}=\frac{n-1}{2}.\]
 

2.设$N$为自然数, $\{x\}$表示$x$的小数部分.证明\[\sum_{n=1}^N{\left( \left\{ x+\frac{n}{N} \right\} -\frac{1}{2} \right)}=\left\{ Nx \right\} -\frac{1}{2}.\]

若$a,b$是互质的正整数,证明
\[\int_0^1{\left( \left\{ ax \right\} -\frac{1}{2} \right) \left( \left\{ bx \right\} -\frac{1}{2} \right) dx}=\frac{1}{12ab}.\]
 
3.求极限
\[\lim_{m\rightarrow \infty}\lim_{n\rightarrow \infty}\int_0^1{\int_0^1{\cdots \int_0^1{\sum_{i=0}^m{\exp \left\{ -\frac{n}{\sum_{k=1}^n{x_{k}^{m-1}}} \right\} \frac{\prod_{j=1}^i{\sum_{k=1}^n{x_{k}^{j-1}}}}{\left( \sum_{k=1}^n{x_{k}^{m-1}} \right) ^i}}dx_1dx_2\cdots dx_n}}}.\]
4.证明
\begin{align*}f\left( x \right) &=\frac{1}{a}+\frac{x}{a\left( a+d \right)}+\cdots +\frac{x^n}{a\left( a+d \right) \cdots \left( a+nd \right)}+\cdots\\&=\frac{e^{x/d}}{dx^{a/d}}\int_0^x{e^{-t/d}t^{a/d-1}dt}.\end{align*}
提示:考虑关于$f(x)$的微分方程.
 
5.证明Ramanujan的恒等式
\[\int_0^{\infty}{e^{-3\pi x^2}\frac{\sin\text{h}\pi x}{\sin\text{h}3\pi x}dx}=\frac{1}{\sqrt{3}e^{2\pi /3}}\sum_{n=0}^{\infty}{\frac{e^{-2n\left( n+1 \right) \pi}}{\left( 1+e^{-\pi} \right) ^2\left( 1+e^{-3\pi} \right) ^2\cdots \left( 1+e^{-\left( 2n+1 \right) \pi} \right) ^2}}.\]
参考:G. N. Watson 1936:The Final Problem : An Account of the Mock Theta Functions
6.Compute 
$$\sum_{n=1}^{\infty}{\left( \frac{n^n}{n!e^n}-\frac{1}{\sqrt{2\pi n}} \right)}.$$
By the software Mathematica, I find
$$\sum_{n=1}^{\infty}{\left( \frac{n^n}{n!e^n}-\frac{1}{\sqrt{2\pi n}} \right)}=-\frac{2}{3}-\frac{\zeta \left( 1/2 \right)}{\sqrt{2\pi}}.$$

Well, $-\frac{1}{\sqrt{2\pi}}\zeta\left(\tfrac{1}{2}\right)$ is the $\zeta$-regularization of the divergent series $\sum_{n\geq 1}\frac{1}{\sqrt{2\pi n}}$, hence the problem boils down to finding the $\zeta$-regularization of the divergent series $\sum_{n\geq 1}\frac{n^n}{n!e^n}$. As pointed out in the comments,
 
$$ W(x) = \sum_{n\geq 1}\frac{n^{n-1}(-1)^{n-1}}{n!}x^n $$
holds for any $x\in\left(-\frac{1}{e},\frac{1}{e}\right)$ by Lagrange inversion theorem, hence
$$ ze^{-z} W'(-ze^{-z})=\sum_{n\geq 1}\frac{n^n}{n!e^{nz}}z^{n}=\frac{z}{1-z} =\sum_{n\geq 1}z^n\tag{1}$$
holds for any $z\in(-W(e^{-1}),1)$. Pretty strange identity, I can give you that.<br>
Similarly, over the same interval
$$ -W(-z e^{-z})=\sum_{n\geq 1}\frac{n^{n-1}}{n!e^{nz}}z^n = z \tag{2}$$
$$ 1=\sum_{n\geq 1}\frac{n^{n}}{n!e^{nz}}z^{n-1}-\sum_{n\geq 1}\frac{n^{n}}{n!e^{nz}}z^{n}=\frac{1}{1-z}-\frac{z}{1-z}.\tag{3}$$
Since $\zeta(0)=-\frac{1}{2}$, it should not be difficult to prove from $(1)$ and $(2)$ that the $\zeta$-regularization of $\sum_{n\geq 1}\frac{n^n}{n!e^n}$ equals $-\frac{2}{3}$ as wanted, for instance by computing $\sum_{n\geq 1}\frac{n^{n-1-k}}{n!e^n}$ for any $k\in\mathbb{N}$:
$$ \sum_{n\geq 1}\frac{n^{n-2}}{n!e^n}=\int_{-1/e}^{1}\frac{W(x)}{x}\,dx = \frac{1}{2},\qquad \sum_{n\geq 1}\frac{n^{n-3}}{n!e^n}=-\int_{-1/e}^{1}\frac{W(x)}{x}(1+\log(-x))\,dx=\frac{5}{12} $$
$$ \sum_{n\geq 1}\frac{n^{n-4}}{n!e^n}=\frac{7}{18},\qquad \sum_{n\geq 1}\frac{n^{n-4}}{n!e^n}=\frac{1631}{4320},$$
$$ \sum_{n\geq 1}\frac{n^{n-1-k}}{n!e^n}= \frac{1}{\Gamma(k)}\int_{0}^{1}(1-x)(x-1-\log x)^{k-1}\,dx.\tag{4} $$
Indeed the substitution $x=e^{-s}$ in $(4)$ and the integral representation for the $\zeta$ function complete the proof.
Taking $$F\left(x\right)=\sum_{n\geq1}\frac{n^{n-1}}{n!e^{n}}x^{n}-\frac{1}{\sqrt{2\pi}}\sum_{n\geq1}\frac{x^{n}}{n^{3/2}}=-W\left(-\frac{x}{e}\right)-\frac{\mathrm{Li}_{3/2}\left(x\right)}{\sqrt{2\pi}},\,\left|x\right|<1$$ where $W\left(x\right)$ is the [Lambert $W$ function][1] and $\mathrm{Li}_{3/2}\left(x\right)$ is the [Polylogarithm function][2], we obtain, differentiating both sides,that $$\sum_{n\geq1}\left(\frac{n^{n}}{n!e^{n}}-\frac{1}{\sqrt{2\pi n}}\right)x^{n-1}=-\frac{W\left(-\frac{x}{e}\right)}{x\left(W\left(-\frac{x}{e}\right)+1\right)}-\frac{\mathrm{Li}_{1/2}\left(x\right)}{x\sqrt{2\pi}}$$ so $$\sum_{n\geq1}\left(\frac{n^{n}}{n!e^{n}}-\frac{1}{\sqrt{2\pi n}}\right)=\lim_{x\rightarrow1^{-}}\left(-\frac{W\left(-\frac{x}{e}\right)}{x\left(W\left(-\frac{x}{e}\right)+1\right)}-\frac{\mathrm{Li}_{1/2}\left(x\right)}{x\sqrt{2\pi}}\right).$$ Now, [we know][3] that $$\mathrm{Li}_{v}\left(z\right)=\left(\Gamma\left(1-v\right)\left(1-z\right)^{v-1}+\zeta\left(v\right)\right)\left(1+O\left(\left|1-z\right|\right)\right),v\neq1,\,z\rightarrow1$$  and now we claim $$-\frac{W\left(-\frac{x}{e}\right)}{x\left(W\left(-\frac{x}{e}\right)+1\right)}\sim\frac{1}{\sqrt{2\left(1-x\right)}}-\frac{2}{3}$$ as $x\rightarrow1^{-}$. This is true because, [since][5] $$W\left(z\right)\sim-1+\sqrt{2ze+2}-\frac{2}{3}e\left(z+\frac{1}{e}\right)$$ as $z\rightarrow-1/e$, we have $$-\frac{W\left(-\frac{x}{e}\right)}{x\left(W\left(-\frac{x}{e}\right)+1\right)}\sim\frac{1-\sqrt{2\left(1-x\right)}+\frac{2}{3}\left(1-x\right)}{x\sqrt{2\left(1-x\right)}-\frac{2}{3}\left(1-x\right)x}$$ $$=\frac{1}{x}\left(-1+\frac{1}{\sqrt{2\left(1-x\right)}}\left(\frac{1}{1-\sqrt{2-2x}/3}\right)\right)=\frac{1}{x}\left(-1+\frac{1}{\sqrt{2\left(1-x\right)}}\sum_{k\geq0}\left(\frac{\sqrt{2-2x}}{3}\right)^{k}\right)$$ $$=\frac{1}{x}\left(-\frac{2}{3}+\frac{1}{\sqrt{2\left(1-x\right)}}+O\left(\sqrt{1-x}\right)\right)$$ then the claim.
 
[1]:https://en.wikipedia.org/wiki/Lambert_W_function
[2]:https://en.wikipedia.org/wiki/Polylogarithm
[3]:http://functions.wolfram.com/ZetaFunctionsandPolylogarithms/PolyLog/06/01/02/01/01/
[4]:http://mathworld.wolfram.com/StirlingsSeries.html
[5]:http://functions.wolfram.com/ElementaryFunctions/ProductLog/06/01/02/
This is a general answer to the followup question by Jack D'Aurizio. 
 
**Proposition**
>Let $n\in\mathbb{N}$. We have the asymptotic expansion
$$n!\sim \sqrt{2\pi n} \frac{n^n}{e^n} \left[ 1+ \frac1{12n} +\frac1{288n^2}-\frac{139}{51840n^3}-\frac{157}{2488320n^4}+\cdots \right]$$
 
Note that this is not a convergent series, but an asymptotic expansion. The error in the truncated series is asymptotically equal to the first omitted term. Regard the series on the right as an element of the ring of power series over rational numbers $\mathbb{Q}[[T]]$. 
$$S(T)=1+ \frac1{12}T+\frac1{288}T^2-\frac{139}{51840}T^3-\frac{157}{2488320}T^4 + \cdots. $$
Consider the multiplicative inverse of $S(T)$ in $\mathbb{Q}[[T]]$.
$$S^{-1}(T)=1-\frac1{12}T+ g_2 T^2 + g_3 T^3 + g_4 T^4 + \cdots. $$
Let $Y_s(T)=\sum_{n=0}^{\infty} h_n(s) T^n \in\mathbb{Q}[s][[T]]$ be defined by
$$\left(\frac12 T^2\right)^{s-1}\sum_{n=0}^{\infty} h_n(s) T^n = \left[ \frac12 T^2 + \frac13 T^3 + \frac14 T^4+\cdots \right]^{s-1}.$$
Then we have
 
**Theorem**
>$$\sum_{n=1}^{\infty} n^p\left[ \frac{n^n}{n!e^n}- \frac1{\sqrt{2\pi n}} \sum_{k=0}^p \frac{g_k}{n^k}\right]=(-2)^p p!h_{2p+1}(-p)  - \frac1{\sqrt{2\pi}}\sum_{k=0}^p g_k \zeta\left(k+\frac12-p\right).$$
 
With $p=0$, it is the original series
$$\sum_{n=1}^{\infty} \left[\frac{n^n}{n!e^n} - \frac1{\sqrt{2\pi n}}\right]=-\frac23 - \frac{\zeta\left(\frac12\right)}{\sqrt{2\pi}}$$
 
With $p=1$, it gives the value of 
$$\sum_{n=1}^{\infty} \left[ \frac{n^{n+1}}{n!e^n} - \sqrt{\frac{n}{2\pi}} + \frac{1}{12\sqrt{2\pi n}}\right] = -\frac 4{135} - \frac{\zeta\left(-\frac12\right)}{\sqrt{2\pi}} + \frac{\zeta\left(\frac12\right)}{12\sqrt{2\pi}}.$$

7.Prove that $$\sum_{m\leqslant x}\sum_{n\leqslant x}\Big\{\frac{x}{m+n}\Big\}=\Big(2\log2-\frac{\pi^2}{12}\Big)x^2+O(x\log x),$$

where $\{x\}$ is the fractional part of the real number $x$.
 
I know
$$\sum_{n\leqslant x} \Big\{\frac{x}{n} \Big\}=(1-\gamma)x+O\big(x^{1/2}\big),$$

where $\gamma$ is Euler's constant. But I don't know whether it is useful. Can you help me?

参考:这里


Write $\{x\} = x - \lfloor x\rfloor$,s so that
\[\sum_{m \leq x} \sum_{n \leq x} \left\{ \frac{x}{m + n}\right\} = \sum_{m \leq x} \sum_{n \leq x} \frac{x}{m + n} - \sum_{m \leq x} \sum_{n \leq x} \left\lfloor \frac{x}{m + n}\right\rfloor.\]
Then note that
\[\sum_{m \leq x} \sum_{n \leq x} \left\lfloor \frac{x}{m + n}\right\rfloor = \sum_{m \leq x} \sum_{n \leq x} \sum_{\ell \leq \frac{x}{m + n}} 1 = \sum_{\ell \leq \frac{x}{2}} \sum_{n \leq \frac{x}{\ell} - 1} \sum_{m \leq \frac{x}{\ell} - n} 1.\]
The sum over $m$ is $\lfloor x/\ell\rfloor - n$. The ensuing sum over $n$ is
\[\left\lfloor \frac{x}{\ell}\right\rfloor \left(\left\lfloor \frac{x}{\ell}\right\rfloor - 1\right) - \sum_{n \leq \frac{x}{\ell} - 1} n.\]
Via partial summation,
\[\sum_{n \leq \frac{x}{\ell} - 1} n = \left(\frac{x}{\ell} - 1\right) \left(\left\lfloor \frac{x}{\ell} \right\rfloor - 1\right) - \int_{1}^{\frac{x}{\ell} - 1} \lfloor t\rfloor \\, dt,\]
and this integral is equal to $\frac{1}{2} \left(\frac{x}{\ell} - 1\right)^2 + O(\frac{x}{\ell})$. So the sum over $n$ and $m$ simplifies to
\[\frac{x^2}{2 \ell^2} + O\left(\frac{x}{\ell}\right).\]
The ensuing sum over $\ell$ is
\[\frac{x^2}{2} \sum_{\ell = 1}^{\infty} \frac{1}{\ell^2} - \frac{x^2}{2} \sum_{\ell > \frac{x}{2}} \frac{1}{\ell^2} + O(x \log x).\]
The first sum over $\ell$ is $\zeta(2) = \pi^2/6$. The second is $O(1/x)$. So this simplifies to
\[\frac{\pi^2 x^2}{12} + O(x \log x).\]
 
Now we deal with
\[\sum_{m \leq x} \sum_{n \leq x} \frac{x}{m + n} = x \sum_{m \leq x} \sum_{n \leq x} \frac{1}{m + n}.\]
We deal with the sum over $n$ via partial summation: it is equal to
\[\frac{\lfloor x\rfloor}{m + x} + \int_{1}^{x} \frac{\lfloor t \rfloor}{(m + t)^2} \\, dt = \log \frac{m + x}{m + 1} + O\left(\frac{1}{m}\right),\]
where we have used the fact that $\lfloor x \rfloor = x - \{x\} = x + O(1)$, the fact that the antiderivative of $t/(m + t)^2$ is $m/(m + t) + \log(m + t)$, and the fact that the antiderivative of $1/(m + t)^2$ is $-1/(m + t)$.
 
So it remains to evaluate
\[\sum_{m \leq x} \log \frac{m + x}{m + 1} = \lfloor x\rfloor \log \frac{2x}{x + 1} + (x + 1) \int_{1}^{x} \frac{\lfloor t\rfloor}{(t + x)(t + 1)} \\, dt.\]
The antiderivative of $\frac{t}{(t + x)(t + 1)}$ is $\frac{x}{x - 1} \log \frac{t + x}{t + 1}$, and so after some simplification, we arrive at $(2\log 2) x + O(\log x)$.
 

 

1-1+1-1+...=1/2?

来源:马明辉QQ空间
 
在一个寒风凛冽的晚上,我一手拿着烤红薯,一手玩着手机。
 
当时我正在看熊哥的微信公众号: Xionger的数学小屋,他在上面发了北大18年研究生考试题(侵删)。
 
当然了,我早已对这种考试题失去了兴趣,但唯独最后一题,却很是在意。我与向老师讨论了这个题目,向老师嘛,以前与之讨论过一些数学问题,感觉他特厉害,随手就写了份解答给我。
 
Suppose $f(x)>0$, $f''(x)\le 0$, and $f(+\infty)=+\infty$ on $[0,+\infty)$, prove the following conclusion
\[ \lim\limits_{s\to 0+} \sum\limits_{n=0}^{\infty} \frac{(-1)^n}{f^s(n)}=\frac{1}{2}. \]
 
\begin{proof}
     Since $f(x)>0$, $f''(x)\le 0$, and $f(+\infty)=+\infty$, it is not hard to see $f'(x)>0$, then
\[ \sum\limits_{n=0}^{\infty} \frac{(-1)^n}{f^s(n)}=\sum\limits_{n=0}^{\infty} \left( \frac{1}{f^s(2n)}-\frac{1}{f^s(2n+1)} \right)=-\sum\limits_{n=0}^{\infty} g_s(\xi_n), \]
where $g_s(x)=(\frac{1}{f^s(x)})'=-\frac{sf'(x)}{f^{s+1}(x)}$ and $\xi_n \in (2n,2n+1)$. Since
\[ g_s'(x)=s\cdot \frac{(s+1)f'^2(x)-f''(x)f(x)}{f^{s+2}(x)}>0, \]
we can get
\[ -\sum\limits_{n=0}^{\infty} g_s(\xi_n)\le -\sum\limits_{n=0}^{\infty} g_s(2n)\le -g_s(0)-\frac{1}{2}\sum\limits_{n=1}^{\infty} \int_{2n-2}^{2n} g_s(x)dx \]
\[ =-g_s(0)-\frac{1}{2}\int_{0}^{+\infty} g_s(x)dx=-g_s(0)+\frac{1}{2f^s(0)}\to \frac{1}{2} \]
as $s\to 0_+$. Similarly, we have
\[ -\sum\limits_{n=0}^{\infty} g_s(\xi_n)\ge -\sum\limits_{n=0}^{\infty} g_s(2n+1)\ge -\frac{1}{2}\sum\limits_{n=0}^{\infty} \int_{2n+1}^{2n+3} g_s(x)dx \]
\[ =-\frac{1}{2}\int_{1}^{+\infty} g_s(x)dx=\frac{1}{2f^s(1)}\to \frac{1}{2} \]
as $s\to 0_+$.
\end{proof}
 
想起前段时间刚好做过这样一道题,来源于Tenenbaum解析数论课后题230,证明基于Euler-Maclaurin公式(不了解的可自行wiki)。
 
Prove that $\lim\limits_{s\to 0+}\sum\limits_{n=0}^{\infty} (-1)^ne^{-sn^2}=\frac{1}{2}. $
 
\begin{proof}
     Let $f_s(x)=e^{-sx^2}$, by Euler-Maclaurin formula, we have
\[ \sum\limits_{n=0}^{N} f_s(n)=\int_{0}^{N} f_s(x)dx+\frac{f_s(0)+f_s(N)}{2}+\frac{f_s'(0)+f_s'(N)}{12}-\frac{1}{2}\int_{0}^{N} B_2(x)f_s''(x)dx. \]
Since $f_s'(x)=-2sxe^{-sx^2}$, $f_s''(x)=2se^{-sx^2}(2sx^2-1)$, we can get
\[ \int_{0}^{N} f_s(x)dx\to \int_{0}^{+\infty} f_s(x)dx=\int_{0}^{+\infty} e^{-sx^2}dx=\frac{\sqrt{\pi}}{2}\cdot \frac{1}{\sqrt{s}} \]
\[ f_s(0)+f_s(N)=1+e^{-sN^2}\to 1 \]
\[ f_s'(0)+f_s'(N)=-2sNe^{-sN^2}\to 0 \]
\[ \int_{0}^{N} B_2(x)f_s''(x)dx\to \int_{0}^{\infty} B_2(x)f_s''(x)dx=\int_{0}^{\infty} B_2(x)2se^{-sx^2}(2sx^2-1)dx \]
\[ =2\sqrt{s}\int_{0}^{\infty} B_2(\frac{x}{\sqrt{s}})e^{-x^2}(2x^2-1)dx \]
as $N\to \infty$. Therefore, we have
\[ \sum\limits_{n=0}^{\infty} e^{-sn^2}=\frac{\sqrt{\pi}}{2}\cdot \frac{1}{\sqrt{s}}+\frac{1}{2}-2\sqrt{s}\int_{0}^{\infty} B_2(\frac{x}{\sqrt{s}})e^{-x^2}(2x^2-1)dx. \]
Since $B_2(x)$ is bounded, we can get $\sum\limits_{n=0}^{\infty} e^{-sn^2}=\frac{\sqrt{\pi}}{2}\cdot \frac{1}{\sqrt{s}}+\frac{1}{2}+O(\sqrt{s})$ as $s\to 0_+$, then $\sum\limits_{n=0}^{\infty} (-1)^ne^{-sn^2}=2\sum\limits_{n=0}^{\infty} e^{-4sn^2}-\sum\limits_{n=0}^{\infty} e^{-sn^2}=\frac{1}{2}+O(\sqrt{s})$.\\
Another method: Let $\vartheta(t)=\sum\limits_{n\in\mathbb{Z}}e^{-\pi n^2t}$, $t>0$ be the Jocabi theta function. Since
\[ \sum\limits_{n=1}^{\infty} e^{-\pi n^2t}\le \sum\limits_{n=1}^{\infty} e^{-\pi nt}=\frac{e^{-\pi t}}{1-e^{-\pi t}}=O(e^{-\pi t}),  \]
we can get
\[ \vartheta(t)=1+2\sum\limits_{n=1}^{\infty} e^{-\pi n^2t}=1+O(e^{-\pi t}) \]
and
\[ \vartheta(\frac{1}{t})=\sqrt{t}\theta(t)=\sqrt{t}+O(\sqrt{t}e^{-\pi t}). \]
as $t\to \infty$. Then we have
\[ \sum\limits_{n=0}^{\infty} e^{-sn^2}= \frac{1}{2}+\frac{1}{2}\vartheta(\frac{s}{\pi})=\frac{\sqrt{\pi}}{2}\cdot \frac{1}{\sqrt{s}}+\frac{1}{2}+O(s^{-\frac{1}{2}}e^{-\frac{1}{s}}) \]
as $s\to 0_+$.
\end{proof}
 
可能利用Jacobi Theta function也能解决,我没试过。现在问题来了,把指数上的n²换成n³结论还对不对?我问了向老师,他也没什么简单的做法。
 
Suppose $p(x)=x^m+a_{m-1}x^{m-1}+...+a_1x+a_0$ is a monic polynomial with degree $m\ge 1$, prove that
\[ \lim\limits_{s\to 0+}\sum\limits_{n=0}^{\infty} (-1)^ne^{-sp(n)}=\frac{1}{2}. \]
 
这种结论一看感觉就是对的呀,但是为什么做不出来啊?这就尴尬了。
 
大概是大二那年十一假期,闲来无事写过一份文档,是讨论Grandi级数的某些意义下的收敛性的,有兴趣可以去看一下http://duodaa.com/blog/index.php/archives/351/。然后呢,我就想起了以前做过的一些题目。这个是借用Γ函数的性质做的,也可以在北大那道题中取f(x)=x。
 
Prove that $\lim\limits_{s\to 0+}\sum\limits_{n=1}^{\infty} \frac{(-1)^n}{n^s}=-\frac{1}{2}. $
 
\begin{proof}
     Since
\[ \Gamma(s)=\int_0^{\infty}x^{s-1}e^{-x}dx=\int_0^{\infty}(at)^{s-1}e^{-at}dat=a^s\int_0^{\infty}t^{s-1}e^{-at}dt, \]
we have
\[ a^{-s}=\frac{1}{\Gamma(s)}\int_0^{\infty}t^{s-1}e^{-at}dt. \]
Then
\[ \sum\limits_{n=1}^{\infty}\frac{(-1)^n}{n^s}=\frac{1}{\Gamma(s)}\int_0^{\infty}t^{s-1}\sum\limits_{n=1}^{\infty}(-1)^ne^{-nt}dt=\frac{1}{\Gamma(s)}\int_0^{\infty}t^{s-1}\frac{-e^{-t}}{1+e^{-t}}dt \]
\[ =-\frac{1}{\Gamma(s)}\int_0^{\infty}\frac{t^{s-1}}{1+e^t}dt=-\frac{\int_0^{\infty}\frac{t^{s-1}}{1+e^t}dt}{\int_0^{\infty}t^{s-1}e^{-t}dt}\to -\frac{1}{2} \]
as $s\to 0_+$.
\end{proof}
 
上下极限讨论的细节可见http://www.duodaa.com/?/question/6738。小试身手之后,再看一道比较难的题,来源于biler数学分析问题集3.58。
 
Let $S(s)=\sum\limits_{n=0}^{\infty} \frac{(-1)^n}{\sqrt{1+n^2s}}$, prove that $\lim\limits_{s\to 0+}S(s)=\frac{1}{2}$.
 
\begin{proof}
     It is generally known that $\sin z=z\mathop{\Pi}\limits_{n=1}^{\infty}(1-\frac{z^2}{n^2\pi^2})$, or
\[ \sin \pi z=\pi z\mathop{\Pi}\limits_{n=1}^{\infty}(1-\frac{z^2}{n^2}). \]
Take logarithmic derivative, we have
\[ \pi\cot \pi z=\frac{1}{z}+\sum\limits_{n=1}^{\infty}\frac{-2z}{n^2-z^2}=\frac{1}{z}+\sum\limits_{n=1}^{\infty}(\frac{1}{z+n}+\frac{1}{z-n})=\frac{1}{2}\sum\limits_{n\in \mathbb{Z}}(\frac{1}{z+n}+\frac{1}{z-n}). \]
Since
\[ \frac{1}{\sin \pi z}-\cot \pi z=\frac{1-\cos \pi z}{\sin \pi z}=\tan \frac{\pi z}{2} \]
and
\[ \pi \tan \frac{\pi z}{2}=\pi\cot (\pi \frac{1-z}{2})=\frac{1}{2}\sum\limits_{n\in \mathbb{Z}}(\frac{1}{\frac{1-z}{2}+n}+\frac{1}{\frac{1-z}{2}-n})  \]
\[ =-\sum\limits_{n\in \mathbb{Z}} \left(\frac{1}{z-(2n+1)}+\frac{1}{z+(2n-1)}\right)=-\sum\limits_{n\in \mathbb{Z}} \left(\frac{1}{z-(2n+1)}+\frac{1}{z+(2n+1)}\right) \]
we have
\[ \frac{\pi}{\sin \pi z}=\frac{1}{2}\sum\limits_{n\in \mathbb{Z}}(-1)^n(\frac{1}{z+n}+\frac{1}{z-n})=z\sum\limits_{n\in \mathbb{Z}}\frac{(-1)^n}{z^2-n^2}, \]
then
\[ \frac{\pi}{\sinh \pi z}=\frac{i\pi}{\sin i\pi z}=z\sum\limits_{n\in \mathbb{Z}}\frac{(-1)^n}{z^2+n^2}. \]
It is easy to show
\[ \int_{0}^{\infty}\frac{\pi dt}{\sinh(\pi y \cosh t)}=\int_{0}^{\infty} y \cosh t\sum\limits_{n\in \mathbb{Z}}\frac{(-1)^n}{y^2\cosh^2t+n^2}dt\]
\[ =\int_{0}^{\infty} \sum\limits_{n\in \mathbb{Z}}\frac{(-1)^n}{y^2\sinh^2t+(y^2+n^2)}d(y\sinh t) \]
\[ =\sum\limits_{n\in \mathbb{Z}}\frac{(-1)^n}{\sqrt{y^2+n^2}}\arctan\frac{y\sinh t}{\sqrt{y^2+n^2}}|_{0}^{\infty} =\sum\limits_{n\in \mathbb{Z}}\frac{(-1)^n}{\sqrt{y^2+n^2}}\cdot\frac{\pi}{2}. \]
Let $s=\frac{1}{y^2}$, then we have $y\to+\infty$ as $s\to 0_+$, and
\[ S(s)=\sum\limits_{n=0}^{\infty} \frac{(-1)^n}{\sqrt{1+n^2s}}=\frac{1}{2}+\frac{1}{2}\sum\limits_{n\in \mathbb{Z}}\frac{(-1)^n}{\sqrt{1+n^2s}}=\frac{1}{2}+\frac{y}{2}\sum\limits_{n\in \mathbb{Z}}\frac{(-1)^n}{\sqrt{y^2+n^2}} \]
\[ =\frac{1}{2}+\frac{1}{\pi}\int_{0}^{\infty}\frac{\pi y dt}{\sinh(\pi y \cosh t)}. \]
\\
Since
\[ \frac{\pi y}{\sinh(\pi y \cosh t)}\to 0\]
as $y\to \infty$, and
\[ \frac{\pi y}{\sinh(\pi y \cosh t)}\le\frac{1}{\cosh t}\in L^1[0,\infty), \]
we have
\[ \int_{0}^{\infty}\frac{\pi y dt}{\sinh(\pi y \cosh t)}\to 0 \]
as $y\to\infty$ by dominated convergence theorem.
 
\end{proof}
 
下面这个问题又作何解呢?那汤松的实变函数论里面,证明傅里叶级数一些性质的时候,用到了这种级数。
 
Prove that $\lim\limits_{s\to 0+}\sum\limits_{n=1}^{\infty} (-1)^n(\frac{\sin ns}{ns})^2=-\frac{1}{2}$.\\ \\ \\ \\
 
 
好像没什么思路?算一下x²的傅里叶级数就行了。前面的证明几个例子基本上都依赖于一些恒等式,也就是说,每做一个这种类型的题目,就要找到一个恒等式。可万一找不到怎么办?比如多项式那个。
 
给一个题做一个题,效率太低。这就促使我们找到对这类问题一个统一的解法。
 
Suppose $a_s(x)=a(x,s)\in C([0,\infty)\times[0,1])$ and $a_s(x)\in C^1[0,\infty)$. Under the following conditions\\
$\text{(i)}a_0(x)=1,$\\
$\text{(ii)}a_s(\infty)=0 \text{ for } s>0,$\\
$\text{(iii)}\int_{0}^{\infty} |a_s'(x)-a_s'(x+1)|dx \to 0 \text{ as } s\to 0_+,$\\
we have
\[ \lim\limits_{s\to 0+}\sum\limits_{n=0}^{\infty} (-1)^na_s(n)=\frac{1}{2}. \]
 
\begin{proof}
     Since
\[ a_s(0)=\int_{0}^{\infty}a_s'(x)dx=\sum\limits_{n=0}^{\infty}\int_{n}^{n+1}a_s'(x)dx, \]
we have
\[ \sum\limits_{n=0}^{\infty} (-1)^na_s(n)=\sum\limits_{n=0}^{\infty} \left( a_s(2n)-a_s(2n+1) \right)=\sum\limits_{n=0}^{\infty}\int_{2n}^{2n+1}a_s'(x)dx \]
\[ =\frac{1}{2}a_s(0)+\sum\limits_{n=0}^{\infty}\int_{2n}^{2n+1}a_s'(x)dx-\frac{1}{2}\sum\limits_{n=0}^{\infty}\int_{n}^{n+1}a_s'(x)dx \]
\[ =\frac{1}{2}a_s(0)+\frac{1}{2}\sum\limits_{n=0}^{\infty}\int_{2n}^{2n+1} \left( a_s'(x)-a_s'(x+1) \right) dx. \]
Since $a_s(0)\to a_0(0)=1$ and
\[ |\sum\limits_{n=0}^{\infty}\int_{2n}^{2n+1} \left( a_s'(x)-a_s'(x+1) \right) dx|\le \sum\limits_{n=0}^{\infty}\int_{2n}^{2n+1} |a_s'(x)-a_s'(x+1)| dx\]
\[ \le \int_{0}^{\infty} |a_s'(x)-a_s'(x+1)|dx \to 0  \]
as $s\to 0_+$, we can get the conclusion.
 
\end{proof}
 
条件(iii)看起来有点奇奇怪怪的,那我们就把它换成一个弱一点但看起来很明了的条件。
 
Suppose $a_s(x)=a(x,s)\in C([0,\infty)\times[0,1])$ and $a_s(x)\in C^1[0,\infty)$. Under the following conditions\\
$\text{(i)}a_0(x)=1,$\\
$\text{(ii)}a_s(\infty)=0 \text{ for } s>0,$\\
$(\text{iii}')a_s(x) \text{ is a convex function },$\\
we have
\[ \lim\limits_{s\to 0+}\sum\limits_{n=0}^{\infty} (-1)^na_s(n)=\frac{1}{2}. \]
 
\begin{proof}
     Since $a_s(x)$ is a convex function, $a_s'(x)$ is increasing, we have
\[ \int_{0}^{\infty} |a_s'(x)-a_s'(x+1)|dx=\int_{0}^{\infty} a_s'(x+1)-a_s'(x)dx=a_s(0)-a_s(1)\to 1-1=0 \]
as $s\to 0_+$, it implys (iii).
\end{proof}
 
再看看北大的那道题,就成了上述结论的简单推论。
 
Suppose $f(x)>0$, $f''(x)\le 0$, and $f(\infty)=\infty$ on $[0,\infty)$, prove the following conclusion
\[ \lim\limits_{s\to 0+} \sum\limits_{n=0}^{\infty} \frac{(-1)^n}{f^s(n)}=\frac{1}{2}. \]
 
\begin{proof}
     If we take $a_s(x)=\frac{1}{f^s(x)}$, then (i), (ii) are trivial, and
\[ a_s''(x)=s\cdot \frac{(s+1)f'^2(x)-f''(x)f(x)}{f^{s+2}(x)}>0 \]
means $a_s(x)$ is convex.
\end{proof}
 
当然了,我们还可以稍微加强一点北大那题的结论,可自行检验这个要更强一点。
 
Suppose $f(x)>0$, $f''(x)\le 0$, and $f(\infty)=\infty$ on $[0,\infty)$, we have the following conclusion
\[ \lim\limits_{s\to 0+} \sum\limits_{n=0}^{\infty} (-1)^ne^{-sf(n)}=\frac{1}{2}. \]
 
\begin{proof}
     If we take $a_s(x)=e^{-sf(x)}$, then (i), (ii) are trivial, and
\[ a_s''(x)=s(sf'^2(x)-f''(x))e^{-sf(x)}\ge 0 \]
means $a_s(x)$ is convex.
\end{proof}
 
现在我们就有能力解决下面的问题了。
 
Suppose $f\in C^{m+1}[0,\infty)$ with $f>0$, $f^{(m+1)}\le 0$, where $m\ge 1$. If $f(\infty)=\infty$, prove the following conclusion
\[ \lim\limits_{s\to 0+} \sum\limits_{n=0}^{\infty} (-1)^ne^{-sf(n)}=\frac{1}{2}. \]
 
\begin{proof}
We have solved the case $m=1$ before, and we only consider $m\ge 2$. Without loss of generality, assume $f, f',\cdots, f^{(m)}>0$. Otherwise, we can replace $m$ by a smaller number. If we take $a_s(x)=\frac{1}{f^s(x)}$, then (i), (ii) are trivial, and
\[ a_s''(x)=s(sf'^2(x)-f''(x))e^{-sf(x)}. \]
By lemma, we have
\[ (\frac{f'^2}{f''})'\ge (1+\frac{1}{m-1}-\varepsilon)f'>0 \]
and
\[ \frac{f'^2}{f''}\ge (1+\frac{1}{m-1}-\varepsilon)f-C, \]
then $\frac{f''}{f'^2}$ is decreasing to $0$, so the number of roots of $a_s''(x)$ is no more than $1$. If $s$ is small enough, then there is $x_s$ s.t. $a_s''(x_s)=0$, i.e. $s=\frac{f''(x_s)}{f'^2(x_s)}$, and
\[ \int_{0}^{\infty} |a_s'(x)-a_s'(x+1)|dx=\int_{0}^{x_s} a_s'(x)-a_s'(x+1)dx+\int_{x_s}^{\infty} a_s'(x+1)-a_s'(x)dx\]
\[ =-a_s(0)+a_s(1)+2(a_s(x_s)-a_s(x_s+1)). \]
Now, we only need to prove that $a_s(x_s)-a_s(x_s+1)\to 0$ as $s\to 0_+$. Since $a_s(x_s+1)-a_s(x_s)=a_s'(\xi_s)$, where $\xi_s\in(x_s, x_s+1)$, we have
\[ |a_s(x_s+1)-a_s(x_s)|\le sf'(\xi_s). \]
Since
\[ f'(\xi_s)=f'(x_s)+\cdots+\frac{f^{(m)}(x_s)}{(m-1)!}(\xi_s-x_s)^{m-1}+\frac{f^{m+1}(\eta_s)}{m!}(\xi_s-x_s)^m \]
\[ \le f'(x_s)+\cdots+\frac{f^{(m)}(x_s)}{(m-1)!}<<f'(x_s) \]
and
\[ s=\frac{f''(x_s)}{f'^2(x_s)}<<\frac{1}{f(x_s)}, \]
we have $sf'(\xi_s)<<\frac{f'(x_s)}{f(x_s)}\to 0$ as $s\to 0_+$.
\end{proof}
 
这里才用了Vinogradov记号,$f<<g$来表示$|f|≤C|g|$对某个正常数$C$成立。如果把上述首一多项式改成指数函数,结论还成立吗?
 
Suppose $f(x)>0$, $f'''(x)\le 0$, and $f(\infty)=\infty$ on $[0,\infty)$, we have the following conclusion
\[ \lim\limits_{s\to 0+} \sum\limits_{n=0}^{\infty} (-1)^ne^{-sf(n)}=\frac{1}{2}. \]
 
\begin{proof}
If there is $x_0\ge 0$ s.t. $f''(x_0)\le 0$, then $f''(x)\le 0$ for $x\ge x_0$, we have solved this case. So we can assume $f''(x)>0$, and $f'(x)>0$. If we take $a_s(x)=e^{-sf(x)}$, then (i), (ii) are trivial, and
\[ a_s''(x)=s(sf'^2(x)-f''(x))e^{-sf(x)}. \]
Since $f''(x)$ is decreasing, and $f'(x)$ is strictly increasing, then $\frac{f''(x)}{f'^2(x)}$ is decreasing to $0$, so the number of roots of $a_s''(x)$ is no more than $1$.\\
If $a_s''(x)>0$ for all $x\ge 0$, then $a_s(x)$ is convex.\\
If there is $x_s$ s.t. $a_s''(x_s)=0$, i.e. $s=\frac{f''(x_s)}{f'^2(x_s)}$, then
\[ \int_{0}^{\infty} |a_s'(x)-a_s'(x+1)|dx=\int_{0}^{x_s} a_s'(x)-a_s'(x+1)dx+\int_{x_s}^{\infty} a_s'(x+1)-a_s'(x)dx\]
\[ =-a_s(0)+a_s(1)+2(a_s(x_s)-a_s(x_s+1)). \]
Since $a_s(x_s+1)-a_s(x_s)=a_s'(\xi_s)$, where $\xi_s\in(x_s, x_s+1)$, we have
\[ |a_s(x_s+1)-a_s(x_s)|\le sf'(\xi_s). \]
Since $f'(\xi_s)-f'(x_s)=f''(\eta_s)(\xi_s-x_s)\le f''(0)$, where $\eta_s \in(x_s, \xi_s)$, and
\[ f'(x_s)=\sqrt{\frac{f''(x_s)}{s}}\le \sqrt{\frac{f''(0)}{s}}, \]
we have
\[ |a_s(x_s+1)-a_s(x_s)|\le s(f''(0)+\sqrt{\frac{f''(0)}{s}})=sf''(0)+\sqrt{sf''(0)}\to 0\]
as $s\to 0_+$.
\end{proof}
 
也许你觉得这个应该也是成立的,但可惜的是,Hardy有个结论却说极限一定不存在。
 
Prove that
$$\lim\limits_{s\to 0+}\sum\limits_{n=0}^{\infty} \frac{(-1)^n}{n!^s}=\frac{1}{2}. $$
 
 
Lemma: Suppose $f\in C^{m+1}[0,\infty)$ and $f, f',\cdots, f^{(m)}>0$, $f^{(m+1)}\le 0$, where $m\ge 1$. Prove that
$$\limsup\limits_{x\to +\infty} \frac{f(x)f''(x)}{f'^2(x)}\le 1-\frac{1}{m}.$$
 
\begin{proof}
We can deal with this problem by induction. The case $m=1$ is trivial, so we can assume $m\ge 2$. Since
\[ f(x)=f(0)+f'(0)x+\frac{f''(\xi)}{2}x^2\ge f'(0)x, \]
we can get $f(x)\to\infty$ as $x\to \infty$. Fixed $\varepsilon>0$, we have
\[ \frac{f'f'''}{f''^2}\le 1-\frac{1}{m-1}+\varepsilon \]
for $x\ge x_0$ by induction. Since
\[ (\frac{f'^2}{f''})'=\frac{2f'f''^2-f'^2f'''}{f''^2}=(2-\frac{f'f'''}{f''^2})f'\ge (1+\frac{1}{m-1}-\varepsilon)f', \]
we have
\[ \frac{f'^2}{f''}\ge (1+\frac{1}{m-1}-\varepsilon)f-C, \]
that means
\[ \frac{ff''}{f'^2}\le \frac{f}{(1+\frac{1}{m-1}-\varepsilon)f-C}\to \frac{1}{1+\frac{1}{m-1}-\varepsilon} \]
as $x\to\infty$. Let $\varepsilon\to 0$, we can obtain
$$\limsup\limits_{x\to +\infty} \frac{ff''}{f'^2}\le 1-\frac{1}{m}.$$
\end{proof}
 
这里才用了Vinogradov记号,$f<<g$来表示$|f|≤C|g|$对某个正常数$C$成立。如果把上述首一多项式改成指数函数,结论还成立吗?
 
Suppose $f\in C^{m+1}[0,\infty)$ with $f>0$, $f^{(m+1)}\le 0$, where $m\ge 1$. If $f(\infty)=\infty$, prove the following conclusion
\[ \lim\limits_{s\to 0+} \sum\limits_{n=0}^{\infty} \frac{(-1)^n}{f^s(n)}=\frac{1}{2}. \]
 
\begin{proof}
Without loss of generality, assume $f, f',\cdots, f^{(m)}>0$. Otherwise, we can replace $m$ by a smaller number. If we take $a_s(x)=\frac{1}{f^s(x)}$, then (i), (ii) are trivial, and
\[ a_s''(x)=s\cdot \frac{(s+1)f'^2(x)-f''(x)f(x)}{f^{s+2}(x)}. \]
By lemma, we can get $a_s''(x)>0$ for $x\ge x_0$, that means $a_s(x)$ is convex.
\end{proof}
 
现在我们就有能力解决有关首一多项式的那道题了。
 
Suppose $p(x)=x^m+a_{m-1}x^{m-1}+...+a_1x+a_0$ is a monic polynomial with degree $m\ge 1$, prove that
\[ \lim\limits_{s\to 0+}\sum\limits_{n=0}^{\infty} (-1)^ne^{-sp(n)}=\frac{1}{2}. \]
 
\begin{proof}
     Let $a_s(x)=e^{-sp(x)}$, then (i), (ii) are trivial. We only need to show that
\[ \lim\limits_{s\to 0+}\sum\limits_{n=2N}^{\infty} (-1)^ne^{-sp(n)}=\frac{1}{2} \]
for some positive integer $N$, or equivalent
\[ \lim\limits_{s\to 0+}\sum\limits_{n=0}^{\infty} (-1)^ne^{-sp(n+2N)}=\frac{1}{2}, \]
so we can consider $p(x+2N)$ instead of $p(x)$. Without loss of generality, we can assume the coefficients of $p(x)$ are positive.
 
Since $a_s'(x)=-sp'(x)e^{-sp(x)}<<sx^{m-1}e^{-sx^m}$, we have $a_s(x)-a_s(x+1)=-a_s'(\xi)<<sx^{m-1}e^{-sx^m}$, where $\xi\in (x,x+1)$. Then
\[ \int_{0}^{\infty} |sp'(x)e^{-sp(x)}-sp'(x)e^{-sp(x+1)}|dx<<s^2\int_{0}^{\infty} x^{2(m-1)}e^{-sx^m}dx\]
\[ =s^{\frac{1}{m}}\int_{0}^{\infty} x^{2(m-1)}e^{-x^m}dx \to 0 \]
as $s\to 0_+$. Since $p'(x)-p'(x+1)<<x^{m-2}$, we have
\[ \int_{0}^{\infty} |sp'(x)e^{-sp(x+1)}-sp'(x+1)e^{-sp(x+1)}|dx<<s\int_{0}^{\infty} x^{m-2}e^{-sx^m}dx\]
\[ =s^{\frac{1}{m}}\int_{0}^{\infty} x^{m-2}e^{-x^m}dx \to 0 \]
as $s\to 0_+$. That means (iii).
\end{proof}
 
 
Suppose $f\in C^1[0,\infty)$, $f'\in L^1[0,\infty)$, and $f(0)=1$, $f(\infty)=0$, then we have
\[ \lim\limits_{s\to 0+}\sum\limits_{n=0}^{\infty} (-1)^nf(ns)=\frac{1}{2}. \]
 
\begin{proof}
     Let $a_s(x)=f(sx)$, then (i), (ii) are trivial. Since $a_s'(x)=sf'(sx)$, we have
\[ \int_{0}^{\infty} |a_s'(x)-a_s'(x+1)|dx=s\int_{0}^{\infty} |f'(sx)-f'(s(x+1))|dx\]
\[ =\int_{0}^{\infty} |f'(x)-f'(s+x)|dx=||f'-f_s'||_{L^1} \to 0 \]
as $s\to 0_+$.
\end{proof}
 
这个结论简洁又好用。随便取一个函数就可以出一道题,拿出来坑人真是不亦乐乎。
 
Let $f(x)=\frac{1}{\sqrt{1+x^2}}, (\frac{\sin x}{x})^2, 2(\frac{1}{x}-\frac{1}{e^x-1})$ and so on, then we have
\[ \lim\limits_{s\to 0+}\sum\limits_{n=0}^{\infty} (-1)^nf(ns)=\frac{1}{2}. \]\\
 
下面说明在某种意义下,可导的条件是不能必不可少的。这里的条件要求导数可积,如果说函数是单调递减的,那么导数自然是可积的,所以对于此类函数而言,连续可微似乎是多于的条件?然而并不是这样。
 
Suppose $F(x)$ is the Cantor-Lebesgue function, let $f(x)=F(1-x)$, then it is obvious that
\[ \sum\limits_{n=0}^{\infty} (-1)^nf(ns)=1\]
for $s=\frac{1}{3^n}$, and
\[ \sum\limits_{n=0}^{\infty} (-1)^nf(ns)=\frac{1}{2} \]
for $s=\frac{2}{3^n}$, where $n\ge 1$.\\
 
然后就猜测会不会对于Cantor-Lebesgue函数而言,对任意s,无穷级数的值都不小于1/2?感觉很有可能是对的,但是费了半天劲没证出来,拿给小罗他也没证出来。最后发现8/45的地方,级数的值是小于1/2的。行吧,竟然有反例。其实我当时的第一想法没想到Cantor-Lebesgue函数,而是强行构造了另一个函数。
 
Suppose $g(x), 0\le x\le 1$ is the unique continuous solution of the following equation
 
$ g(x)=\frac{1}{4}f(2x), 0\le x\le\frac{1}{2},$\\
 
$g(x)=\frac{1}{4}+\frac{3}{4}f(2x-1), \frac{1}{2}\le x\le 1.$\\
or equivalent,
 
\[ g: \sum\limits_{n=1}^{\infty} \frac{a_n}{2^n}\mapsto  \sum\limits_{n=1}^{\infty} \frac{a_n 3^{S_{n-1}}}{4^n} \]
where $a_n=0, 1$, and $S_n=\sum\limits_{i=1}^{n} a_i$, $S_0=0$. Let $f(x)=g(1-x)$, then it is obvious that
 \[ \sum\limits_{n=0}^{\infty} (-1)^nf(ns)=\frac{3}{4} \]
for $s=\frac{1}{2^n}$, where $n\ge 1$.\\
 
对于这个函数而言,会不会极限刚好等于3/4呢?肯定不可能,猜也能猜到极限存在肯定就是1/2。
 
Suppose $f\in C[0,\infty)$ is an decreasing function with $f(0)=1$ and $f(\infty)=0$. If the limit
\[  \lim\limits_{s\to 0+}\sum\limits_{n=0}^{\infty} (-1)^nf(ns)=\lambda \]
exists, then $\lambda=\frac{1}{2}$.
 
其实呢,我们有下述更一般些的结论。
 
Suppose $f\in C[0,\infty)$ is an increasing function with $f(0)=1$ and $f(\infty)=0$, then
\[  \liminf\limits_{s\to 0+}\sum\limits_{n=0}^{\infty} (-1)^nf(ns)\le\frac{1}{2}\le \limsup\limits_{s\to 0+}\sum\limits_{n=0}^{\infty} (-1)^nf(ns).\]
 
\begin{proof}
     Let $f_1(x)=\int_{0}^{1}f(xt)dt=\frac{1}{x}\int_{0}^{x} f(t)dt$, then we can get the conclusion.
\end{proof}
 
再积分一次,还可以把连续弱化为在0处连续,结论也是一样的。槊神说我的做法跟他不谋而合。
 
如果函数导数不可积但是二阶导可积呢?比如下面这个例子?
 
Let $f(x)=\frac{\sin\sqrt{x}}{\sqrt{x}}$, then we have
\[ \lim\limits_{s\to 0+}\sum\limits_{n=0}^{\infty} (-1)^nf(ns)=\frac{1}{2}. \]
 
幸好,我们也有类似的结论,其证明也都是很类似的。
 
Suppose $f\in C^2[0,\infty)$, $f''\in L^1[0,\infty)$, and $f(0)=1$, $f(\infty)=0$, then we have
\[ \lim\limits_{s\to 0+}\sum\limits_{n=0}^{\infty} (-1)^nf(ns)=\frac{1}{2}. \]
\begin{proof}
     Since $f''\in L^1[0,\infty)$, $f'(\infty)$ exists, then we have $f'(\infty)=0$ by $f(\infty)=0$. Let $f_s(x)=f(sx)$, by Euler-Maclaurin formula, we have
\[ \sum\limits_{n=0}^{N} f_s(n)=\int_{0}^{N} f_s(x)dx+\frac{f_s(0)+f_s(N)}{2}+\frac{f_s'(0)+f_s'(N)}{12}-\frac{1}{2}\int_{0}^{N} B_2(x)f_s''(x)dx. \]
Since
$f_s(0)=f(0)=1$, $f_s'(x)=sf'(sx)$, $f_s''(x)=s^2f''(sx)$,
\[ \int_{0}^{N} f_s(x)dx=\frac{1}{s}\int_{0}^{sN}f(x)dx, \]
\[\int_{0}^{N} B_2(x)f_s''(x)dx=s^2\int_{0}^{N} B_2(x)f''(sx)dx=s\int_{0}^{sN} B_2(\frac{x}{s})f''(x)dx \]
we have
\[ \sum\limits_{n=0}^{N} f_{2s}(n)=\frac{1}{2s}\int_{0}^{2sN}f(x)dx+\frac{1+f(2sN)}{2}+\frac{sf'(0)+sf'(2sN)}{6} \]
\[ -s\int_{0}^{2sN} B_2(\frac{x}{2s})f''(x)dx\]
and
\[ \sum\limits_{n=0}^{2N} f_{s}(n)=\frac{1}{s}\int_{0}^{2sN}f(x)dx+\frac{1+f(2sN)}{2}+\frac{sf'(0)+sf'(2sN)}{12} \]
\[ -\frac{s}{2}\int_{0}^{2sN} B_2(\frac{x}{s})f''(x)dx\]
Since $f''\in L^1[0,\infty)$, we can get $f'$ is bounded.
Thus,
\[ \sum\limits_{n=0}^{2N} (-1)^nf(ns)=2\sum\limits_{n=0}^{N} f(2ns)-\sum\limits_{n=0}^{2N} f(ns)=2\sum\limits_{n=0}^{N} f_{2s}(n)-\sum\limits_{n=0}^{2N} f_s(n) \]
\[ =\frac{1+f(2sN)}{2}+\frac{sf'(0)+sf'(2sN)}{4}-2s\int_{0}^{2sN} B_2(\frac{x}{2s})f''(x)dx \]
\[ +\frac{s}{2}\int_{0}^{2sN} B_2(\frac{x}{s})f''(x)dx\]
Let $N\to \infty$, we have
\[ \sum\limits_{n=0}^{\infty} (-1)^nf(ns)=\frac{1}{2}+s\frac{f'(0)}{4}-2s\int_{0}^{\infty} B_2(\frac{x}{2s})f''(x)dx+\frac{s}{2}\int_{0}^{\infty} B_2(\frac{x}{s})f''(x)dx \]
Since $B_2(x)$ is bounded, we can get $\sum\limits_{n=0}^{\infty} (-1)^nf(ns)=\frac{1}{2}+O(s)$.
\end{proof}
 
有兴趣还可以得到高阶导可积对应的情形。最后再考虑一个例子,它的任意阶导数不可积,所以我们只能截断一下。
 
Let $f(x)=\frac{\sin x}{x}$, then we have
\[ \lim\limits_{s\to 0+}\sum\limits_{n=0}^{\infty} (-1)^nf(ns)=\frac{1}{2}. \]
 
\begin{proof}
     Let $f_N(x)=f(x)\chi_{[0,N\pi]}$, it is not difficult to show that
\[ \lim\limits_{s\to 0+}\sum\limits_{n=0}^{\infty} (-1)^nf_N(ns)=\frac{1}{2}, \]
then we have
\[ \limsup\limits_{s\to 0+}|\sum\limits_{n=0}^{\infty} (-1)^nf(ns)-\frac{1}{2}|=\limsup\limits_{s\to 0+}|\sum\limits_{ns>N\pi}^{\infty} (-1)^nf(ns)|  \]
\[ =\limsup\limits_{s\to 0+}|\sum\limits_{ns>N\pi}^{\infty} \frac{(-1)^n\sin ns}{ns}| \]
By Abel formula, take $a_n=(-1)^n\sin ns=\sin n(\pi+s)$, $b_n=\frac{1}{ns}$, we have
\[ |\sum\limits_{ns>N\pi}^{\infty} \frac{(-1)^n\sin ns}{ns}|\le \frac{A}{N\pi}. \]
Let $N\to \infty$, we can get the conclusion.
\end{proof}
 
证明过程中用到了Abel分部求和公式。
 
\[ \lim\limits_{s\to 0+}\sum\limits_{n=0}^{\infty} (-1)^ne^{-se^n}=\frac{1}{2}? \]
 
也许你觉得这个应该也是成立的,但可惜的是,Hardy有个结论却说极限一定不存在。
 
 
For any $a>1$, the limit
\[ \lim\limits_{x\to 1-}\sum\limits_{n=0}^{\infty} (-1)^nx^{a^n} \]
does not exist.
 
也就是说,我们不能让指数上的部分增长的太快。其实我也忘了Hardy这个结论咋证的了,就不贴证明了。。。
 
当然了,也可以考虑x的傅里叶级数。所以做一些题目的时候千万别被套路了!就像Dirichlet有个定理是说(a,q)=1,那么等差数列a+nq中有无穷个素数一样,经常被拿来出题,比如4n+1,6n+5这种,要是不想想根本原因,不知道要被坑多少次。
 
这类例子很多,希望能起到抛砖引玉的作用,以后看问题能看到更深刻的地方,别老是被人家给套路了。如果想不到,更甚说不愿去想更深层的东西,庸俗的人生大多都相仿。
 
(2018年中科大考研题) Suppose $a_n>0$ and
\[ \left| \sum\limits_{n=1}^{\infty} \frac{\sin (a_nx)}{n^2} \right|\le |\tan x| \]
for $x\in (-1,1)$, prove that $a_n=o(n^2)$ as $n\to\infty$.
\begin{proof}
Since $\sum\limits_{n=1}^{\infty} \cfrac{\sin (a_nx)}{n^2}$ is uniform convergence, we have
\[ \int_{0}^{t}\sum\limits_{n=1}^{\infty} \frac{\sin (a_nx)}{n^2}dx=\sum\limits_{n=1}^{\infty} \frac{1}{n^2}\int_{0}^{t} \sin (a_nx)dx=\sum\limits_{n=1}^{\infty} \frac{1-\cos(a_nt)}{n^2a_n} \]
and
\[ \int_{0}^{t} \tan x dx=-\ln\cos t, \]
then we can get
\[ \sum\limits_{n=1}^{\infty} \frac{1-\cos(a_nt)}{n^2a_n}\le -\ln\cos t  \]
for $t\in (0,1)$, so that
\[ 2\sum\limits_{n=1}^{N} \frac{1-\cos(a_nt)}{n^2a_nt^2}\le -\frac{2\ln\cos t}{t^2}  \]
for any $N\in\mathbb{N}$. Let $t\to 0_+$, then
\[ \sum\limits_{n=1}^{N} \frac{a_n}{n^2}\le 1. \]
Let $N\to\infty$, it is obvious that
\[ \sum\limits_{n=1}^{\infty} \frac{a_n}{n^2}\le 1. \]
\end{proof}

第一届熊赛分析与方程部分试题

\documentclass[11pt,a4paper]{article}
\RequirePackage{xeCJK}
%\usepackage{amsmath,amssymb,amsthm}
%%%修改数学字体
\usepackage{fontspec}
\usepackage[T1]{fontenc}
\usepackage{times}
\usepackage[complete,amssymbols,amsbb,eufrak,nofontinfo,
             subscriptcorrection,zswash,mtpscr]{mtpro2}
 
%\usepackage{mathpazo}
%\renewcommand{\rmdefault}{ibh}
%\usepackage{fourier}
%\usepackage{charter}
%\usepackage{helvet}
\usepackage{amsmath,amsthm}
%%%修改数学字体
%\usepackage{CJKnumb}
 
\setCJKmainfont[BoldFont={方正黑体简体},ItalicFont={方正楷体简体}]{方正书宋简体}
%\setCJKsansfont[BoldFont={黑体}]{方正中等线简体}
%\setCJKmonofont{方正中等线简体}
 
\setCJKfamilyfont{kd}{华文行楷}
 
%%%%%%%%%%%%%%%%%%%%%%%%%%%%%%%%%%%%%%%%%%%%%%
 
\newcommand{\chaoda}{\fontsize{55pt}{\baselineskip}\selectfont}
\newcommand{\chuhao}{\fontsize{42pt}{\baselineskip}\selectfont}     % 字号设置
\newcommand{\xiaochuhao}{\fontsize{36pt}{\baselineskip}\selectfont} % 字号设置
\newcommand{\yihao}{\fontsize{28pt}{\baselineskip}\selectfont}      % 字号设置
\newcommand{\erhao}{\fontsize{21pt}{\baselineskip}\selectfont}      % 字号设置
\newcommand{\xiaoerhao}{\fontsize{18pt}{\baselineskip}\selectfont}  % 字号设置
\newcommand{\sanhao}{\fontsize{15.75pt}{\baselineskip}\selectfont}  % 字号设置
\newcommand{\xiaosanhao}{\fontsize{15pt}{\baselineskip}\selectfont} % 字号设置
\newcommand{\sihao}{\fontsize{14pt}{\baselineskip}\selectfont}      % 字号设置
\newcommand{\xiaosihao}{\fontsize{12pt}{14pt}\selectfont}           % 字号设置
\newcommand{\wuhao}{\fontsize{10.5pt}{12.6pt}\selectfont}           % 字号设置
\newcommand{\xiaowuhao}{\fontsize{9pt}{11pt}{\baselineskip}\selectfont}   % 字号设置
\newcommand{\liuhao}{\fontsize{7.875pt}{\baselineskip}\selectfont}  % 字号设置
\newcommand{\qihao}{\fontsize{5.25pt}{\baselineskip}\selectfont}    % 字号设置
 
 
 
 
 
\usepackage{ifthen}
\usepackage{eso-pic}
%\usepackage{esvect}
\usepackage{graphicx}
\usepackage{color}
%\usepackage[table,dvipsnames,svgnames]{xcolor}
\colorlet{backg}{green!20}
 
\usepackage{tikz}
\usepackage{makeidx}
\usepackage{enumitem}
\usepackage[left=2.6cm,right=2.6cm,top=2.54cm,bottom=2.54cm]{geometry}
\usepackage{mathtools}
\usepackage{mathrsfs}
 
\usepackage{multirow,booktabs}
\usepackage{makecell}
%\title{中国科学技术大学\\
%2015年硕士学位研究生入学考试试题}
%\author{(线性代数与解析几何)}
%\date{}
 
\newenvironment{newproof}{\par\CJKfamily{xiao}\noindent{\makebox[0pt][r]{\;\;}\textbf{证明:}}\color{black!90}\small}{\hfill$\Box$\quad\par}
 
\newenvironment{solution}{\par\CJKfamily{xiao}\noindent{\makebox[0pt][r]{\;\;}\textbf{解:}}\small}{\hfill$\Box$\quad\par}
 
\newcommand{\Ker}{\mathrm{Ker}\,}
 
\usepackage{fancyhdr}
\usepackage{fancybox}
\usepackage{lastpage}%此宏包是获取总页数用的。
\pagestyle{fancy}
\renewcommand{\headrulewidth}{0pt}%设置页眉线
\renewcommand{\footrulewidth}{1pt}%设置页脚线
 
\fancyhf{}%清除所有页眉页脚
 
\lfoot{科目名称:分析与方程}
\cfoot{}
\rfoot{第\thepage 页\quad 共\pageref{LastPage} 页}
 
 
 
 
\begin{document}
%\maketitle
 
%\fancypage{%
%\setlength{\fboxsep}{13pt}%
%\setlength{\fboxrule}{0.8pt}%
%\setlength{\shadowsize}{0pt}%
%\shadowbox}{}
 
\begin{center}
{\erhao \CJKfamily{kd}{第一届Xionger网络数学竞赛} }\\
\vspace{0.3cm}
{\sanhao \textbf{分析与方程部分试题解答} }\\
\vspace{0.2cm}
{\sihao 整理编辑: 马明辉 \qquad
2018年6月8日}
%{\sanhao \textbf{解答:Eufisky (Xiongge)}}\\
\end{center}
%\textbf{分析与代数:}
%满分为100分,考试时间为120分钟,答案必须写在答题纸上\\
%\vspace*{-0.05cm}
\rule{\textwidth}{0.5mm}
%\rule{\textwidth}{0.4pt}
%\vspace*{-0.05cm}
%\boxed{\text{\textbf{注意:答案必须写在答题纸上,写在试卷或草稿纸上均无效。}}}
 
 
 
 
\begin{enumerate}
%\renewcommand{\labelenumi}{\textbf{{\theenumi}.}}
% 重定义第一级计数显示
\renewcommand{\theenumi}{\textbf{\arabic{enumi}.}}
\renewcommand{\labelenumi}{\theenumi}
 
% 重定义第二级计数显示
\renewcommand{\theenumii}{\alph{enumii}}
\renewcommand{\labelenumii}{(\theenumii)}
 
 
\item Suppose $S_n=a_1+\cdots+a_n$, and $T_n=S_1+\cdots+S_n$. If $\lim\limits_{n\to\infty} \frac{1}{n}T_n=s$ and $na_n$ is bounded, then $\lim\limits_{n\to\infty} S_n=s.$
\begin{proof}
We may assume that $\lim\limits_{n\to\infty} \frac{1}{n}T_n=0$, or we can replace $a_1$ by $a_1-s$. For $\forall \varepsilon>0$, there is a large number $N$, $\forall n\ge N$, we have $|T_n|\le \varepsilon n.$ Since
$$T_{n+k}-T_n=S_{n+1}+\cdots+S_{n+k}$$
$$=kS_n+(ka_{n+1}+(k-1)a_{n+2}+\cdots+2a_{n+k-1}+a_{n+k}),$$
we have
$$
kS_n=T_{n+k}-T_n-(ka_{n+1}+(k-1)a_{n+2}+\cdots+2a_{n+k-1}+a_{n+k}).
$$
Suppose $|na_n|\le C$, then
$$k|S_n|\le (2n+k)\varepsilon+C\left(\frac{k}{n+1}+\frac{k-1}{n+2}+\cdots+\frac{1}{n+k}\right)\le (2n+k)\varepsilon+C\frac{k^2}{n},$$
or equivalent,
$$|S_n|\le \left(\frac{2n}{k}+1\right)\varepsilon+C\frac{k}{n}.$$
Take $k=[\sqrt{\varepsilon}n]>\frac{1}{2}\sqrt{\varepsilon}n$, then we have
$$|S_n|\le \left(\frac{4}{\sqrt{\varepsilon}}+1\right)\varepsilon+C\sqrt{\varepsilon}=\left(4+\sqrt{\varepsilon}+C\right)\sqrt{\varepsilon}.$$
\end{proof}
 
%\begin{solution}
 
%\end{solution}
 
\item Suppose $f$ is a real value function on $\mathbb{R}$, and $f(x+y)=f(x)+f(y)$ for $\forall x, y\in \mathbb{R}$. If the set $\{(x,f(x)): x\in\mathbb{R}\}$ is not dense in $\mathbb{R}^2$, then $f$ is continuous.
\begin{proof}
In fact, we have $f(x)\equiv f(1)x$. It is not hard to see $f(rx)=rf(x)$ for $x\in\mathbb{R}, r\in \mathbb{Q}$. We denote $c=f(1)$, then $f(r)=cr$ for $r\in \mathbb{Q}$. If $f(x)\equiv cx$ is false, then there exists a number $x_0\in \mathbb{R}-\mathbb{Q}$, s.t. $y_0=f(x_0)\ne cx_0$. Then we have
$$f(x_0s+r)=y_0s+cr=c(x_0s+r)+(y_0-cx_0)s, \forall s, t\in \mathbb{Q}.$$
Fix $y\in\mathbb{R}$. For $\forall \varepsilon>0$, there is a number $s\in\mathbb{Q}$, s.t. $|s(y_0-cx_0)-y|<\varepsilon$, then $|s|< \frac{|y|+\varepsilon}{|y_0-cx_0|}\le \frac{|y|+1}{|y_0-cx_0|}=C$. Take $r\in\mathbb{Q}$ s.t. $|x_0+r|<\varepsilon$, we have
$$f(x_0+r)=c(x_0+r)+(y_0-cx_0),$$
and
$$f(s(x_0+r))=sc(x_0+r)+s(y_0-cx_0),$$
then $|s(x_0+r)|\le C\varepsilon$ and
$$|f(s(x_0+r))-y|\le |sc(x_0+r)|+|s(y_0-cx_0)-y|< (Cc+1)\varepsilon.$$
Fix $(x,y)\in\mathbb{R}^2$. For $\forall \varepsilon>0$, there exists a number $a\in \mathbb{R}$ s.t. $|a|<\varepsilon$, and $|f(a)-(y-cx)|< \varepsilon$. Take $r\in \mathbb{Q}$ s.t. $|x-r|<\varepsilon$, we have
$$f(a+r)=f(a)+cr=f(a)+cx+c(r-x)$$
then $|(a+r)-x|<2\varepsilon$ and
$$|f(a+r)-y|=|f(a)-(y-cx)|+|c(r-x)|< (1+c)\varepsilon.$$
It is a contradiction.
\end{proof}
 
 
 
\item Suppose $\Omega\subset \mathbb{C}$ is a domain, and $u_n=Re f_n$, where $f_n\in H(\Omega)$. If $\{u_n\}$ is uniform convergence on arbitrary compact subset of $\Omega$, and $\{f_n(z_0)\}$ is convergence for some $z_0\in \Omega$. Prove that $\{f_n\}$ is uniform convergence on arbitrary compact subset of $\Omega$.
\begin{proof}
Since the compact set has a finite open covering property, we only need to consider the case $\Omega=\mathbb{D}$. We can obtain the conclusion by the Borel-Carath$\acute{\text{e}}$odory lemma: Suppose $f\in H(\mathbb{D})$. Let $M(r)=\max\limits_{|z|=r} |f(z)|$, $A(r)=\max\limits_{|z|=r} Re f(z)$, then for $0<r<R<1$, we have
$$M(r)\le \frac{2r}{R-r}A(R)+\frac{R+r}{R-r}|f(0)|.$$
\end{proof}
 
 
\item  Suppose $\Omega\subset \mathbb{C}$ is a convex domain, and $f\in H(\Omega)$. If $Re f'(z)\ge 0$ for $\forall z\in\Omega$ and $f$ is not constant function, then $f$ is injective.
\begin{proof}
Consider $e^{-f'(z)}\in H(\Omega)$, then $|e^{-f'(z)}|=e^{-Re f'(z)}\le 1$. If $Re f'(z_0)=0$ for some $z_0\in\Omega$, then $e^{-f'(z)}=const$, or $f'(z)=const$ by the maximum principle. That is to say $f(z)=cz$ with $c\ne 0$, then $f$ is injective. If $Re f'(z)> 0$ for $\forall z\in\Omega$, then for $\forall z_1, z_2\in\Omega$ with $z_1\ne z_2$, we have
$$f(z_2)-f(z_1)=\int_{z_1}^{z_2}f'(z)dz=(z_2-z_1)\int_{0}^{1} f'(z_1+t(z_2-z_1))dt,$$
then
$$Re\frac{f(z_2)-f(z_1)}{z_2-z_1}=\int_{0}^{1} Re f'(z_1+t(z_2-z_1))dt>0.$$
\end{proof}
 
%\begin{enumerate}[itemsep=0pt,parsep=0pt,label=(\arabic*)]
%\item
%\end{enumerate}
 
%\begin{newproof}
 
%\end{newproof}
 
 
 
 
\item  Prove the linear span of $t^ne^{-t}, n=0,1,2,\cdots$ is dense in $L^2(0,\infty)$.
\begin{proof}
Let $M$ be the closed linear span of $t^ne^{-t}, n=0,1,2,\cdots$. Take any $\varphi\in M^{\bot}$, we have
$$\int_{0}^{\infty} t^ne^{-t}\varphi(t)dt=0, n=0,1,2,\cdots.$$
Let $z$ be a complex with $\Im\ z>-1$, and
$$f(z)=\int_{0}^{\infty} e^{izt}e^{-t}\varphi(t)dt.$$
Since $|\frac{e^z-1}{z}|\le C\max\{1,e^{|z|}\}$, we have
$$f'(z)=\lim\limits_{h\to 0}\frac{f(z+h)-f(z)}{h}=\lim\limits_{h\to 0} \int_{0}^{\infty} \frac{e^{iht}-1}{h}e^{izt}e^{-t}\varphi(t)dt$$
$$=\int_{0}^{\infty} ite^{izt}e^{-t}\varphi(t)dt$$
by the dominated convergence theorem. That means $f$ is analytic. Similarly, we have
$$f^{(n)}(z)=\int_{0}^{\infty} i^nt^ne^{izt}e^{-t}\varphi(t)dt.$$
Since $f^{(n)}(0)=i^n\int_{0}^{\infty} t^ne^{-t}\varphi(t)dt=0, n=0,1,2,\cdots$, we have $f(z)\equiv 0$, that means $e^{izt}e^{-t}\in M$. According to the Weierstrass approximation theorem, every continuous periodic function $h(t)$ is the uniform limit of trigonometric polynomials, we can get $h(t)e^{-t}\in M$. Let $g(t)$ be a continuous function with compact support, and $g_1(t)=g(t)e^t$. Denote by $h(t)$ a $T$ periodic function such that
$$h(t)\equiv g_1(t), t\in [0,T],$$
where $T$ is large enough so that the support of $g_1(t)$ is contained in the interval $[0,T]$. Then
$$|g_1(t)-h(t)|\le ||g_1||_{L^{\infty}}\chi_{(T,\infty)}(t),$$
so that
$$|g(t)-h(t)e^{-t}|\le ||g_1||_{L^{\infty}}e^{-t}\chi_{(T,\infty)}(t).$$
Let $T\to\infty$, we can get $g(t)\in M$. Since the set of all continuous functions with compact support is dense in $L^2(0,\infty)$, we have $M=L^2(0,\infty)$.
\end{proof}
 
 
\item Let  $A$ is a unital commutative Banach algebra that is generated by $\{1,x\}$ for some $x\in A$. Then the complement set of $\sigma(x)$ is connected.
\begin{proof}
Let us decompose $\sigma(x)^c$ into its connected components, obtaining an unbounded component $\Omega_{\infty}$ together with a sequence of holes $\Omega_1, \Omega_2, \cdots,$
$$\sigma(x)^c=\Omega_{\infty}\cup\Omega_1\cup\Omega_2\cup\cdots.$$
Let $\Omega=\Omega_1\cup\Omega_2\cup\cdots$. If $\sigma(x)^c$ is not connected, the $\Omega\ne \emptyset$. Suppose $\lambda\in\Omega$, then for arbitrary polynomial $p(z)$, since $p(z)$ is analytic, we have
$$|p(\lambda)|\le \max_{z\in \sigma(x)} |p(z)|=\max\limits_{\omega\in Sp(A)} |\omega(p(x))|\le ||p(x)||$$
by the maximum principle and Gelfand theorem. If we defind
$$\omega: p(x)\mapsto p(\lambda),$$
then $\omega$ is bounded on $\{p(x)\}$. Since $\{p(x)\}$ is dense in $A$, $\omega$ have unique extension on $A$, and $\omega(xy)=\omega(x)\omega(y)$, that means $\omega\in Sp(A)$. Then $\lambda=\omega(x)\in\sigma(x)$, it is a contradiction.
\end{proof}
 
 
\item Suppose $X$ is a compact  Hausdorff space. $\Omega$ is a family of colsed connected subset of $X$, and $\Omega$ is totally order with respect to inclusion relation. Then $Y=\cap\{A: A\in\Omega\}$ is connected.
\begin{proof}
If $Y$ is not connected, then are open set $B$ and $C$, with $B\cap C=\emptyset$, $B\cap Y\ne \emptyset$ and $C\cap Y\ne \emptyset$. Consider the set $Y_1=\cap\{A-(B\cup C): A\in\Omega\}$, then $Y_1=Y-(B\cup C)=\emptyset$. Since $A$ is connected, if $A-(B\cup C)=\emptyset$, or $A\subset B\cup C$, then $A\subset B$, or $A\subset C$, it is impossible. Thus $A-(B\cup C)\ne\emptyset$. Since $A-(B\cup C)$ is compact, and finite intersection is not empty, then $Y_1\ne\emptyset$. It is a contradiction.
\end{proof}
 
\item Suppose the measurable set $A\subset \mathbb{R}$ with $0<m(A)<\infty$. Let $f(x,r)=m(A\cap[x-r,x+r])/2r$, then there exists $x\in\mathbb{R}$ s.t.
$$0<\liminf\limits_{r\to 0_+}f(x,r)\le \limsup\limits_{r\to 0_+}f(x,r)<1.$$
\begin{proof}
Since $0<m(A)<\infty$, there are interval $I_1, I_2$ with $|I_1|=|l_2|=2r_0$ s.t. $m(A\cap I_1)>\frac{1}{2}|I_1|$, $m(A\cap I_2)<\frac{1}{2}|I_2|$.
Since $f(x,r)$ is continuous about $x$, there exists $x_0$ with $f(x_0,r_0)=\frac{1}{2}$. Since we have
$$m(A\cap [x_0-r_0,x_0+r_0])=m(A\cap [x_0-r_0, x_0])+m(A\cap [x_0, x_0+r_0])=r_0,$$
there exists $x_1\in [x_0-\frac{r_0}{2},x_0+\frac{r_0}{2}]$ with $f(x_1,\frac{r_0}{2})=\frac{1}{2}$. Or equivalently, there exists $x_1\in\mathbb{R}$ with
$$|x_1-x_0|\le\frac{r_0}{2}, f(x_1,\frac{r_0}{2})=\frac{1}{2}.$$
Similarly, there exists $x_n\in\mathbb{R}$ with
$$|x_n-x_{n-1}|\le\frac{r_0}{2^n},\quad f\left(x_n,\frac{r_0}{2^n}\right)=\frac{1}{2}.$$
Let $x=\lim\limits_{n\to\infty} x_n$, then $|x-x_n|=\left|\sum\limits_{k=n+1}^{\infty} (x_k-x_{k-1})\right|\le \sum\limits_{k=n+1}^{\infty} |x_k-x_{k-1}|\le\frac{r_0}{2^n}$. For any $r<r_0$, there exists unique $N\ge 1$ s.t. $\frac{r_0}{2^N}\le r<\frac{r_0}{2^{N-1}}$. Then we have
$$\left[x_{N+1}-\frac{r_0}{2^{N+1}}, x_{N+1}+\frac{r_0}{2^{N+1}}\right]\subset\left[x-\frac{r_0}{2^N},x+\frac{r_0}{2^N}\right]\subset[x-r,x+r],$$
and
\begin{align*}
f(x,r)&=\frac{m\left( A\cap \left[ x-r,x+r \right] \right)}{2r}\ge \frac{m\left( A\cap \left[ x_{N+1}-\frac{r_0}{2^{N+1}},x_{N+1}+\frac{r_0}{2^{N+1}} \right] \right)}{2\times\frac{r_0}{2^{N-1}}}\\
&=\frac{1}{4}f\left( x_{N+1},\frac{r_0}{2^{N+1}} \right) =\frac{1}{8}.
\end{align*}
 
On the other hand, we have
\begin{align*}
m\left( A\cap \left[ x-r,x+r \right] \right) &\le m\left( A\cap \left[ x_{N+1}-\frac{r_0}{2^{N+1}},x_{N+1}+\frac{r_0}{2^{N+1}} \right] \right)
\\
&+m\left( \left[ x-r,x+r \right] -\left[ x_{N+1}-\frac{r_0}{2^{N+1}},x_{N+1}+\frac{r_0}{2^{N+1}} \right] \right)
\\
&=\frac{r_0}{2^{N+1}}+2r-\frac{2r_0}{2^{N+1}}=2r-\frac{r_0}{2^{N+1}}\le 2r-\frac{1}{4}r=\frac{7}{4}r,
\end{align*}
then
$$f(x,r)=\frac{m(A\cap[x-r,x+r])}{2r}\le\frac{7}{8}.$$
That is to say
$$\frac{1}{8}\le\liminf\limits_{r\to 0_+}f(x,r)\le \limsup\limits_{r\to 0_+}f(x,r)\le\frac{7}{8}.$$
\end{proof}
 
\item Suppose $\{f_n\}_{n=1}^{\infty}$ is a bounded sequence in $L^p$ with $1\le p<\infty$. If $f_n\to f$ a.e., then $f\in L^p$ and
$$\lim\limits_{n\to\infty}\int |f_n|^p-|f_n-f|^p=\int |f|^p.$$
\begin{proof}
We denote $M=\mathop{\sup}_{n\ge 1}\int |f_n|^p<\infty$.Since $f_n\to f$ a.e., we have $|f_n|^p\to|f|^p$ a.e. and by Fatou Lemma
$$\int |f|^p\le\mathop{\underline{\lim}}\limits_{n\to\infty}\int |f_n|^p\le M<\infty,$$
that is to say $f\in L^p$. For $\forall a,b\ge 0$, we have
$$|a^p-b^p|=p\xi^{p-1}|a-b|\le p\max\{a,b\}^{p-1}|a-b|$$
by Lagrange Mean Value Theorem, where $\xi$ is a real number between $a$ and $b$. Then we obtain
$$||f_n|^p-|f_n-f|^p|\le p\max\{|f_n|,|f_n-f|\}^{p-1}|f|.$$
Fixed $\varepsilon>0$. Suppose $A$ is a measurable set with $m(A)<\infty$, and the follow inequality holds
$$\int_{A^c} |f|^p\le \varepsilon.$$
There is a $\delta>0$ such that
$$\int_B |f|^p<\varepsilon \text{ whenever } m(B)<\delta$$
by absolute continuity. Since $f_n\to f$ a.e. on $A$ and $m(A)<\infty$, we can find a measurable subset $a\subset A$ such $m(A\setminus a)<\delta$ and $f_n\to f$ uniformly on $a$ by Egorov Theorem. Then we have
$$\int_a |f_n|^p-|f_n-f|^p\to \int_a |f|^p,$$
as $n\to\infty$, and
$$\int_{a^c} |f|^p=\int_{A\setminus a} |f|^p+\int_{A^c} |f|^p< 2\varepsilon.$$
Since the function $\max\{|f_n|,|f_n-f|\}^{p-1}\in L^{p'}$, and $$||\max\{|f_n|,|f_n-f|\}^{p-1}||_{p'}=||\max\{|f_n|,|f_n-f|\}||_p^{p-1}\le (3M^p)^{\frac{1}{p'}}.$$
We have
\begin{align*}
\int_{a^c} ||f_n|^p-|f_n-f|^p|&\le p\int_{a^c}\max\{|f_n|,|f_n-f|\}^{p-1}|f|\\
&\le p(3M)^{\frac{1}{p'}}(\int_{a^c} |f|^p)^{\frac{1}{p}}
\le p(3M)^{\frac{1}{p'}}(2\varepsilon)^{\frac{1}{p}},
\end{align*}
by H\"{o}lder inequality. Thus we obtain
$$\mathop{\overline{\lim}}\limits_{n\to\infty}\left|\int |f_n|^p-|f_n-f|^p-\int |f|^p\right|\le (1+p(3M)^{\frac{1}{p'}})(2\varepsilon)^{\frac{1}{p}}.$$
\end{proof}
 
\item Suppose $D_n(t)$ are the Dirichlet kernels, and $F_N(t)$ is the $N$-th Fej$\acute{\text{e}}$r kernel given by
$$F_N(t)=\frac{D_0(t)+\cdots+D_{N-1}(t)}{N}.$$
Let $L_N(t)=\min\left(N,\frac{\pi^2}{Nt^2}\right)$. Prove
$$F_N(t)=\frac{1}{N}\frac{1-\cos Nt}{1-\cos t}\le L_N(t)$$
and $\int_{\mathbb{T}} L_N(t)dt\le 4\pi$. If $f\in L^1(\mathbb{T})$ and the $N$-th Ces$\grave{\text{a}}$ro mean of Fourier series is
$$\sigma_N(f)(x)=\frac{S_0(f)(x)+\cdots+S_{N-1}(f)(x)}{N},$$
then $\sigma_N(f)(x)\to f(x)$ for every $x$ in the Lebesgue set of $f$.
\begin{proof}
Since $D_N(t)=\sum\limits_{n=-N}^{N}e^{int}=\frac{\sin(N+\frac{1}{2})t}{\sin\frac{t}{2}}$, we have
$$F_N(t)=\frac{1}{N}\frac{\sin^2\frac{Nt}{2}}{\sin^2\frac{t}{2}}=\frac{1}{N}\frac{1-\cos Nt}{1-\cos t}.$$
Since $|D_N(t)|\le 2N+1$, we can get $F_N(t)\le N$. For $0<x<\frac{\pi}{2}$, we have $\sin x\ge\frac{2}{\pi}x$, then
$$F_N(t)=\frac{1}{N}\frac{\sin^2\frac{Nt}{2}}{\sin^2\frac{t}{2}}\le\frac{1}{N}\frac{1}{\sin^2\frac{t}{2}}\le\frac{\pi^2}{Nt^2}.$$
That mens $F_N(t)\le L_N(t)$. And
$$\int_{\mathbb{T}} L_N(t)dt=2\int_0^{\pi}L_N(t)dt=2\int_0^{\frac{\pi}{N}}Ndt+2\int_{\frac{\pi}{N}}^{\pi}\frac{\pi^2}{Nt^2}dt=4\pi-\frac{2\pi}{N} \le 4\pi.$$
Since $\int_{\mathbb{T}}F_N(t)=1$ and $F_N(t)\le L_N(t)$, we can get $\{F_N(t)\}$ is an approximation to the identity, then $\sigma_N(f)(x)=(f*F_N)(x)\to f(x)$ for every $x$ in the Lebesgue set of $f$.
\end{proof}
 
\item Suppose the sequence $\{a_n\}$ satisfying $a_{n+1}=(4n-2)a_n+a_{n-1}$. Prove that $\{a_n\}$ is convergence if and only if
$$(e-1)a_0+(e+1)a_1=0.$$
\begin{proof}
If $\{a_n\}$ is convengence and not vanishing, then it is obvious that $a_na_{n+1}<0$. We can assume $a_0>0$, $a_1<0$, then $a_{2n}>0$, $a_{2n+1}<0$. Let $b_n=4n-2$ and
$$a_n=p_{n-2}a_1+q_{n-2}a_0, n\ge 2.$$
Since $a_2=b_1a_1+a_0$, and $a_3=(1+b_1b_2)a_1+b_2a_0$, we can get
$$p_0=b_1, p_1=1+b_1b_2, q_0=1, q_1=b_2.$$
Since $a_{n+2}=b_{n+1}a_{n+1}+a_n=b_{n+1}(p_{n-1}a_1+q_{n-1}a_0)+(p_{n-2}a_1+q_{n-2}a_0)=(b_{n+1}p_{n-1}+p_{n-2})a_1+(b_{n+1}q_{n-1}+q_{n-2})a_0$, we can get
$$p_n=b_{n+1}p_{n-1}+p_{n-2}, q_n=b_{n+1}q_{n-1}+q_{n-2}.$$
That is to say
$$\frac{p_n}{q_n}=\left[b_1,b_2,\cdots,b_{n+1}\right]=b_1+\cfrac{1}{b_2+\cfrac{1}{\cdots+\cfrac{1}{b_{n+1}}}},$$
and we have $\frac{p_n}{q_n}\to [b_1,b_2,\cdots]$ as $n\to\infty$. Since
$$a_{2n+1}=p_{2n-1}a_1+q_{2n-1}a_0<0, a_{2n+2}=p_{2n}a_1+q_{2n}a_0>0,$$
we have
$$\frac{p_{2n}}{q_{2n}}(-a_1)<a_0<\frac{p_{2n-1}}{q_{2n-1}}(-a_1),$$
Let $n\to\infty$, we can get
$$a_0+[b_1,b_2,\cdots]a_1=0.$$
On the contrary, if $a_0+[b_1,b_2,\cdots]a_1=0$, since $\left|[b_1,b_2,\cdots]-\frac{p_n}{q_n}\right|<\frac{1}{q_nq_{n+1}}$, we can get
$$|a_n|=|a_1|\cdot\left|p_{n-2}-q_{n-2}[b_1,b_2,\cdots]\right|\le\frac{|a_1|}{q_{n-1}}\to 0.$$
Since
$$\frac{e-1}{e+1}=[0,2,6,10,\cdots]=\cfrac{1}{2+\cfrac{1}{6+\cfrac{1}{10+\cfrac{1}{\cdots}}}},$$
we can get $[b_1,b_2,\cdots]=\frac{e+1}{e-1}$, then $a_0+[b_1,b_2,\cdots]a_1=0$ is equivalent to
$$(e-1)a_0+(e+1)a_1=0.$$
\end{proof}
 
\item Suppose $\{x_n\}$ satisfying $x_1=1$, $x_{n+1}=x_n+\frac{1}{S_n}$, where $S_n=x_1+\cdots+x_n$. Prove that\\
(a) $x_n^2-2\ln S_n$ is increasing and $x_n^2-2\ln S_{n-1}$ is decreasing for $n\ge 2$.\\
(b) $x_n^2-2\ln n-\ln\ln n$ is convengence.\\
(c) $\lim\limits_{n\to\infty} \cfrac{\ln n}{\ln \ln n}\left(\cfrac{x_n}{\sqrt{2\ln n}}-1\right)=\frac{1}{4}$.
\begin{proof}
For $n\ge 2$, we have
\begin{align*}
&\left( x_{n+1}^{2}-\text{2}\ln S_n \right) -\left( x_{n}^{2}-\text{2}\ln S_{n-1} \right) =\left( x_n+\frac{1}{S_n} \right) ^2-x_{n}^{2}+\text{2}\ln \frac{S_{n-1}}{S_n}
\\
&=\frac{2x_n}{S_n}+\frac{1}{S_{n}^{2}}+\text{2}\ln \left( 1-\frac{x_n}{S_n} \right) <\frac{2x_n}{S_n}+\frac{1}{S_{n}^{2}}-2\left( \frac{x_n}{S_n}+\frac{x_{n}^{2}}{2S_{n}^{2}} \right) =\frac{1-x_{n}^{2}}{2S_{n}^{2}}\le 0.
\end{align*}
 
Similarly,
\begin{align*}
&\left( x_{n+1}^{2}-\text{2}\ln S_{n+1} \right) -\left( x_{n}^{2}-\text{2}\ln S_n \right) =\left( x_n+\frac{1}{S_n} \right) ^2-x_{n}^{2}-\text{2}\ln \frac{S_{n+1}}{S_n}
\\
&=\frac{2x_n}{S_n}+\frac{1}{S_{n}^{2}}-\text{2}\ln \left( 1+\frac{x_{n+1}}{S_n} \right) >\frac{2x_n}{S_n}+\frac{1}{S_{n}^{2}}-2\left( \frac{x_{n+1}}{S_n}-\frac{x_{n+1}^{2}}{2S_{n}^{2}}+\frac{x_{n+1}^{3}}{3S_{n}^{3}} \right)
\\
&=\frac{x_{n+1}^{2}-1}{S_{n}^{2}}-\frac{2}{3}\frac{x_{n+1}^{3}}{S_{n}^{3}}=\frac{x_{n+1}^{2}}{S_{n}^{2}}\left( 1-\frac{1}{x_{n+1}^{2}}-\frac{2x_{n+1}}{3S_n} \right).
\end{align*}
Since $x_1=1, x_2=2$, and $S_1=1, S_2=3$, we have
$$x_{n+1}-S_n=x_n+\frac{1}{S_n}-S_n=\frac{1}{S_n}-S_{n-1}\le \frac{1}{S_2}-S_1=-\frac{1}{2}<0,$$
that means $\frac{2x_{n+1}}{3S_n}\le\frac{2}{3}$, then
$$1-\frac{1}{x_{n+1}^2}-\frac{2x_{n+1}}{3S_n}>\frac{1}{3}-\frac{1}{x_{n+1}^2}>0.$$
That implys (a).\\
Since $x_n^2-2\ln S_n$ is increasing, we can get $x_n^2-2\ln S_n\ge x_2^2-2\ln S_2=4-2\ln 3>1$. Since $x_n\ge 1$, we have $S_n\ge n$, then
$$x_n^2\ge 1+2\ln S_n\ge 1+2\ln n.$$
Since $x_n^2-2\ln S_{n-1}$ is decreasing, we can get $x_n^2-2\ln S_n\le x_{n+1}^2-2\ln S_n\le x_2^2-2\ln S_2=4$, then
$$x_n^2\le 4+2\ln S_n.$$
Since $S_n\ge n$, we can get $x_n\le 1+1+\frac{1}{2}+\cdots+\frac{1}{n-1}\le 2+\ln n$, then $S_n\le nx_n\le n(2+\ln n)$, and
$$x_n^2\le 4+2\ln n+2\ln(2+\ln n).$$
Thus, it is obvious that
$$\frac{x_n}{\sqrt{\ln n}}\to\sqrt{2},$$
and we have
$$\frac{S_n}{n\sqrt{\ln n}}\to \sqrt{2}$$
by Stolz formula. Since $x_n^2-2\ln S_n\le 4$, we can get $x_n^2-2\ln S_n$ is convengence, then
$$x_n^2-2\ln n-\ln\ln n=x_n^2-2\ln S_n+2\ln\frac{S_n}{n\sqrt{\ln n}}$$
is convengence. That implys (b).\\
Since $x_n^2=2\ln n+\ln\ln n+a+o(1)$, we can get
$$\frac{x_n^2}{2\ln n}-1=\frac{\ln \ln n}{2\ln n}+\frac{a+o(1)}{2\ln n},$$
then
\begin{align*}
\cfrac{\ln n}{\ln \ln n}\left(\cfrac{x_n}{\sqrt{2\ln n}}-1\right)&=\left(\cfrac{x_n}{\sqrt{2\ln n}}+1\right)^{-1}\cfrac{\ln n}{\ln \ln n}\left(\frac{x_n^2}{2\ln n}-1\right)\\
&\to \frac{1}{2}\cdot\frac{1}{2}=\frac{1}{4}.
\end{align*}
Moreover, we have
$$\lim\limits_{n\to\infty} \ln \ln n\left(\cfrac{\ln n}{\ln \ln n}\left(\cfrac{x_n}{\sqrt{2\ln n}}-1\right)-\frac{1}{4}\right)=\frac{a}{4}.$$
\end{proof}
 
\item Suppose $f\in C[0,\infty)$ and for $\forall a\ge0$, we have
$$\lim\limits_{x\to\infty} f(x+a)-f(x)=0.$$
Then there exist $g\in C[0,\infty)$ and $h\in C^1[0,\infty)$ with $f=g+h$, such that
$$\lim\limits_{x\to\infty} g(x)=0, \text{ } \lim\limits_{x\to\infty} h'(x)=0.$$
\begin{proof}
In fact, $f$ is uniformly continuous. Otherwise, there are two sequences $\{x_n\}_{n=1}^{\infty}$, $\{y_n\}_{n=1}^{\infty}$ and a positive $\varepsilon$, such that
$$x_n, y_n\to\infty, |x_n-y_n|\to 0, |f(x_n)-f(y_n)|\ge\varepsilon_0$$
as $n\to\infty$. Consider the functions
$$\varphi_n(x)=f(x_n+x)-f(x_n)$$
and
$$\phi_n(x)=f(y_n+x)-f(y_n)$$
defined on the interval [0,1]. Then we have
$$\varphi(x), \phi(x)\to 0$$
as $n\to\infty$ due to $\lim\limits_{x\to\infty} f(x+a)-f(x)=0$. For $\forall 0<\varepsilon<\cfrac{1}{2}$, there is a set $A_{\varepsilon}\in[0,1]$ such that $m([0,1]\setminus A_{\varepsilon})<\varepsilon$ and $\varphi_n, \phi_n\to 0$ uniformly on $A_{\varepsilon}$ by Egorov Theorem. Take a integer $N$ such that $\forall n\ge N$ and $\forall x\in A_{\varepsilon}$, we have $|x_n-y_n|<1-2\varepsilon$ and
$$|\varphi_n(x)|\le\cfrac{\varepsilon_0}{3},\qquad |\phi_n(x)|\le\cfrac{\varepsilon_0}{3}.$$
Since $m((x_n+A_{\varepsilon})\cap(y_n+A_{\varepsilon}))=m(x_n+A_{\varepsilon})+m(y_n+A_{\varepsilon})-m((x_n+A_{\varepsilon})\cup(y_n+A_{\varepsilon}))\ge 2(1-A_{\varepsilon})-(1+|x_n-y_n|)=1-2\varepsilon-|x_n-y_n|>0$, there is a point $x\in(x_n+A_{\varepsilon})\cap(y_n+A_{\varepsilon})$. We have $x-x_n,x-y_n\in A_{\varepsilon}$, and then
$$|\varphi(x-x_n)|=|f(x)-f(x_n)|\le\cfrac{\varepsilon_0}{3}, \quad |\phi(x-y_n)|=|f(x)-f(y_n)|\le\cfrac{\varepsilon_0}{3},$$
thus $|f(x_n)-f(y_n)|\le \cfrac{2}{3}\varepsilon_0$, it is a contradiction.
 
Let $h(x)=\int_{x}^{x+1} f(t)dt$, and $g(x)=f(x)-h(x)$, then we have
$$h'(x)=f(x+1)-f(x)\to 0$$
as $x\to\infty$. Since $f$ is uniformly continous, there is positive $M$ such $\forall x,y\ge 0$ with $|x-y|\le 1$, we have $|f(x)-f(y)|\le M$. Since $f(x)-f(x+t)\to 0$ as $x\to\infty$ and $|f(x)-f(x+t)|\le M$ for $\forall t\in [0,1]$, by DCT (Dominated convergence theorem), we have
$$g(x)=\int_{0}^{1} f(x)-f(x+t)\to 0 \text{ as } n\to\infty.$$
\end{proof}
 
 
\end{enumerate}
 
\end{document}

优化专题

印卧涛:主页

Emmanuel Candes:Advanced Topics in Convex Optimization

主页

 

 

使用 Euler 字体

Euler in Modern Toolchains

Today, there exists an OpenType edition of Euler, Neo Euler, by Khaled Hosny. However, it was abandoned in 2011. If you download Neo Euler from its GitHub page, it is possible to use with either LuaLaTeX or XeLaTeX, but there are a few gotchas.

This is lengthy because I give many complex examples that you can cut and paste into your preamble. They should just about cover all the use cases I’ve ever heard anyone recommend.

The Classic Look

Neo Euler, like AMS Euler, contains only glyphs for upright math. To get the classic look of the euler and eulervm packages, or the book Concrete Mathematics, you’ll want to set the unicode-math option math-style=upright.

Then, you want to load only the glyphs Neo Euler provides, with a fallback math font for the rest. Here, I use Khaled Hosny’s newer math font, Libertinus Math. It shows some influence from Euler, especially in its integrals. Finally, make sure to load the Greek letters as the upright math alphabet, since unicode-math expects lowercase Greek letters to be slanted. It sets up the other alphabets the euler package does, including Euler Script as both \mathcal and \mathscr, but not all the alphabets unicode-math supports. Legacy documents should still compile with this preamble.

For the text font, this sets up the Computer Modern Unicode version of Concrete. (Note that the CMU Concrete Bold font is a recent addition. DEK did not create a bold face for Concrete Roman, and used Computer Modern Roman Bold Extended in Concrete Mathematics instead. The beton documentation traditionally recommended Computer Modern Sans Serif Demibold Condensed. Today, you might try Gill Sans/Gillius ADF and see if you like it.)

\documentclass{article}
\pagestyle{empty}

\usepackage{amsmath}
\DeclareMathOperator\Res{Res}
\newcommand*\diff{\mathop{}\!\mathup{d}}

\usepackage{amsthm}
\newtheorem{theorem}{Theorem}

\usepackage{unicode-math}

%%%
% Set up you text and math fonts
%%%

\unimathsetup{math-style=upright}
\setmainfont{CMU Concrete}
\defaultfontfeatures{Scale=MatchLowercase}
\setmathfont{Libertinus Math}
\setmathfont[range={"0000-"0001,"0020-"007E,
                    "00A0,"00A7-"00A8,"00AC,"00AF,"00B1,"00B4-"00B5,"00B7,
                    "00D7,"00F7,
                    "0131,
                    "0237,"02C6-"02C7,"02D8-"02DA,"02DC,
                    "0300-"030C,"030F,"0311,"0323-"0325,"032E-"0332,"0338,
                    "0391-"0393,"0395-"03A1,"03A3-"03A8,"03B1-"03BB,
                    "03BD-"03C1,"03C3-"03C9,"03D1,"03D5-"03D6,"03F5,
                    "2016,"2018-"2019,"2021,"2026-"202C,"2032-"2037,"2044,
                    "2057,"20D6-"20D7,"20DB-"20DD,"20E1,"20EE-"20EF,
                    "210B-"210C,"210E-"2113,"2118,"211B-"211C,"2126-"2128,
                    "212C-"212D,"2130-"2131,"2133,"2135,"2190-"2199,
                    "21A4,"21A6,"21A9-"21AA,"21BC-"21CC,"21D0-"21D5,
                    "2200,"2202-"2209,"220B-"220C,"220F-"2213,"2215-"221E,
                    "2223,"2225,"2227-"222E,"2234-"2235,"2237-"223D,
                    "2240-"224C,"2260-"2269,"226E-"2279,"2282-"228B,"228E,
                    "2291-"2292,"2295-"2299,"22A2-"22A5,"22C0-"22C5,
                    "22DC-"22DD,"22EF,"22F0-"22F1,
                    "2308-"230B,"2320-"2321,"2329-"232A,"239B-"23AE,
                    "23DC-"23DF,
                    "27E8-"27E9,"27F5-"27FE,"2A0C,"2B1A,
                    "1D400-"1D433,"1D49C,"1D49E-"1D49F,"1D4A2,"1D4A5-"1D4A6,
                    "1D4A9-"1D4AC,"1D4AE-"1D4B5,"1D4D0-"1D4E9,"1D504-"1D505,
                    "1D507-"1D50A,"1D50D-"1D514,"1D516-"1D51C,"1D51E-"1D537,
                    "1D56C-"1D59F,"1D6A8-"1D6B8,"1D6BA-"1D6D2,"1D6D4-"1D6DD,
                    "1D6DF,"1D6E1,"1D7CE-"1D7D7
                   }]{Neo Euler}
\setmathfont[range=up/{greek,Greek}, script-features={}, sscript-features={}
            ]{Neo Euler}
\setmathfont[range=up/{latin,Latin,num}, script-features={}, sscript-features={}
            ]{Neo Euler}

\begin{document}

\begin{theorem}[Residue theorem]
  Let $f$ be analytic in the region $G$ except for the isolated
  singularities $a_1,a_2,\dots,a_m$. If $\gamma$ is a closed
  rectifiable curve in $G$ which does not pass through any of the
  points $a_k$ and if $\gamma\approx 0$ in $G$, then
  \[
    \frac{1}{2\symup{\pi i}} \int\limits_\gamma f\Bigl(x^{\mathbf{N}\in\mathbb{C}^{N\times 10}}\Bigr)
    = \sum_{k=1}^m n(\gamma;a_k)\Res(f;a_k)\,.
  \]
\end{theorem}

\begin{theorem}[Maximum modulus]
  Let $G$ be a bounded open set in $\BbbC$ and suppose that $f$ is a
  continuous function on $G^-$ which is analytic in $G$. Then
  \[
    \max\{\, |f(z)|:z\in G^- \,\} = \max\{\, |f(z)|:z\in \partial G \,\}\,.
  \]
\end{theorem}

First some large operators both in text:
$\iiint\limits_{Q}f(x,y,z) \diff x \diff y \diff z$
and
$\prod_{\gamma\in\Gamma_{\bar{C}}}\partial(\tilde{X}_\gamma)$;\
and also on display
\[
  \iiiint\limits_{Q}f(w,x,y,z) \diff w \diff x \diff y \diff z
  \leq
  \oint_{\partial Q} f'\Biggl(\max\Biggl\{
  \frac{\Vert w\Vert}{\vert w^2+x^2\vert};
  \frac{\Vert z\Vert}{\vert y^2+z^2\vert};
  \frac{\Vert w\oplus z\Vert}{\vert x\oplus y\vert}
  \Biggr\}\Biggr)\,.
\]
\end{document}

Neo Euler with CMU Concrete, classic style

This is a variant of the classic “Survey of Free Math Fonts for TeX and LaTeX.”

Extending to ISO Style with Concrete

If you want to get the full range of math alphabets, you would need to kitbash the Euler math font with others to supply the missing glyphs. Here is a version that selects ISO style (upright sum and product symbols, constants π and i, and numerals; Italic Γ function and variables), retains the upright ∂ from Neo Euler, keeps all the math alphabets that exist in Neo Euler (except the digits, which ought to match the text font and have bad spacing as superscripts anyway), and supplies the missing pieces of itbfbfup and bfit from the CMU Concrete family.

\unimathsetup{math-style=ISO, partial=upright, nabla=upright}
\setmainfont{CMU Concrete}
\defaultfontfeatures{Scale=MatchLowercase}
\setmathfont{Libertinus Math}
\setmathfont[range={"0000-"0001,"0020-"007E,
                    "00A0,"00A7-"00A8,"00AC,"00AF,"00B1,"00B4-"00B5,"00B7,
                    "00D7,"00F7,
                    "0131,
                    "0237,"02C6-"02C7,"02D8-"02DA,"02DC,
                    "0300-"030C,"030F,"0311,"0323-"0325,"032E-"0332,"0338,
                    "0391-"0393,"0395-"03A1,"03A3-"03A8,"03B1-"03BB,
                    "03BD-"03C1,"03C3-"03C9,"03D1,"03D5-"03D6,"03F5,
                    "2016,"2018-"2019,"2021,"2026-"202C,"2032-"2037,"2044,
                    "2057,"20D6-"20D7,"20DB-"20DD,"20E1,"20EE-"20EF,
                    "210B-"210C,"210E-"2113,"2118,"211B-"211C,"2126-"2128,
                    "212C-"212D,"2130-"2131,"2133,"2135,"2190-"2199,
                    "21A4,"21A6,"21A9-"21AA,"21BC-"21CC,"21D0-"21D5,
                    "2200,"2202-"2209,"220B-"220C,"220F-"2213,"2215-"221E,
                    "2223,"2225,"2227-"222E,"2234-"2235,"2237-"223D,
                    "2240-"224C,"2260-"2269,"226E-"2279,"2282-"228B,"228E,
                    "2291-"2292,"2295-"2299,"22A2-"22A5,"22C0-"22C5,
                    "22DC-"22DD,"22EF,"22F0-"22F1,
                    "2308-"230B,"2320-"2321,"2329-"232A,"239B-"23AE,
                    "23DC-"23DF,
                    "27E8-"27E9,"27F5-"27FE,"2A0C,"2B1A,
                    "1D400-"1D433,"1D49C,"1D49E-"1D49F,"1D4A2,"1D4A5-"1D4A6,
                    "1D4A9-"1D4AC,"1D4AE-"1D4B5,"1D4D0-"1D4E9,"1D504-"1D505,
                    "1D507-"1D50A,"1D50D-"1D514,"1D516-"1D51C,"1D51E-"1D537,
                    "1D56C-"1D59F,"1D6A8-"1D6B8,"1D6BA-"1D6D2,"1D6D4-"1D6DD,
                    "1D6DF,"1D6E1,"1D7CE-"1D7D7
                   }]{Neo Euler}
\setmathfont[range=up/{greek,Greek}, script-features={}, sscript-features={}
            ]{Neo Euler}
\setmathfont[range=up/{latin,Latin,num}, script-features={}, sscript-features={}
            ]{Neo Euler}
\setmathfont[range=up/num]{CMU Concrete}
\setmathfont[range=it/{latin,Latin,greek,Greek}]{CMU Concrete Italic}
\setmathfont[range=bfup/{Greek}]{CMU Concrete Bold}
\setmathfont[range=bfit/{latin,Latin,greek,Greek}]{CMU Concrete Bold Italic}

Neo Euler with CMU Concrete, ISO style

Euler plus Palatino (My Personal Favorite)

Here is the version that, after a lot of trial and error, I like the best.

Another common recommendation, including in the earlier answer, is to combine Euler math symbols with Palatino text. It sets the text font to Palatino, or one of its many clones, in this case Pagella (although you might have the original Palatino or Palatino Linotype). It then fills in all the missing parts of Neo Euler with another Palatino clone, Asana Math. It sets up \mathcalas Euler Script, \mathbfcal from the alternate style of Asana, \mathbb from Latin Modern Math (more legible and more similar to classic amsfonts), digits from Asana (as these need to match the main text) and all other symbols not defined in Euler from Asana.

We now have complete coverage of all Unicode math symbols, which allows us to set this example in ISO style. This uses italic math letters as the default, including uppercase Greek such as the Gamma function, but leaves symbols such as ∑ and ∏ intact and sets constants such as 2πi in the denominator as upright. (Observe that unicode-math is smart enough to set \symup{\pi i} as Euler and \mathrm and operator names in the text font.) It keeps the upright partial derivative and nabla, as Euler does not define cursive forms.

You can change back to math-style=upright to get a more classic look that still allows you to use all the math alphabets.

\unimathsetup{math-style=ISO, partial=upright, nabla=upright}
\setmainfont{TeX Gyre Pagella}
\defaultfontfeatures{Scale=MatchLowercase}
\setmathfont{Asana Math}
\setmathfont[range={"0000-"0001,"0020-"007E,
                    "00A0,"00A7-"00A8,"00AC,"00AF,"00B1,"00B4-"00B5,"00B7,
                    "00D7,"00F7,
                    "0131,
                    "0237,"02C6-"02C7,"02D8-"02DA,"02DC,
                    "0300-"030C,"030F,"0311,"0323-"0325,"032E-"0332,"0338,
                    "0391-"0393,"0395-"03A1,"03A3-"03A8,"03B1-"03BB,
                    "03BD-"03C1,"03C3-"03C9,"03D1,"03D5-"03D6,"03F5,
                    "2016,"2018-"2019,"2021,"2026-"202C,"2032-"2037,"2044,
                    "2057,"20D6-"20D7,"20DB-"20DD,"20E1,"20EE-"20EF,
                    "210B-"210C,"210E-"2113,"2118,"211B-"211C,"2126-"2128,
                    "212C-"212D,"2130-"2131,"2133,"2135,"2190-"2199,
                    "21A4,"21A6,"21A9-"21AA,"21BC-"21CC,"21D0-"21D5,
                    "2200,"2202-"2209,"220B-"220C,"220F-"2213,"2215-"221E,
                    "2223,"2225,"2227-"222E,"2234-"2235,"2237-"223D,
                    "2240-"224C,"2260-"2269,"226E-"2279,"2282-"228B,"228E,
                    "2291-"2292,"2295-"2299,"22A2-"22A5,"22C0-"22C5,
                    "22DC-"22DD,"22EF,"22F0-"22F1,
                    "2308-"230B,"2320-"2321,"2329-"232A,"239B-"23AE,
                    "23DC-"23DF,
                    "27E8-"27E9,"27F5-"27FE,"2A0C,"2B1A,
                    "1D400-"1D433,"1D49C,"1D49E-"1D49F,"1D4A2,"1D4A5-"1D4A6,
                    "1D4A9-"1D4AC,"1D4AE-"1D4B5,"1D4D0-"1D4E9,"1D504-"1D505,
                    "1D507-"1D50A,"1D50D-"1D514,"1D516-"1D51C,"1D51E-"1D537,
                    "1D56C-"1D59F,"1D6A8-"1D6B8,"1D6BA-"1D6D2,"1D6D4-"1D6DD,
                    "1D6DF,"1D6E1,"1D7CE-"1D7D7
                   }]{Neo Euler}
\setmathfont[range=up/{greek,Greek}, script-features={}, sscript-features={}
            ]{Neo Euler}
\setmathfont[range=up/{latin,Latin}, script-features={}, sscript-features={}
            ]{Neo Euler}
\setmathfont[range={bfup/{latin, Latin, greek, Greek}, frak, bffrak, cal},
             script-features={}, sscript-features={}
            ]{Neo Euler}
\setmathfont[range={up/num, bfup/num, it, bfit, scr, bfscr,
                    sfup, sfit, bfsfup, bfsfit, tt}
            ]{Asana Math}
\setmathfont[range=bfcal, Scale=MatchUppercase, Alternate]{Asana Math}
\setmathfont[range=bb, Scale=MatchUppercase]{Latin Modern Math}

Neo Euler plus Asana plus Latin Modern with Pagella

Euler Symbols, Palatino Letters

This alternative uses use only the math symbols from Neo Euler and overwrite all its math alphabets, for the most consistency between text and math modes.

\unimathsetup{math-style=ISO, partial=upright, nabla=upright}
\setmainfont{TeX Gyre Pagella}
\defaultfontfeatures{Scale=MatchLowercase}
\setmathfont{Asana Math}
\setmathfont[range={"0000-"0001,"0020-"007E,
                    "00A0,"00A7-"00A8,"00AC,"00AF,"00B1,"00B4-"00B5,"00B7,
                    "00D7,"00F7,
                    "0131,
                    "0237,"02C6-"02C7,"02D8-"02DA,"02DC,
                    "0300-"030C,"030F,"0311,"0323-"0325,"032E-"0332,"0338,
                    "0391-"0393,"0395-"03A1,"03A3-"03A8,"03B1-"03BB,
                    "03BD-"03C1,"03C3-"03C9,"03D1,"03D5-"03D6,"03F5,
                    "2016,"2018-"2019,"2021,"2026-"202C,"2032-"2037,"2044,
                    "2057,"20D6-"20D7,"20DB-"20DD,"20E1,"20EE-"20EF,
                    "210B-"210C,"210E-"2113,"2118,"211B-"211C,"2126-"2128,
                    "212C-"212D,"2130-"2131,"2133,"2135,"2190-"2199,
                    "21A4,"21A6,"21A9-"21AA,"21BC-"21CC,"21D0-"21D5,
                    "2200,"2202-"2209,"220B-"220C,"220F-"2213,"2215-"221E,
                    "2223,"2225,"2227-"222E,"2234-"2235,"2237-"223D,
                    "2240-"224C,"2260-"2269,"226E-"2279,"2282-"228B,"228E,
                    "2291-"2292,"2295-"2299,"22A2-"22A5,"22C0-"22C5,
                    "22DC-"22DD,"22EF,"22F0-"22F1,
                    "2308-"230B,"2320-"2321,"2329-"232A,"239B-"23AE,
                    "23DC-"23DF,
                    "27E8-"27E9,"27F5-"27FE,"2A0C,"2B1A,
                    "1D400-"1D433,"1D49C,"1D49E-"1D49F,"1D4A2,"1D4A5-"1D4A6,
                    "1D4A9-"1D4AC,"1D4AE-"1D4B5,"1D4D0-"1D4E9,"1D504-"1D505,
                    "1D507-"1D50A,"1D50D-"1D514,"1D516-"1D51C,"1D51E-"1D537,
                    "1D56C-"1D59F,"1D6A8-"1D6B8,"1D6BA-"1D6D2,"1D6D4-"1D6DD,
                    "1D6DF,"1D6E1,"1D7CE-"1D7D7
            }]{Neo Euler}
\setmathfont[range={up/{latin, Latin, greek, Greek, num},
                    it, bfup, bfit, bb, bbit, scr, bfscr, frak, bffrak,
                    sfup, sfit, bfsfup, bfsfit, tt }
            ]{Asana Math}
\setmathfont[Alternate, range={cal, bfcal}]{Asana Math}

Neo Euler/Asana Math Kitbash with Pagella

Backward-Compatiblity with PDFTeX

If you cannot use unicode-math, it is still possible to get Type 1 fonts, bold math symbols, upright Greek, and more. Here is a sample:

\documentclass{article}
\pagestyle{empty}

\usepackage{amsmath}
\DeclareMathOperator\Res{Res}
\newcommand*\diff{\mathop{}\!\mathrm{d}}

\usepackage{amsthm}
\newtheorem{theorem}{Theorem}

%%%
% Set up you text and math fonts
%%%

\usepackage[T1]{fontenc}
\usepackage{textcomp}
\usepackage[utf8]{inputenc}
\usepackage{amssymb}
\usepackage{ccfonts}
\usepackage{eucal}
\usepackage{eufrak}
\usepackage{eulervm}

\newcommand\BbbC{\ensuremath{\mathbb{C}}}
\DeclareSymbolFont{eulerup}{U}{zeur}{m}{n}
\DeclareMathSymbol{\uppi}{\mathord}{eulerup}{"19}
\DeclareMathSymbol{\upi}{\mathord}{eulerup}{"69}

\begin{document}

\begin{theorem}[Residue theorem]
  Let $f$ be analytic in the region $G$ except for the isolated
  singularities $a_1,a_2,\dots,a_m$. If $\gamma$ is a closed
  rectifiable curve in $G$ which does not pass through any of the
  points $a_k$ and if $\gamma\approx 0$ in $G$, then
  \[
    \frac{1}{2 \uppi \upi} \int\limits_\gamma f\Bigl(x^{\mathbold{N}\in\mathbb{C}^{N\times 10}}\Bigr)
    = \sum_{k=1}^m n(\gamma;a_k)\Res(f;a_k)\,.
  \]
\end{theorem}

\begin{theorem}[Maximum modulus]
  Let $G$ be a bounded open set in $\BbbC$ and suppose that $f$ is a
  continuous function on $G^-$ which is analytic in $G$. Then
  \[
    \max\{\, |f(z)|:z\in G^- \,\} = \max\{\, |f(z)|:z\in \partial G \,\}\,.
  \]
\end{theorem}

First some large operators both in text:
$\iiint\limits_{Q}f(x,y,z) \diff x \diff y \diff z$
and
$\prod_{\gamma\in\Gamma_{\bar{C}}}\partial(\tilde{X}_\gamma)$;\
and also on display
\[
  \iiiint\limits_{Q}f(w,x,y,z) \diff w \diff x \diff y \diff z
  \leq
  \oint_{\partial Q} f'\Biggl(\max\Biggl\{
  \frac{\Vert w\Vert}{\vert w^2+x^2\vert};
  \frac{\Vert z\Vert}{\vert y^2+z^2\vert};
  \frac{\Vert w\oplus z\Vert}{\vert x\oplus y\vert}
  \Biggr\}\Biggr)\,.
\]

\end{document}

Note that \mathrm selects the text font and \mathbf doesn’t work, so I instead define \uppi and \upi (\mathnormal would work until you changed the math font to something other than Euler, whereas these select upright Greek letters from Euler in any document) and use the \mathbold command from eulervm.

eulervm with ccfonts

2018中科院考博事宜

关于查询2018年秋季入学博士招生考试初试成绩的通知
招生办公室 2018年04月05日
各位考生:
 
中国科学院大学招生信息网定于4月10日开通2018年秋季博士入学考试初试成绩查询功能。查询路径:中国科学院大学招生信息网--在线查询--博士--博士成绩查询。考生输入证件号码、准考证号和验证码后查询成绩。
 
若丢失准考证或遗忘准考证号,请在开通成绩查询功能期间使用自助找回功能,找回路径:中国科学院大学招生信息网--在线查询--博士--博士成绩查询--说明--第6条找回准考证号。
 
在线查询已提供考生的初试成绩及成绩打印功能,各单位将不再向考生寄发纸质成绩单。初试成绩以中国科学院大学招生信息网下载打印的初试成绩单为准,无须加盖单位公章。
 
考生若对初试成绩有异议,请于4月16日中午12点前与你报考的研究所或学院招生部门联系,提出成绩核查申请,过时不再受理。
 
专业课成绩由各研究所或学院招生部门受理并负责核查;国科大统一命题科目(含外国语和政治理论)成绩由国科大考试中心受理各研究所或学院招生部门收集汇总的核查申请,并协同有关部门核查。
 
考试中心将于4月17日安排专人统一办理研究所或学院的复查申请,并将核查结果反馈相关研究所或学院。考试中心不受理考生的个人核查申请。请考生直接与研究所或学院招生部门咨询成绩复查结果。
 
 
 
                                                         中国科学院大学招生办公室  
 
                                                              2018年4月5日
 
中国科学院大学关于2018年秋季入学博士研究生招生考试分数线的公告
招生办公室 2018年04月11日
各位考生:
 
 
 
中国科学院大学2018年秋季入学博士研究生招生考试分数线基本要求如下:
 
 
 
(一)普通招考考生(含全日制和非全日制)进入拟录取名单的基本条件为其思想政治品德考核与体检必须合格,复试成绩不低于60分,入学考试初试各门业务课成绩不低于60分,且符合以下条件:
 
1.报考北京、上海地区各单位的考生,入学考试英语初试成绩不低于55分,其中报考农学(学科代码09)各专业的考生,入学考试英语成绩不低于53分;
 
报考陕西、四川、云南、贵州、甘肃、青海、新疆、重庆等西部地区各单位的考生,入学考试英语初试成绩不低于51分,其中报考农学(学科代码09)各专业的考生,入学考试英语成绩不低于50分;
 
报考其他地区各单位的考生,入学考试英语初试成绩不低于53分,其中报考农学(学科代码09)各专业的考生,入学考试英语成绩不低于51分。
 
2.外语选考小语种的,小语种考试初试成绩不低于55分。
 
3.同等学力考生的政治理论和加试的2门业务课成绩均不低于60分。
 
 
 
(二) “少数民族高层次骨干人才计划”考生进入拟录取名单的基本条件为其思想政治品德考核与体检必须合格,同等学力政治理论和两门业务课加试成绩均不低于60分,复试成绩不低于60分,且符合以下条件:
 
1.报考博士新疆班和民族药专项的少数民族骨干计划考生,其初试成绩分数线由相关培养单位根据报考该专项的生源状况和招生计划数自行划定,报国科大招生办公室备案。
 
2.报考各单位除上述两项专项计划外的少数民族考生,入学考试外语初试成绩不低于50分,各门业务课初试成绩不低于60分,初试总分不低于225分;报考各单位除上述两项专项计划外的汉族骨干计划考生,入学考试外语成绩不低于55分,各门业务课初试成绩不低于60分,初试总分不低于230分。
 
 
 
(三)拟录取的硕博连读转博考生,其转博考核成绩不低于60分。
 
 
 
(四)开展博士招考改革“申请-考核”制的单位,其分数线要求执行与对应地区全校普通招考考生相同的分数线。
 
 
 
(五)“顶尖千人”创新团队博士生专项计划、或为其他单位导师代招的,一般应执行挂靠单位对应专业的分数线。
 
 
 
(六)直博生是指从优秀本科推免生中遴选并录取为直接攻读博士学位的考生。拟录取的直博生,必须是在接收推免阶段已面试合格,且已按直博生办理接收手续的高校推免生。在录取阶段,须按推免生要求审核其本科最后一学年的学习成绩和现实表现。若有下列情况之一者,取消其拟录取资格:
 
1.在本科阶段最后一学年(四年制的指第七、八学期,五年制的指第九、十学期)主干课程学习成绩有不及格科目。
 
2.毕业设计(论文)未取得良好以上(含良好)成绩。
 
3.毕业时未获得本科毕业证或学士学位。
 
4.政审或体检不合格。
 
5.考试作弊或违纪(法)受到“警告”以上处分的,或有其他情节严重的违法乱纪行为受到处罚者。
 
6.申请人提交的材料有弄虚作假者。
 
7.考生主动申请自愿放弃者。
 
 
 
(七)各单位可根据上述第(一)~(五)款的要求,结合本单位的生源情况、招生计划和学科状况,制定具体的分专业分数线(含单科成绩要求和总分要求)。各单位制定的分数线可高于上述第(一)~(五)款的要求,但不得低于上述第(一)~(五)款规定的分数线。
 
 
 
 
 
 
 
中国科学院大学招生办公室
 
2018年4月11日
 
2018年博士录取名单公示
2018-05-03
序号 姓名 性别 学科专业名称
1 余渊洋 基础数学
2 郭琪 基础数学
3 董晓婧 基础数学
4 姜姗 计算数学
5 李翊君 计算数学
6 杨鹏飞 计算数学
7 王晓荣 计算数学
8 郭益敏 统计学
9 李娜 统计学
10 李春亚 统计学
11 苏苗苗 统计学
12 刘也驰 应用数学
13 陈跃文 应用数学
14 刘明明 应用数学
15 王丽 应用数学
16 李嘉旭 应用数学
17 司苏亮 应用数学
18 成程程 应用数学
19 王齐 运筹学与控制论
20 董康宁 运筹学与控制论
21 徐汾 运筹学与控制论
22 刘泽群 系统理论
23 井立达 系统分析与集成
24 杨光 管理科学与工程
25 冷洁 管理科学与工程
26 祝灵秀 管理科学与工程
27 程旭 基础数学
28 亓博 基础数学
29 游志兴 基础数学
30 虞天龙 基础数学
31 肖博 基础数学
32 张义平 基础数学
33 王宏钰 基础数学
34 肖冠举 基础数学
35 颜晓军 基础数学
36 朱秀武 基础数学
37 邵锋 基础数学
38 徐旺 基础数学
39 林章立 基础数学
40 刘楚箫 基础数学
41 马元庆 基础数学
42 陈雅丹 计算数学
43 党海坤 计算数学
44 付姚姚 计算数学
45 桂升 计算数学
46 金典聪 计算数学
47 金鹏展 计算数学
48 刘瑶宁 计算数学
49 史言言 计算数学
50 徐岚 计算数学
51 刘世伟 计算数学
52 徐孜立 计算数学
53 杨沐明 计算数学
54 曾慧兰 计算数学
55 张晓迪 计算数学
56 周雪梅 计算数学
57 吴宇宸 计算数学
58 谢小月 概率论与数理统计
59 崔凯源 概率论与数理统计
60 苏厚齐 概率论与数理统计
61 冯伟 概率论与数理统计
62 刘松 应用数学
63 苏佩 应用数学
64 詹伟成 应用数学
65 陈凯伦 应用数学
66 李想 应用数学
67 王永建 应用数学
68 郭婧 应用数学
69 骆丽夏 应用数学
70 赵明阳 应用数学
71 刘珍 应用数学
72 王贺松 应用数学
73 马鸿宇 应用数学
74 彭鑫 运筹学与控制论
75 邬潇莹 运筹学与控制论
76 苑秋月 运筹学与控制论
77 陈良彪 运筹学与控制论
78 钟声 运筹学与控制论
79 彭程 运筹学与控制论
80 张小城 运筹学与控制论
81 任晓涛 运筹学与控制论
82 王嘉成 运筹学与控制论
83 熊健康 系统理论
84 朱艳 系统理论
85 王成杰 统计学
86 靳瑾 统计学
87 郑思明 统计学
88 邹家辉 统计学
89 孟珍 统计学
90 李萌 计算机软件与理论
91 刘帅 管理科学与工程
92 宣一 基础数学
93 薛威 基础数学
94 王杰 基础数学
95 桂弢 基础数学
96 李源 概率论与数理统计
97 龙鸣亚 概率论与数理统计
98 夏梓耕 概率论与数理统计
99 许锐航 应用数学
100 秦国林 应用数学
101 王石玮 运筹学与控制论
102 冷嘉承 运筹学与控制论
103 王子轩 概率论与数理统计
104 池文豪 管理科学与工程
105 崔如鸿 管理科学与工程
106 张瑜 管理科学与工程
107 相鑫 管理科学与工程
108 陈辉 系统理论
109 涂植鹏 系统理论
110 黄世杰 系统理论
111 赵玉涛 系统理论
112 刘俊良 应用数学
113 杨剑霆 应用数学
114 徐戍 计算数学
115 黄磊 计算数学
116 王辉 计算数学
117 姜博鸥 计算数学
  
  公示期间(10个工作日),对公示名单如有异议,可通过电话、电子邮件或书面形式,向招生办反映情况,要求实事求是且署真实姓名。 
 
  联系电话:010-82541871 
 
  邮 箱:yyh@amss.ac.cn 
 
  中国科学院数学与系统科学研究院 
 
  研究生部 
 
  2018年5月3日 

漫谈微分几何(丘成桐专辑)

漫谈微分几何(丘成桐专辑) 
Prince 发表于 2006/9/23 21:01:00 
漫谈微分几何 
 
丘成桐 
 
今天很高兴能够在各位面前讲讲我做学问的经验,可以供大家参考一下。我讲「如何学好微分几何」的题目,主要是想跟大家讲讲有关于从前我做学问的态度,因为我是做几何的,所以我就讲做微分几何。很明显的,大部份的同学不会选几何,不过没有关系,其实就是讲讲我做学问的态度。
 
首先,讲讲我从前的一些经验。我从前在香港长大,在香港念中学、大学,然后到美国念研究所,所以至少在前一半跟大家的经验应该差不了太远,不过是时代有点不同。我在多年前念数学,你们现在念数学,看法上已经有许多不相同,事实上我也不太了解你们现在的想法。不过基本上,我们都是中国文化出生的,所以我想仍有一部份共同的地方。基本上我们是要讲怎么作科学研究,也就是纯科学的研究,我们要看的是我们的志向是怎样的。假如我们想做一个好的科学家,当然我讲的是怎么做一个好的数学家。先说我自己的经验,我从前在香港培正中学念中学的时候,就开始对数学有兴趣。当然还有一些其它的课程,我对数学有兴趣,一方面是受到我家庭的影响,我父亲是做哲学的,所以对于念数学一直都相当鼓励,到了中学以后,我父亲去世了。不过也因此对于自然科学有很浓厚的兴趣。另一方面受老师的影响也很大。我想很重要的当我们开始要做一个学问,尤其是你真的要做一个出色的科学家,跟你的兴趣和你一开始所立下的志向有很大的关系。就是说,开始的时候你期望能够做到什么。假如说开始的时候你根本不想做一个好的科学家,那么你就永远也不可能做一个好的科学家。从前有位大学老师跟我讲说:「假如你不买马票,你永远也中不了。」倒不是说我鼓励你们去买马票,是说假如你不准备做好的科学家,就永远也做不了一个好的科学家。不过是不是讲,你想做一个好的科学家,你就可以做个好的科学家呢?当然不是,你还要有很多其它的因素在里面,我想第一点是要你将做人的目标先决定。 
 
我在国外二十多年了,也教了不少的学生,有些在世界上算是很出名,但有些不是太行。从这方面来讲,比较好的学生和不好的学生我可以晓得不同的经验。我想好的学生大部份一开始就决定他要做到什么程度的科学家,从很早就可以看得出来,因为有了志向以后,才晓得怎么去用功、怎么去花时间在上面。这看起来倒是老生常谈,因为你从小学、中学到大学,大概很多老师都跟你讲同样的意见,可能你听多了都觉得没有什么意思,但是事实上这是成功的第一个因素。我的一位老师跟我讲,你要决定以后你想做什么,讲明了,不是为名就是为利。当时我很惊讶,老师为什么讲这一句话。我们不能否定大部份的想法不是为名就是为利,同时这个想法也推动了不少科学的研究。不过我们也晓得,单是为名为利不可能将科学达到最高峰的研究,我们一定要对这个科学有浓厚的兴趣。我们应当晓得,做科学,我们有一个很纯正的想法,就是对真理的追寻,在真理的背后有一个很漂亮的境界在里面,我们到了一个境界以后,对我们追求学问的人来讲,是无法抗拒的,就算是没有名没有利,我们也希望能够将这个真理搞清楚。举例来讲,如果你喜欢下棋的话,有时你会晓得下到一半的时候,结局会是怎样,你非为名也非为利,当然可以讲说你是为了好胜,但是有时候你总是想追求,想晓得怎么解决这个问题。在科学上来讲我们要追求的是比这个高的境界。我为什么讲为名为利这个事实呢?举例来讲,我们这几年在哈佛大学里教了几个在大学里念数学念得很好的学生,可是到了毕业的时候,我晓得他们明明对数学有很大的兴趣,但是他们选取了完全不同的途径,他们有些人宁愿选取做生意或是到银行里面做事。我并不反对你们去做生意、赚大钱,我失望的缘故是因为这些学生明明是对做学问兴趣特别大,但是他们没有办法去抗拒赚钱的引诱而放弃了继续做学问的前途,有些人甚至过了几年赚了钱,又想重新再做学问,但问题是无论你资质有多好,一般来讲你将做学问的机会放弃以后,再想重新做起将会遇到许多困难。并不是说不可能,也曾有这种情形发生过,但是真正能够达到的情形,几乎是绝无仅有,做学问是不能中断的。我遇见过很多朋友,有些甚至是很有名的数学家,他们有些人会讲我现在一方面做行政的工作,一方面可以做学问,可是事实上,这是没有办法可以达到两者兼顾的情形。我们晓得做学问几乎是全心全意的工作,当对证明追寻的时候,很难说受到其它外界的打扰,仍能够达到很高的成功的。以我的经验来讲,在想问题的时候,晚上睡觉也在想这个问题,躺在床上也在想,早上起床第一件事就是想这个问题。我并不是讲你们也要这样子,我是希望你们在遇到一个问题要解决的时候,你要全力以赴,不可能在中间慢慢想一点而在其它也可以花点功夫,这样精神不集中的态度是不可能做好学问的。我想对大家做个建议,假如你想做个真正的好科学家的话,就不能够再往回走,假如你想做生意,那干脆一开始就不要想这个问题,并不是你要做个好的教员就要照我刚才讲的,要花这么多功夫,倒是要念好科学这是很重要的,所以这是第一点,立志很重要。 
 
第二点我要讲的,我在国外多年,遇见过许多很出名的数学家,甚至许多有名的物理学家我也见过许多。在我认为并没有一个是真正的像一般报纸上所讲的是天才,在我所亲身认识的大科学家,都是经过很大的努力,才能够达到他所达到的成就。我的学生问我:「为什么你做的比我好?」,我说很简单,我比你用功。我在办公室或是在家里边,我天天在想问题,你们在外面玩,而我花了功夫在解决想了很久的问题,我总比你不想、不花时间成就大一点。你可能去听个大科学家或大数学家演讲,你会觉得漂亮得不得了,怎么一个人能够讲得这么好!这个人是个天才!可是你有没有想到,他在后面准备花了多少时间想这个问题?大概你们听过最出名的科学家费因曼,《费因曼物理》注1漂亮得不得了,所有出名的物理学家都这么讲,去听的人不是学生,都是老师或物理学家。费因曼在准备费因曼物理的时候是什么事都不做,就只有脑子在花功夫,整天在想这个问题,跟许多学生不停的在谈这个问题。费因曼是个有名的天才,可是他准备这个研究也花了许多不同的功夫。我想很多出名的科学家在有所表现出不同的时候,你会觉得他是天才,事实上他用在后面的功夫都是很不少的。 
 
有许多很聪明很厉害的人可能是研究生甚至是教授,往往你给他一个问题,他可以很快给你一个答案,同时是很不错的一个答案。可是很多这样出色的学生或是教授,过了很久以后,你总会觉得他没有做出很好的成绩出来。问题是,你解决的问题太容易了;没有再花很多精神去考虑这个问题。尤其在我们中国人最缺乏的,就是在做中学生或是大学生的时候,没有将一个问题从头到尾仔细考虑清楚,并没有真正的全部了解,这是个很重要的问题。从一个很小的问题,我们可以引发很多不同而且有意思的问题。思考要自己训练,不单是在联考或在大学的时候,老师出个题目,你考了一百分就完了,假如这样的话,你很容易就满足你自己,你不觉得问题有什么意思。往往出名的研究是在很平凡的问题里面,不停的思考所找出来的,很多人因为很快将问题解决了,便不愿再想下去,所以不能够再启发新的东西。科学的研究,不是解决人家已经晓得的问题。当一个科学家问一个好的问题的时候,即是成功的一半。因为科学的推动是从不断的找寻新的问题,新的方向出来的,解决从前的问题虽是个重要的推动方向,可是我们还要找出新的方向,而不单是解决从前的问题。我们知道在物理上解决问题的时候,往往大的或出名的公式是将前面固定的理论推翻,而找出新的路子。为什么大数学家或大物理学家能够做到这个地步呢?因为他们不断的问问题。有时候在一般人来讲很明显的问题,在出名的科学家看起来,就不见得很明显。为什么不明显呢?因为我们有不同层次的问题要一路考虑下去。问问题的能力是一个很重要的训练,并不是花很多功夫就可做到,我想在我们中国的小学、中学或大学里都没有很好的做到这一点,我想从小应该做到这一点的。 
 
在我们来看数学跟其它物理、化学或生物等实验科学有那些不同?物理或化学等科学是从一般实验、现象界所找的题目,最后再经过实验的证实,才能算是个成功的理论。理论物理学家可以发展很多不同漂亮的理论,但最后假如不能够在实验里做出来的话,对物理学家来讲就是一篇废话。数学家有个好处。就是说,我们做了学问,一方面大部份是从一般的科学里面产生给我们的,一方面可以当作文学作品来欣赏。我们的取材多采多姿,一方面是比较基本的,从自然界或物理上的基本粒子、广义相对论、重力场去拿出很多基本的大自然的问题。这方面对近代几何学上的影响很大,另一方面可从比较没那么基本的理论里发生出来。所谓不基本,并不是说不重要。我们要了解到我们有些问题是从工业界来的,譬如说做飞机、做螺丝,甚至做流体变动的问题,都是可产生许多有趣的几何问题或是数学问题。例如说机械人手怎么去拿东西?这都可以看做是基本的几何问题,物理学家不一定有兴趣,可是数学家却有很大的兴趣。另外我们也可以对与实际问题不相近的问题产生兴趣,我们对一个图画得漂不漂亮,我们也可以在数学上研究。几何在数学上的取材有三个不同方向:第一是从基本自然界里产生的问题。从基本粒子、重力场到电磁波基本上如何产生的种种重要几何问题,从表面上你看不出来为什么它跟几何有关,但事实上近代物理将很多这种基本场论的问题变成几何问题,对微分几何来讲有很大的贡献。第二是刚才所讲,工业界与古典力学出了很多很重要的几何问题。第三就是纯粹从美的观点来找问题。举例来讲,从数论里面找了许多很漂亮的问题,尤其是近十或二十年来,大部份重要的数论问题大多是用几何的方法来解决的,这是几何在数学上三个重要的取材方向。 
 
我为什么讲取材的问题呢?因为很多中学生或大学生在念几何或是某些数学课程的时候,认为我们念那个学科就念那个学科就够了,而不要念其它的学问,这是个很错误的观念。因为数学里面每一门的学问都有密切关联的,不单是数学,其实所有的理论科学中间都有很密切的关系。例如我们刚刚所讲的,高能物理与数学的关系,或是化学甚至生物都跟数学有很大的关系,所以我想怎么学几何呢?第一点是当你决定好要做一个好的几何学家时,你一定要广泛的学不同的学问,基础要比较广,如微分方程、代数、物理学以及其它学科,至少在心理上有个准备,就是说这些学科将来是对你有帮助的。你听起来会觉得这是很困难的事情,你不可能学会这么多种不同的学问。这主要的分别就是你要有一个层次,你的专科是那一方面,就要多学一点,但不可忘掉其它的学科。有时在某个意义下,我们可以很惊讶的看到同一个学问、同一个命题,在两个不同的学科里面,可以以不同的方法出现,就是说以不同的方法证明。我想主要的原因是根本上这两个学科的分别并不是很大。在几十年前有个出名的物理学家说数学有不可思议的力量。为什么数学能够在物理上有这么大的影响呢?因为从物理学家的看法,数学家祇是在玩一些简单的符号,纯粹是在家里想一些自己的问题,与自然界的关系好象不大,其实这是个错误的想法。我们数学家研究的问题是很具体的,只是有不同的层次,所以有点不同而已。举例来说我们研究微分几何上一个最简单的图形-圆球,这圆球可以说是一个抽象的观念,我们也可以说它是自然界很具体的一部份。也就是说我们将所研究的圆球视为自然界的一部份,其实跟物理的现象差不了太远的。尤其在现代的高能物理里,我们研究基本粒子,尤其到了量子力学的观念以后,因为能量已经到了很高的地步,所以有很多根本没有办法做实验,所以基本上也是在家里或课堂里或办公室里用纸笔来算,这跟数学家想象的差不了太远。假如物理学家可以这么做,表示数学家也能够坐在家里面而对自然界达到某种程度的了解。 
 
为什么我要讲这些呢?这些与微分几何有什么关系呢?我要讲的是你在选题的时候,我们虽然有个自由度对于选题与自然界无关,但是我们也有一个限度在里面,假如我们选的问题与现实相差太远,最后我们的命题会被淘汰掉。在历史上出现很多不同的研究,过了十年、二十年后就完全被淘汰的。你看现在的图书馆里面有许多的文章出现,不过再过个十年八年以后,我想大部份的文章是会被淘汰掉的,根本在整个数学历史上起不了任何作用。这是因为很多的文章实在没有解决问题,其次是对我们研究的对象没有产生任何效果。所以虽然我们数学界不用时间来做证明,可是我们有某种程度的测试。一般来讲,证的很好的数学,二十年或五十年内都可以看到它在现实里出现帮助。我们晓得在这个二十年以来,从前许多不重要的问题,在今日的工程上发生很大的影响。举例来讲,从前在数论里对于质数的搜查这个问题,这完全是一个无聊的命题。就是说一个很大的数,你怎么将它因子分解得很快。近十多年来,在国防科学上这问题变成一个重要的命题,有许多国防科学家在做这方面的研究,所以说数学上的选题很重要。为什么因子分解很重要呢?表面上看来跟真正的用途好象没有什么关联,可是它是一个很自然的问题,一个很大的整数它怎么分解,很快地,表面上并不重要,但可以帮助我们了解质数的分布情形,所以我说选题是一个很重要的问题。我记得从前我们在做大学生的时候,花了很多功夫去念一些文章与参考书,有些对数学来讲是很无意义的,可是反过来说因为花了很多功夫,所以可以了解到有些问题比较重要,有些问题比较不重要,所以花的功夫并没有白费。 
 
其次我们讲做一个学生应该是怎么一个看法。对于做数学或做微分几何来讲,我觉得研究的气氛很要紧,尤其在中国的环境里,好象是不太容易培养出这种气氛来。假如你旁边的朋友或同学跟你谈的都是其它的问题,譬如说股票涨了或跌了或其它问题,久而久之,你大概对于做学问也没有很大的兴趣,所以培养做学问的态度与你交的朋友、跟的老师的关系很大。如果你们时常讨论学术上的问题,你就不会觉得自己很孤单,能够激励你对数学上有更大的兴趣。假如你自暴自弃,就是说你认为自己不能够在数学上做研究,不能够在数学上达到贡献的话,你永远也达不到,而且同时也影响到你旁边的朋友,使得大家都不能向前走。我们晓得许多出名的数学家甚至在牢里也可以写一些出名的文章,倒不是你永远关在牢里就能做好的文章,是说人在最困难的时候也可以做研究。除了气氛很重要外,你也需要得到先进的支持,从前我们念中学的时候,念了很多关于做学问的方法,从前觉得很好笑,以后念书念得多了以后就觉得这些很重要,事实上这些是很重要的经验。有句话说「学而不思则罔,思而不学则怠」,你单是学而不想是不行的,你单是想而不学也是不行的,这两句话看起来很简单,其实就是怎么分配你的学习跟思想,这是一个很微妙很重要的问题。一个人无论你多用功多天才,你假如不将前人做过的东西去体验去学习,是不可能做好的。这道理很简单,一个人的智能有限,我们不可能与前面十年、五年所有人做过的加起来的智能相比,我们要靠前人的经验,要靠他们的启发,才能够向前迈进,虽然有人自夸的讲比他们加起来都行,我不相信这种情形,也没见过这种情形。所以出名的贡献如爱因斯坦、牛顿的贡献,也是在前人的成果方面再向前走一大步或一小步。所以学是一定要的,可是如果你学过这个东西以后而不去思考,不去消化,就算你可以考第一,考一百分,但是你不想是绝对没有用的。我们看过很多出名的天才,十二岁就拿到学士学位,甚至拿了很高分,可是往往我们看不出他以后的成就。为什么很多所谓的天才在以后的科学发展里没有任何的贡献?这是因为他们没有思考,没有思考在科学上完全不会引起任何的波澜、任何的贡献,对于整个科学完全没有好处。所以学了以后一定要思考,怎么分配你的学习跟思考就往往要有导师的帮忙或是同学的帮忙。所谓的帮忙并不是说老师跟你讲你应当这么做或应当怎么做,这样往往是没有很大的效果,所以我刚刚讲的气氛很重要。从人家用功的程度或是讲话的态度的启发,或是讲话的时候能够去听,追根出什么东西来,从它而得到很大的帮助。从前我到柏克莱去念研究所时,我花了很多功夫去听很多不同的科目,有些人觉得很奇怪,为什么我会去听那些课?我觉得这些课对我有好处,过了几十年后我还是觉得有好处。有些课在我去听的当时可能不懂,可是听了还是觉得有好处,因为一个人的脑袋的想法并不是那么简单的,有时候某些东西当时可能不懂,可是慢慢的就能领悟很多东西。我举例来讲,我做博士论文的时候,我刚好要用到群论的东西,当时我问过许多专家,但是都不懂,我突然想到从前在某一课上听过一个有关这方面的论文,我忘了当时讲什么课,但我记得大概在那里可以找这方面的文章,所以我花了2天的时间在图书馆,结果给我找到差不多是我所要的文章。假如当初不去听这门课的话,我完全没有这个机会,所以有时候听一门不懂的课,有很多不同的帮助,所以很多研究生我跟他们讲,你们去听课不一定要懂,你坐在那边总比不坐在那边好,你不坐在那边的话,你完全不可能知道有其它的方法。
 
我想最后还是你对整个学问有多大兴趣的问题,假如你对这个学问兴趣不大的话,你没办法长年累月的坐在图书馆,坐在办公厅里,或是坐在一个课堂上听课,所以你一定要先决定你对这学问的兴趣有多大,当然做研究还有许多其它方面比较复杂的原因,以后有机会我们再讲下去。我想现在你们在大学的阶段,最要紧的是决定以后你要做什么东西,其它的可能就容易做到了。 
 
 
新语丝是个好网站,文章不错!且看下文: 
 
从丘成桐的杭州之行说起 
 
作者:拉布拉多天师 
 
  教育这个问题似乎现在来谈已经变成了老话题。中国的教育制度一变再变,已经变成了一个畸形怪胎。好像医学院大玻璃瓶里面用福尔马林泡着的四肢不备心脑不全的怪物。任何人看见了都难堪得不吐不快,但是几乎所有的言论都无法指导其向一个健全的方向上发展。最近哈佛的华裔数学家丘成桐大师又对中国的教育体系发了一通牢骚。大约颇能吸引诸位关心中国教育的看官们的眼球罢。 
 
  据说7月8号丘成桐跑到杭州来了一趟,估计是希望来消夏,孰料又和什么杭州市高考的数学尖子之类见了面。结果让他大失所望。许多人根本对数学体系没有清晰的概念,于定理亦不甚了了。令我们的丘老教授大失所望。认为中国的教育体系培养的学生不过是做题的机器,根本不能培养出数学人才云云。又把哈佛大学高高捧了一遍,说最近几年面试的中国学生,根本没有符合要求的。想必又让许多想靠数学晋身哈佛女孩刘亦婷一流人物的未来精英大大义愤填膺了一把。丘教授虽然在国内被许多大学联聘,但是对于国内的现状才是真的“不甚了了”。中国教育自古不过是社会的一个缩影。社会上时兴什么,就学什么。连颜之推老先生看到鲜卑语吃香,还巴望着子孙学学外语之类。一代大儒语言学家都难免势利一回,何况今日大学之狗头校长乎。国贸金融计算机这类专业,几乎敢挂大学牌子的学校都有了。这两年工业设计时兴,七七八八的学校都纷纷开设。 其所为人才乎?钱乎?当然钱是一定要到手的,人才么,看造化。 
 
  大学如此,学生的算盘就打得更加精了。学什么当然无需兴趣,只要毕业有钱赚就不成问题。僧多粥少自然就要靠考分定天下,考分高低和做题准确性挂钩,所以只要做对就可以,至于懂或不懂,并非主要问题也。如此心态的学习,想做好怕是不易。能具备基本的专业知识都难,何况叫本科生“每年发表2—3篇文章”? 
 
  本科教育如此,毕业之后还是如此。丘教授觉得国内做学问缺少“做学问的热情和科研的大视野”,却不觉得这是国内体制的必然。综观国内任何一所大学,无论是毕业加薪升职分房,无不与论文产出的数量和速率挂钩。在这种情况下,选题要想不偏于简单方便速成,简直就是天方夜谭。本来学生学习的目的就是赚钱,难道还能不求多年的教育投资有所回报,反而去做事倍功半的研究?就算自己愿意,单位还不愿意。学校的考核制度就要看每人每年的论文数。不够?对不起,请你下岗,斯文扫地。丘教授贵为浙大特聘,不知道是否了解过敬爱的浙大校长,中国共产党中央委员会候补委员潘云鹤院士否?潘院士在当了浙大校长之后的7年里,平均每年发表 32篇文章。平均十天一篇,实在是让我等小辈望尘莫及。然而文章的质量,倒是不见人细细考究。而潘院士校务缠身,又经常出国联谊联谊。大约文章只要一夜就有一篇的。年轻教师自然是惭愧无地,赶快要打起精神多快好省建设社会主义。一线课题短期不见成效,岂不是要被潘院士辈笑话?还是算了。 
 
  我不知道丘先生为什么只是奚落在这种环境下产生的懵懂学生,而没有更深一步质疑一下同样是这个环境之下产生的一批所谓“专家学者”。或许数学界还算比较干净,然而如今的生化学界已经算是名誉扫地。中国千不该万不该不该出了个方舟子,方又偏偏学了生物。这下可是苦了一帮生物学界的大尾巴狼。成天就怕被方舟子抓到把柄。于是国家一横心封了新语丝,可是最近似乎连国内也无法忍受这些功利性的专家了。徐匡迪院士公开站出来质疑这些所谓专家,他们究竟扮演什么角色?所谓的 “社会活动院士”是人们不能忍受的。作为学者,所代表的应当是真理而非某个集团的私利。为了个人利益而歪曲事实甚至编造事实的“专家”是可耻的。然而就是在一个这样的教育体系之下,没有数学头脑,只会做题的学生,最终就会被培养成为这类无耻专家,因为这和他们功利的目标是一脉相承的。能够为了个人的功利心甘愿成为作题机器的学生,最终也将为因为利欲熏心而放弃真理和良知。我们经历了所谓的应试教育改革而跨入素质教育,而一切告诉我们这不过只是无聊的文字把戏而已。如果所能做的仅仅只是诅咒这该死的社会风气,那么我们不如一起冷笑着看它毁灭吧。 
 
(XYS20050813) 
 
◇◇新语丝(www.xys.org)(xys.dxiong.com)(xys.3322.org)(xys.dyndns.info)◇◇ 
 
 
数学和中国文学的比较(丘成桐在西交演讲)(附报告的ppt) 
 
数学和中国文学的比较 
 
丘成桐 8.8.2005 
 
很多人会觉得我今日的讲题有些奇怪,中国文学与数学好像是风马牛不相及,但我却讨论它。其实这关乎个人的感受和爱好,不见得其他数学家有同样的感觉,“如人饮水,冷暖自知”。每个人的成长和风格跟他的文化背景、家庭教育有莫大的关系。我幼受庭训,影响我至深的是中国文学,而我最大的兴趣是数学,所以将两者做一个比较,对我来说是相当有意义的事。 
   
数学与文学之源于自然 
 
数学之为学,有其独特之处,它本身是寻求自然界真相的一门科学,但数学家也如文学家般天马行空,凭爱好而创作,故此数学可说是人文科学和自然科学的桥梁。 
 
数学家研究大自然所提供的一切素材,寻找它们共同的规律。这里所说的大自然比一般人所了解的来得广泛,我们认为数字、几何图形和各种有意义的规律都是自然界的一部分,我们希望用简洁的数学语言将这些自然现象的本质表现出来。 
 
大略言之,数学家以其对大自然感受的深刻肤浅,来决定研究的方向,这种感受既有其客观性,也有其主观性,后者则取决于个人的气质。气质与文化修养有关,无论是选择悬而未决的难题,或者创造新的方向,文化修养皆起着关键性的作用。因为人文知识也致力于描述心灵对大自然的感受,所以司马迁写史记除了“通古今之变”外,也要“究天人之际”。 
 
刘勰在《文心雕龙·原道篇》说,文章之道在于“写天地之辉光,晓生民之耳目。” 
 
刘勰以为文章之可贵,在尚自然,在贵文采。他又说:“人与天地相参,乃性灵所集聚,是以谓之三才,为五行之秀气,实天地之灵气。灵心既生,于是语言以立。语言既立,于是文章着明,此亦原于自然之道也。” 
 
历代的大数学家如阿基米德如牛顿莫不以自然为宗,见物象而思数学之所出,即有微积分的创作。费尔玛和尤拉对变分法的开创性发明,也是由于探索自然界的现象而引起的。 
 
广义相对论提出了场方程,它的几何结构成为几何学家梦寐以求的对象,因为它能赋予空间一个调和而完美的结构。我研究这种几何结构垂30年,时而迷惘,时而兴奋,自觉同《诗经》、《楚辞》的作者,或晋朝的陶渊明一样,与大自然浑为一体,自得其趣。 
 
捕捉大自然的真和美,实远胜于一切人为的造作,正如《文心雕龙》说的:“云霞雕色,有踰画工之妙。草木菁华,无待锦匠之奇,夫岂外饰,盖自然耳。” 
 
在空间上是否存在满足引力场方程的几何结构是一个极为重要的物理问题,它也逐渐地变成几何中伟大的问题。尽管其他几何学家都不相信它存在,我却锲而不舍,不分昼夜地去研究它,就如屈原所说:“亦余心之所善兮,虽九死其犹未悔。” 
 
我花了5年工夫,终于找到了具有超对称的引力场结构,并将它创造成数学上的重要工具。当时的心境,可以用以下两句来描述:“落花人独立,微雨燕双飞。” 
 
数学的文采 
 
数学的文采,表现于简洁,寥寥数语,便能道出不同现象的法则,甚至在自然界中发挥 
作用。我的老师陈省身先生创作的陈氏类,就文采斐然,令人赞叹。它在扭曲的空间中找到 
简洁的不变量,在现象界中成为物理学界求量子化的主要工具,可说是描述大自然美丽的诗 
篇,直如陶渊明“采菊东蓠下,悠然见南山”的意境。 
 
从欧氏几何的公理化,到笛卡儿创立的解析几何,到牛顿、来布尼兹的微积分,到高斯 
、黎曼创立的内蕴几何,一直到与物理学水乳相融的近代几何,都以简洁而富于变化为宗, 
其文采绝不逊色于任何一件文学创作。 
 
文学家为了达到最佳意境的描述,需要追究“僧推月下门”与“僧敲月下门”的区别。 
数学家为了创造美好的理论,也不必依随大自然的规律,只要逻辑推导没有问题,就可以尽 
情地发挥想像力。 
   
文学与数学的赋比兴 
 
然而文章终究有高下之分,大致来说,好的文章“比兴”的手法总会比较丰富。 
 
中国古诗十九首,作者年代不详,但大家都认为是汉代的作品。刘勰说:“比采而推, 
两汉之作乎。”这是从诗的结构和风格进行推敲而得出的结论。在数学的研究过程中,我们 
亦利用比的方法去寻找真理。我们创造新的方向时,不必凭实验,而是凭数学的文化涵养去 
猜测去求证。 
 
举例而言,30年前我提出一个猜测,断言三维球面里的光滑极小曲面,其第一特征值等 
于二。当时这些曲面例子不多,只是凭直觉,利用相关情况模拟而得出的猜测。最近有数学 
家写了一篇文章证明这个猜想。其实我的看法与文学上的比兴很相似。 
 
我们看《洛神赋》:“翩若惊鸿,婉若游龙。荣曜秋菊,华茂春松。髣髴兮若轻云之蔽 
月,飘飘兮若流风之回雪。”由比喻来刻划女神的体态。 
 
我一方面想像三维球的极小曲面应当是如何的匀称,一方面想像第一谱函数能够同空间 
的线性函数比较该有多妙,通过原点的平面将曲面最多切成两块,于是猜想这两个函数应当 
相等,同时第一特征值等于二。 
 
当时我与卡拉比教授讨论这个问题,他也相信这个猜测是对的。旁边我的一位研究生问 
为什么会做这样的猜测,不待我回答,卡教授便微笑说这就是洞察力了。 
 
伟大的数学家远瞩高瞻,看出整个学问的大流,有很多合作者和跟随者将支架建立起来 
,解决很多重要的问题。正如曹雪芹创造《红楼梦》时,也是一样,全书既有真实,亦有虚 
构;既有前人小说如《西厢记》、《金瓶梅》、《牡丹亭》等的踪迹,亦有作者家族凋零、 
爱情悲剧的经验,通过各种不同人物的话语和生命历程,道出了封建社会大家族的腐败和破 
落。《红楼梦》的写作影响了清代小说垂200年。 
   
数学家看事物的多面性 
 
由于文学家对事物有不同的感受,同一事或同一物可以产生不同的吟咏。例如对杨柳的 
描述,温庭筠写“柳丝长,春雨细……”吴文英写“一丝柳,一寸柔情,料峭春寒中酒…… 
”李白写“年年柳色,灞陵伤别。”周邦彦写“柳阴直,烟里丝丝弄碧,隋堤上,曾见几番 
,拂水飘绵送行色……长亭路,年去岁来,应折柔条过千尺。”晏几道写“舞低杨柳楼心月 
,歌尽桃花扇底风。” 
 
对事物有不同的感受后,往往通过比兴的方法另有所指,例如“美人”有多重意思,除 
了指美丽的女子外,也可以指君主:屈原《九章》:“结微情以陈词兮,矫以遗夫美人。” 
也可以指品德美好的人,《诗经·邶风》:“云谁之思,西方美人。”苏轼《赤壁赋》:“ 
望美人兮天一方。” 
 
数学家对某些重要的定理,也会提出很多不同的证明。例如勾股定理的不同证明有10个 
以上,等周不等式亦有五六个证明,高斯则给出数论对偶定律六个不同的看法。不同的证明 
让我们可以从不同的角度去理解同一个事实,往往引导出数学上不同的发展。 
 
对空间中的曲面,微分几何学家会问它的曲率如何,有些分析学家希望沿着曲率方向来 
推动它一下看看有甚变化,代数几何学家可以考虑它可否用多项式来表示,数论学家会问上 
面有没有整数格点。这种种主观的感受由我们的修养来主导。 
 
反过来说,文学家对同一事物亦有不同的歌咏,但在创作的工具上,却有比较统一的对 
仗韵律的讲究,可以应用到各种不同的文体。从数学的观点来说,对仗韵律是一种对称,而 
对称的观念在数学发展至为紧要,是所有数学分枝的共同工具。另外,数学家又喜欢用代数 
的方法来表达空间的结构,同调群乃是重要的例子,由拓朴学出发而应用到群论、代数、数 
论和微分方程学上去。 
 
 
数学的意境 
 
王国维在《人间词话》里说:“词以境界为最上。有境界则自成高格……有造境,有写境,此理想与写实两派之所由分。然两者颇难分别,因大诗人所造之境必合乎自然,所写之境亦必邻于理想故也。有有我之境,有无我之境。‘泪眼问花花不语,乱红飞过秋千去。’……有我之境也。‘采菊东蓠下,悠然见南山。’……无我之境也。有我之境,以我观物,故物皆着我之色彩。无我之境,以物观物,故不知何者为我,何者为物……无我之境,人唯乎静中得之。有我之境,于由动入静时得之,故一优美,一宏壮也。自然之物互相关系,互相限制。然其写之于文学及美术中也,必有其关系限制之处。故虽写实家亦理想家也。又虽如何虚构之境,其材料必求之于自然,而其构造亦必从自然之法律。故虽理想家亦写实家也。” 
 
数学研究当然也有境界的概念,在某种程度上也可谈有我之境、无我之境,当年尤拉开创变分法和推导流体方程,由自然现象引导,可谓无我之境,他又凭自己的想像力研究发散级数,而得到zeta函数的种种重要结果,开300年数论之先河,可谓有我之境矣。另外一个例子是法国数学家Grothen-dick,他著述极丰,以个人的哲学观点和美感出发,竟然不用实例,建立了近代代数几何的基础,真可谓有我之境矣。 
 
在几何的研究中,我们发现狄拉克在物理上发现的旋子在几何结构中有魔术性的能力,我们不知道它的内在的几何意义,它却替我们找到几何结构中的精髓,在应用旋子理论时,我们常用的手段是通过所谓消灭定理而完成的,这是一个很微妙的事情,我们制造了曲率而让曲率自动发酵去证明一些几何量的不存在,可谓无我之境矣。以前我提出用Einstein结构来证明代数几何的问题和用调和映像来看研究几何结构的刚性问题也可作如是观。 
 
不少伟大的数学家,以文学、音乐来培养自己的气质,与古人神交,直追数学的本源,来达到高超的意境。《文心雕龙·神思》:“文之思也,其神远矣。故寂然凝虑,思接千载;悄然动容,视通万里。吟咏之间,吐纳珠玉之声,眉睫之前,卷舒风云之色,其思理之致乎。” 
   
数学的品评 
 
好的工作应当是文已尽而意有余,大部分数学文章质木无文,流俗所好,不过两三年耳。但是有创意的文章,未必为时所好,往往十数年后始见其功。 
 
我曾经用一个崭新的方法去研究调和函数,以后和几个朋友一同改进了这个方法,成为热方程的一个重要工具。开始时没有得到别人的赞赏,直到最近5年大家才领会到它的潜力。然而我们还是锲而不舍地去研究,觉得意犹未尽。 
 
我的老师陈省身先生在他的文集中引杜甫诗“文章千古事,得失寸心知。”而杜甫就曾批评初唐四杰的作品“王杨卢骆当时体,不废江河万古流。” 
 
数学华丽的作品可从泛函分析这种比较广泛的学问中找到,虽然有其美丽和重要性,但与自然之道总是隔了一层。举例来说,从函数空间抽象出来的一个重要概念叫做巴拿赫空间,在微分方程学中有很重要的功用,但是以后很多数学家为了研究这种空间而不断地推广,例如有界算子是否存在不变空间的问题,确是漂亮,但在数学大流上却未有激起任何波澜。 
   
数学的演化 
 
数学的演化和文学有极为类似的变迁。从平面几何至立体几何,至微分几何等等,一方面是工具得到改进,另一方面是对自然界有进一步的了解,将原来所认识的数学结构的美发挥尽至后,需要进入新的境界。上面谈到的高维拓朴文气已尽,假使它能与微分几何、数学物理和算术几何组合变化,亦可振翼高翔。 
 
当一个大问题悬而未决的时候,我们往往以为数学之难莫过于此。待问题解决后,前途豁然开朗,看到比原来更为灿烂的火花,就会有不同的感受。 
 
其实,科学家对自然界的了解,都是循序渐进,在不同的时空自然会有不同的感受。有学生略识之无后,不知创作之难,就连陈省身先生的大作都看不上眼,自以为见识更为丰富,不自见之患也。人贵自知,始能进步。 
 
庄子:“今尔出于崖涘,观于大海,乃知尔丑,尔将可与语大理矣。”我曾经参观德国的葛庭根大学,看到19世纪和20世纪伟大科学家的手稿,他们传世的作品只是他们工作的一 部分,很多杰作都还未发表,使我深为惭愧而钦佩他们的胸襟。今人则不然,大量模仿,甚至将名作稍为改动,据为己有,尽快发表。或申请院士,或自炫为学术宗匠,于古人何如哉 。 
 
 
数学家的感情 
 
为了达到深远的效果,数学家需要找寻问题的精华所在,需要不断地培养我们对问题的感情和技巧,这一点与孟子所说的养气相似。气有清浊,如何寻找数学的魂魄,视乎我们的文化修养。 
 
白居易说:“圣人感人心而天下和平,感人心者,莫先乎情,莫始乎言,莫切乎声,莫深乎义……未有声入而不应,情交而不感者。” 
 
严羽《沧浪诗话》:“盛唐诸公唯在兴趣,羚羊挂角,无迹可求。故其妙处透澈玲珑,不可凑拍,如空中之音,相中之色,水中之影,镜中之象,言有尽而意无穷。” 
 
我的朋友Hamilton先生,他一见到问题可以用曲率来推动,他就眉飞色舞。另外一个澳洲来的学生,见到与爱因斯坦方程有关的几何现象就赶快找寻它的物理意义,兴奋异常,因此他们的文章都是清纯可喜。反过来说,有些成名的学者,文章甚多,但陈陈相因,了无新意。这是对自然界、对数学问题没有感情的现象,反而对名位权利特别重视。为了院士或政协委员的名衔而甘愿千里仆仆风尘地奔波,在这种情形下,难以想像他们对数学、对自然界有深厚的感情。 
 
数学的感情是需要培养的,慎于交友才能够培养气质。博学多闻,感慨始深,堂庑始大。欧阳修:“人间自是有情痴,此恨不关风与月。”“直须看尽洛城花,始与东风容易别。”能够有这样的感情,才能够达到晏殊所说:“昨夜西风凋碧树,独上高楼,望尽天涯路。” 
 
浓厚的感情使我们对研究的对象产生直觉,这种直觉看对象而定,例如在几何上叫做几 何直觉。好的数学家会将这种直觉写出来,有时可以用来证明定理,有时可以用来猜测新的 命题或提出新的学说。 
 
报告的ppt 
http://www.aoxue.org/cgi-bin/bbs/attachment.cgi?aid=697677 
 
转贴一个,看是否可以帮助了解一些内涵?
发信人: stoneboy (Denise|石头哥), 信区: Joke 
标 题: 试图通俗地讲一下庞加莱猜想是怎么回事(zz) 
发信站: 水木社区 (Sat Jun 10 13:39:23 2006), 站内
 
发信人: zhrb (碧涛), 信区: square 
标 题: 试图通俗地讲一下庞加莱猜想是怎么回事(zz) 
发信站: 一见如故 (Sat Jun 10 11:54:22 2006), 本站(yjrg.net)
 
发信人: apc (拿什么拯救你我的大兵瑞恩), 信区: News 
标 题: 试图通俗地讲一下庞加莱 猜想是怎么回事zz 
发信站: 人间仙境 (Sat Jun 10 09:40:54 2006), 转信
 
The Magic Flute, Act 1, Scene 4, 1791 魔笛,第一幕,第四场 点击放大
据说庞加莱猜想被中国人证明了,那个证明的长度有三百多页,这样一来就成了中国人的骄傲。本贴子因此就打算通俗地介绍一下庞加莱猜想是怎么回事。 
因为,要说起来这个猜想的术语那是很抽象的,是说“单连通的闭三维流型同胚于三维球面”,但是这让数学的外行害怕,一害怕就不敢研究。但这样就有问题,万一其它专业的人要利用这个原理呢?所以我尝试用通俗的办法来讲一下什么是庞加莱猜想。
 
首先,我以前一直就是有一个观点,那就是数学家真没有意思,数学家要证明的东西,往往在常人看来,都是废话。什么是废话呢? 比如人不吃饭要饿死,汽车没有火车跑得快这样的肯定对头的话,或者在常人看来理当如此的话。但是数学家们偏要证明一下,而且证明起来还挺难。
 
比方说吧,两点之间直线最近,这件事情不要说每一个人知道,甚至连一条狗都知道。但是你要真正证明它,光大学的高等数学知识还是不够的,还要进修泛函分析,变分法,这才能够证明这件事情,瞧这多麻烦?
 
好,现在来讲这个庞加莱猜想是什么回事,后面大家会看到,那其实也是一个废话。当然,现在已经证明了,就是庞加莱定理了。因为是在三维空间,因此就好说了。
 
我们居住的房子,如果里面没有摆放任何家具,当然就是一个长方体的形状的空间,有长,宽,高。当然,我们不讨论这样的通常的房子。
 
我们想象这样一个房子,这个空间是一个球。或者,想象一下,一只巨大的足球,里面充满了气,我们钻到里面看,这就是一个球形的房子。
 
嗨,我不妨假设这个球形的房子周边其实是钢做的表面,非常结实,没有窗户没有门,我们现在在这样的球型房子里呆着。
 
现在拿一个汽球来,带到这个球形的房子里。随便什么汽球都可以(我一开始故意这么说,其实对这个汽球是有要求的)。这个汽球并不是瘪的,而是已经吹大成某一个形状了,什么形状都可以(后面要说明这也是胡说, 其实对形状也是有要求的)。 但是这个汽球,我们还可以继续吹大它,而且假设汽球的皮特别结实,肯定不会被吹炸了。还要假设,这个汽球的皮是无限薄的。当然,又无限薄又能够结实,这本身就是脱离实际了,但是没有办法啊,科学总是要抽象的嘛,不让抽象我们就得不出什么成果。
 
好,现在我们继续吹大这个汽球,一直吹啊吹。吹到最后会怎么样呢?那个庞加莱先生就猜想了,吹到最后,一定是这个汽球的表面和整个球形房子的墙壁表面紧紧地贴住,中间没有缝隙了。
 
当然,还要有一些假设,就是我们这个人不能呆在这个球形房子里,否则的话汽球会有一部分贴到人身上,而不是贴到墙壁上了。可是没有人怎么吹汽球呢?哎呀抽象嘛。我们可以假设有一个小精灵躲在汽球里面吹,用一个压缩的空气瓶吹。或者,也可以不是吹这个汽球,而是在这个大球形的,非常结实的钢制的房子外面抽气,把房里的气抽光,则汽球里的空气就能够膨胀,也能够达到效果,反正最后一定是能够汽球的表面和房子墙壁紧紧贴着,一点缝隙都没有。
 
但是这个猜想到现在还不严格。如果这个汽球只是一个长形的,或者球形的,那是可以做到的。但是,如果这个汽球是一个救生圈的形状,那就不行了,因为救生圈在不断吹大的时候,最后有一些表面并不是紧贴在墙面上,而是会相互挤在一起。
 
因此,这个猜想就必须把类似救生圈一类的汽球排除开。认为拿这样的汽球来吹属于赖皮行为。
 
最后定的规则是这样,就是,如果我们钻到那个汽球里去(假设我们是小人国里的小精灵,会飞),我们用一只苍蝇,用一根线绑在苍蝇身上,(假设这根线无限细且没有重量。然后让苍蝇随意地到处飞。这样,我手中的线就象风筝线一样不断地放出去,最后那个苍蝇还要飞回来,飞回来以后,我把栓在苍蝇身上的线头解下来,和我手中的线系在一起,这就构成了一个圈,或者叫一个绳套吧,能够把人勒死的那种。然后把这个绳套往自己怀里拉,拉呀拉,最后总能够把这个绳套统统都给拉回来。比如说,救生圈形状就不行,因为如果苍蝇在救生圈里飞了一圈回来,我这个结成的绳套就肯定收不会来,而给挡在那里了。那么,这样的汽球就不符合要求。
 
因此,我要求的汽球,它的形状虽然可以随意,但是,里面的任何一根封闭的曲线,或者说绳套吧,都不会绕过一根类似柱子这样的东西,或者说,这个汽球看上去没有“孔”,不象救生圈那样,可以把一个头伸进去。这样的汽球,数学家起了一个名字叫“单连通”,之所以要起这么吓人的名子,无非是为的显示自己挺有学问罢了,吓唬人的,无非是一个整个的不带孔的汽球嘛。
 
也就是说,庞加莱定理,说的就是,一个单连通的汽球(市面上卖的汽球大多数都是单连通的),在一个球形的房子里使劲地吹,最后一定能够使汽球的表面和球形房子的墙壁紧紧贴着,一点缝隙都没有。当然,得假设这个球形的房子里的空气,随着汽球的吹大,是会被排光的。
 
瞧,就这么个事,象不象废话啊?为证明这件事情花了三百多页,是不是有一些吃饱了撑得慌?不光如此,这说法还如此地学究,什么“单连通的闭三维流型同胚于三维球面”,吓唬人不是?硬要将汽球说成是流型,显摆自己学问深不是?唉,总算球面大家还是知道的。什么叫“同胚”?也够吓唬人的,就是把汽球吹大后两个表面紧紧贴着。
 
所以啊,诸位小朋友们也可以想一些这样的废话,也就可以给出中国人给出的猜想了。现在光是外国人有猜想,中国人却没有。要我早知道庞加莱瞎猜的东西有这么简单,我就提前猜想了,让别人累得半死去证明去。那我多有名啊。
 
其实这样的猜想我也已经想到了一个。上面不是讲如果一个汽球是球生圈的形状,就不能够在一个球形的房间里吹大且和球形的墙壁紧密接触吗?那么好了,我这儿也设计一个巨大的房子,不是球形的,是一个球生圈形状的,而且,那个救生圈形状的汽球也套在这个巨大的房子里,这样我再吹这个汽球,它就肯定和这个房子的墙壁紧密接触了吧?
 
好,现在本人提出二十一人民八阕论坛数学网友提出的最伟大的数学猜想如下: 
将一个内胎置入一个外胎里,然后对这个内胎使劲打气,最后的结果一定是内胎的外表面和 
外胎的内表面亲密接触。 
 
数学的内容、方法和意义——丘成桐
今天要讲的是数学的内容、方法和意义,这原是苏联人写的一本书的书名,和今天的演讲内容借过来作为演讲的名称。
 
   今天是北大百周年校庆,五四运动便是北大学生发动的。作为演讲的引子,让我们先简略地回顾一下“五四”前后中西文化之争。十九世纪中业以后,中国对西文科技的认识,是“船竖炮利”,在屡次战争失利后,张之洞提出了“中学为体、西学为用”的主张,即以传统儒家精神为主,加入西方的技术。到了五四运动前后便有了科玄论战。以梁漱溟为主的一派以东方精神文明为上,捍卫儒学,以为西方文明强调用理性和知识去征服自然,缺乏生命之道,人变成机械的奴隶;而中国文化自适自足,行其中道,必能发扬光大。其时正值第一次世界大战结束,西方哲学家罗素等对西方物质文明深恶痛绝,也主张向东方学习。另一派以胡适为首者则持相反意见,他们以为在知识领域内科学万能,人生观由科学方法统驭,未经批判及逻辑研究的,皆不能成为知识。
 
   科玄论战最终不了了之,并无定论。两派对近代基本科学皆无深究,也不收集数据,理论无法严格推导,最后变得空泛。其实这便是中国传统文化之一特点。一方面极抽象,有质而无量,儒道皆云天人合一,禅宗又云不立文字,直指心性。另一方面则极实际,庄子说“蔽于天而不知人”。古代的科学讲求实用,一切为人服务,四大发明之一指南针、造纸、印刷术、火药莫不如此。要知道西方技术之基础在科学,实际和抽象的桥梁乃是基本科学,而基本科学的工具和语言就是数学。
 
   历代不少科学家对数学都有极高的评价。我们引一些物理学家的话作为例子。R.Feyman在「物理定律的特性」一书中说我们所有的定律,每一条都由深奥的数学中的纯数学来叙述,为什么?我一点也不知道。E.Wigner说数学在自然科学中有不合常理的威力。F.Dyson说:在物理科学史历劫不变的一项因此,就是由数学想像力得来的关键贡献,基本物理既然由高深的数学来表示。应用物理,流体等大自然界的一切现象,只要能得到成熟的了解时,都可以用数学来描述。写过「湖滨散记」的哲人梭罗也说有关真理最明晰,最美丽的陈述,最终必以数学形式展现。
 
   其实数学家不只从自然界吸收养分,也从社会科学和工程中得到启示。人类心灵中由现象界启示而呈现美的概论,只要能够用严谨逻辑来处理的都是数学家研究的对象。数学和其他科学不同之处是容许抽象,只要是美丽的,就足以主宰一切,数学和文学不同之处是一切命题都可以由公认的少数公理推出。数学正式成为系统性的科学始于古希腊的欧机里德,他的「几何原本」是不朽名作。明末利玛窦和徐光启把它译成中文,并指出“十三卷中五百余题,一脉贯通,卷与卷,题与题相结倚,一先不可后,一后不可先,累累交承渐次积累,终竟乃发奥微之义”。复杂深奥的定理都可以由少数简明的公理推导,至此真与美得到确定的意义,水乳交融,再难分开。值得指出,欧机里德式的数学思维,直接影响了牛顿在物理上三大定律的想法,牛顿距著「自然哲学的数学原理」与「几何原本」一脉相承。从爱因斯坦到现在的物理学家都希望完成统一场论,能用同一种原理来解释宇宙间的一切力场。
 
   数学的真与美,数学家的体会深刻。Sylvester说“它们揭露或阐明的概念世界,它们导致的对至美与秩序的沉思,它各部分的和谐关联,都是人类眼中数学最坚实的根基”。数学史家M.Kline说“一个精彩巧妙的证明,精神上近乎一首诗”。当数学家吸收了自然科学的精华,就用美和逻辑来引导,将想像力发挥的淋漓尽致,创造出连作者也惊叹不已的命题。大数学家往往有宏伟的构思,由美作引导,例如Weil猜想促成了重整算数机何的庞大计划,将拓扑和代数几何融入整数方程论中。由A.Grothendieck和P.Deligne完成的Weil猜想,可说是抽象方法的伟大胜利。回顾数学的历史,能够将几个不同的重要观念自然融合而得出的结果,都成为数学发展的里程碑。爱因斯坦将时间和空间的观念融合,成为近百年来物理学的基石;三年前A.Wiles对自守型式和Fermat最后定理的研究,更是扣人心魄。数学家能够不依赖自然科学的启示得出来的成就,令人惊异,这是因为数字和空间本身就是大自然的一部分,它们的结构也是宇宙结构的一部分。然而,我们必须紧记,大自然的奥秘深不可测,不仅仅在数字和空间而已,它的完美无处不在,数学家不能也不应该抗拒这种美。
 
   本世纪物理学两个最主要的发现:相对论和量子力学对数学造成极大的冲击。广义相对论使微分几何学“言之有物”,黎曼几何不再是抽象的纸上谈兵。量子场论从一开始就让数学家迷惑不已,它在数学上作用仿如魔术。例如Dirac方程在几何上的应用使人难以捉摸,然而它又这么强而有力地影响着几何的发展。超对称是最近二十年物理学家发展出来的观念,无论在实验或理论上都颇为诡秘,但借着超弦理论的帮助,数学家竟能解决了百多年来悬而未决的难题。超弦理论在数学上的真实性是无可置疑的,除非造化弄人,它在物理上终会占一席位。
 
   上世纪末数学公理化运动使数学的严格性坚如盘石,数学家便以为工具已备,以后工作将无往而不利。本世纪初Hilbert便以为任何数学都能用一套完整的公理推导出所有的命题。但好景不常,Godel在931年发表了著名的论文“「数学原理」中的形式上不可断定的命题及有关系统I”。证明了包含着通常逻辑和数论的一个系统的无矛盾性是不能确立的。这表示Hilbert的想法并非是全面的,也表示科学不可能是万能的。然而由自然界产生的问题,我们还是相信Hilbert的想法是基本正确的。
 
   数学家因其品禀各异,大致可分为下列三种:
 
(一)创造理论的数学家。这些数学家工作的模式,又可粗分为七类。
 
●从芸芸现象中窥见共性。从而提炼出一套理论,能系统地解释很多类似的问题。一个明显的例子便是上世纪末Lie在观察到数学和物理中出现大量的对称后,便创造出有关微分方程的连续变换群论。李群已成为现代数学的基本概念。
 
●把现存理论推广或移植到其它结构上。例如将微积分由有限维空间推广到无限维空间,将微积分用到曲面而得到连络理论等便是。当Ricci,Christofel等几何学家在曲面上研究与座标的选取无关的连络理论时,他们很难想像到它在数十年后的Yang-Mills场论中的重要性。
 
●用比较方法寻求不同学科的共同处而发展新的成果。例如:Weil比较整数方程和代数几何而发展算数几何:三十年前Langlands结合群表示论和自守形式而提出“Langlands纲领”,将可以交换的领域理论推广到不可交换的领域去。
 
●为解释新的数学现象而发展理论。例如:Gauss发现了曲面的曲率是内蕴(即仅与其第一基本形式有关)之后,Riemann便由此创造了以他为名的几何学,成就了近百年来的几何的发展;H.Whitney发现了在纤维丛上示性类的不变性后,Pontryagin和陈省身便将之推广到更一般的情况,陈示性类在今日已成为拓扑和代数几何中最基本的不变量。
 
●为解决重要问题而发展理论。例如J.Nash为解决一般黎曼流形等距嵌入欧氏空间而发展的隐函数定理,日后自成学科,在微分方程中用处很大。而S.Smale用h-协边理论解决了五维或以上的Poincare猜想后,此理论成为微分拓扑的最重要工具。
 
●新的定理证明后,需要建立更深入的理论。如Atiyah-Singer指标定理,Donaldson理论等提出后,都有许多不同的证明。这些证明又引起重要的工作。
 
●在研究对象上赋予新的结构。Kahler在研究复流形时引入了后来以他为名的尺度;近年Thurston在研究三维流形时,也引进了“几何化”的概念。一般而言,引进新的结构使广泛的概念得到有意义的研究方向。有时结构之上还要再加限制,如Kahler流形上我们要集中精神考虑Kahler-Einstein尺度,这样研究才富有成果。
 
(二)从现象中找寻规律的数学家。这些数学家或从事数据实验,或在自然和社会现象中发掘值得研究的问题,凭着经验把其中精要抽出来,作有意义的猜测。如Gauss检视过大量质数后,提出了质数在整数中分布的定律;Pascal和Fermat关于赌博中赔率的书信,为现代概率论奠下基石。五十年代期货市场刚刚兴起,Black和Scholes便提出了期权定价的方程,随即广泛地应用于交易上。Scholes亦因此而于去年获得诺贝尔的经济学奖。这类的例子还有很多,不胜枚举。
 
   话说回来,要作有意义的猜测并非易事,必须对面对的现象有充分的了解。以红楼梦为例,只要看了前面六七十回,就可以凭想像猜测后面大致如何。但如果我们对其中的诗词不大了解,则不能明白它的真义。也无从得到有意义的猜测。
 
(三)解决难题的数学家。所有数学理论必须能导致某些重要问题的解决,否则这理论便是空虚无价值的。理论的重要性必与其能解决问题的重要性成正比。一个数学难题的重要性在于由它引出的理论是否丰富。单是一个漂亮的证明并不是数学的真谛,比如四色问题是著名的难题,但它被解决后我们得益不多,反观一些难题则如中流砥柱,你必须将它击破,然后才能登堂入室。比如一日不能解决Poincare猜测,一日就不能说我们了解三维空间!我当年解决Calabi猜测,所遇到的情况也类似。
 
   数学家要承先启后,解掉难题是“承先”,再进一步发展理论,找寻新的问题则是“启后”。没有新的问题数学便会死去,故此“启后”是我们数学家共同的使命。我们最终目标是用数学为基础,将整个自然科学,社会科学和工程学融合起来。
 
   自从A Wiles在1994年解决了Fermat大定理后,很多人都问这有什么用。大家都觉得Fermat大定理的证明是划时代的。它不仅解决了一个长达350年的问题,还使我们对有理数域上的椭圆曲线有了极深的了解;它是融合两个数论的主流——自守式和椭圆曲线——而迸发出来的火花。值得一提的是,近十多年来椭圆曲线在编码理论中发展迅速,而编码理论将会在电脑贸易中大派用场,其潜力无可估计。
 
   最后我们谈谈物理学家和数学家的差异。总的来说,在物理学的范畴内并没有永恒的真理,物理学家不断努力探索,希望能找出最后大统一的基本定律,从而达到征服大自然的目的。而在数学的王国里,每一条定理都可以从公理系统中严格推导,故此它是颠扑不破的真理。数学家以美作为主要评选标准,好的定理使我们从心灵中感受大自然的真与美,达到“天地与我并生,万物与我为一”的悠然境界,跟物理学家要征服大自然完全不一样。
 
   物理学家为了捕捉真理,往往在思维上不断跳跃,虽说是不严格和容易犯错,但他们欲能把自然现象看得更透更远,这是我们十分钦佩的。毕竟数学家要小心奕奕、步步为营,花时间把所有可能的错误都去掉,故此这两种做法是互为表里,缺一不可的。
 
   在传统文化中,我们说立德,但即从不讨论如何求真,不求真,则何以立德?我们又说“温柔敦厚,诗教也”,但只是含糊的说美,数学兼讲真美,是中华民族需要的基本科学。

中英文论文写作的最大不同

以前跟博士老板写文章的时候,老板面授机宜,说了一些写作要点。大体知道宏观上怎么安排文章了。然后跟博士后老板写第一篇文章的时候自我感觉总体上的把握没有问题,结果却发生如下对话:
 
 
 
我写(为了说明简单,这样都翻译成中文):
A是由于B的存在造成的,
而C会产生B,
为了减少A,我们必须减少C。
 
 
 
老板说:你这样写,我不知道第二句为什么要提C呢?
我一愣,很奇怪:因为C会产生B啊?(心想这不是很明显吗?)
老板改成了:
A是由于B的存在造成的,
B会被C所诱发,
所以减少C,就会减少A。
 
 
 
我一看,换了个表述而已,有什么不同吗?百思不得其解。问老板为什么要这么改,老板还是那句话,按原先的写法,她不知道为什么要提C。我最后还是不明白。直到有一天,我看到了这篇文章《The Science of Scientific Writing》by George D. Gopen and Judith A. Swan。
 
 
 
原来,句子有所谓的the stress position,就是读者期望看到重要的信息和新信息的位置,所以写作的时候要把你重要的东西放在这个位置。句子还有所谓的the topic position,它建立了读者对这个句子的期望。更奇妙的是,不同的语言,由于阅读习惯的不同,它的the stress position是不太的:中文的the stress position是在句子的前面,英文是在句子的后面,这叫save the best for the last。对英文来说,它的the topic position是在句子的前面,而中文没有the topic position。最理想的英文句子的组织方式是用前面的the topic position来提到前面的旧信息,这样读者就知道这个句子是要跟旧信息相关的,而用后面的the stress position来提到新信息,这样一个句子实现了一个建立旧信息和新信息联系的过程。这样一句接一句形成了信息的流动和逻辑的建立或转换。
 
反过来看我的表述,我还是用中文的表达习惯,在第二句中把新信息放在了前面。对我来说这个逻辑没有问题。可是对于英文国家的人来说会很别扭,因为在 他们期望看到旧信息的地方看到了新的信息,他们就会迷失。而在应该有新信息的地方看到旧信息,他们又会失去兴趣。虽然最终他们也能懂,但是对他们来说,读 起来就很费劲。我老板改的,就完全符合英文读者的阅读习惯。所以有人说,你看一篇英文文章,如果读起来特别顺口的话,往往它是中国人写的,因为我们对the stress position有相同的习惯。知道了上面组织英文句子的原则,写起来就可以把句子搞的紧凑一点。
 
 
 
还有一个问题是,有时候我们不能完全做到每次都用旧信息带入新信息,那怎么办呢?那就要借助连词的作用了。这一点,中英文差别倒不是很大。
 
比如:A类材料中会有性质B。
B很普遍,可以用理论C解释。
但是,最近在A类中发现有某些材料没有性质B。
其中第三句的A在第二句没有出现,而如果把在第二句有出现的B作主语又不好造句,只好利用连词:但是。
 
 
 
同样的道理推广到段落:我们最好在每一段的开头用一句topic sentence(主题句),句子前半部分提到上一段的主题名词,后半部分提到这一段的主题。通过这样,我们又把大的单元,段落,给有机的连接起来了。
 
 
 
至于英文写作中的整体结构问题,实际跟中外区别倒不大。想想近代科学都是西方传到中国,所以这些模式都是西方的模式。只不过我们有自己的语言,所以最大的区别还是句子的组织上。
 
 
 
当明白中英文论文写作的最大不同后,你就可以写出比较符合英文阅读习惯的文章来了。
 

书脊指书的厚度,最准确的计算方法是:
P数除以2乘以0.001346乘以纸张克数=书脊。
P数:指同种纸张总页数,通常一张A4纸为2P,设计公司计算P数是按210mmX285mm,计算,即大度16开计算。
无论多大开度的书,计算书脊时P数就是计算同种纸共多少页,如有不同纸,再计算其他纸的厚度,最后相加得书总厚度,如:一本书:内页80g书写纸共240P,中间有16P157g双铜,算书脊。
书写纸厚度:240÷2×0.001346×80=12.92mm。
铜版纸厚度:16÷2×0.001346×157=1.69mm。
书总厚度(书脊):12.92mm+1.69mm=14.61mm。

 

“侯沉,你的论文充斥着垃圾”: 浅谈英文科研写作

面对着满屋子的师生,我的博士委员会中的一位美籍匈牙利教授直言不讳地在我的博士答辩会上说:“沉,我要把你的工作和你的论文分开来评论。”
 
   我一头雾水,什么意思?我的论文不就是我的工作,我的工作不就是我的论文吗?
 
   “你的工作不错,这个这个和那个那个,但是你的论文充斥着垃圾。写作一塌糊涂,我读着读着都想撕了它。” (其实他用了一个比”垃圾“更不堪入耳的词儿,和犬科动物有关,大家都能猜到,我就不点明了,呵呵。)
 
   台下的哥们一脸坏笑,台上的我一脸讪笑,满屋的人哄堂大笑。东欧国家的人好像不讲政治正确啊,是什么就说什么(所以人家遇到的恐怖袭击也少呢。)
 
   我博士导师从来不发火,唯一一次抱怨就是博士答辩之前:”你看那个阿根廷小妹妹,人家母语也不是英语,比你来美国还晚,人家毕业论文写的可以直接拿去发表。你,你媳妇儿是说英语的,你也不让她给你改改,你自己看的懂吗?“
 
   博士后的老板在我刚进去时曾经评论:你的写作是hopeless。博士后二老板比较客气一些:我读你的东西有一些困难。
 
       2009年,我痛定思痛,知耻后勇,发奋图强,悬梁刺股,花了一年时间废寝忘食地提高英语写作。一年之后,基本上每篇文章,审稿人或编辑都会称赞写的不错,很容易懂(well written, easy to follow),再没有在写作上被人质难过。前年写了一篇长文(我自认是自己最重要的工作),给审稿人很深的一个印象就是“写的很好”,嘿。
 
   今天周末,这就花点儿时间和博士生,博士后,还有青椒们交流一下我的逆袭过程。以我这么差的底子都可以逆袭(大学四级68分,托福惨不忍睹,写作被所有的老板批判),那我相信除了陈楷翰陈疯子,所有的人都可以逆袭。
 
   先声明一下,本文是交流理工科的初级或中级科研论文写作的经验,文艺创作或高级科普写作不在此列。而且主要是集中在生命科学这种使用自然语言的学科。像物理数学这种使用公式语言比较多的学科,可能用处也不是很大。另外,此文的目的是完完全全功利性的,讲的就是你辛辛苦苦学好文武艺后,如何卖与帝王家。蔡宁兄和陈楷翰陈疯子这种超然物外的,准备好清水洗耳朵哈。
 
   科研写作最常见的毛病就是没有逻辑。这一点和语言无关,中国人和美国人没有区别。我博后老板经常说,很多美国博士生写的东西也没法看。
 
   关于写作的逻辑,网上已经有很多文章了。比如图表的取舍,顺序的安排等等。最重要的就是要讲一个完整的故事,不要把做过的实验,事无巨细统统写出来。你自己做的,你觉得重要,但别人不一定这么想。套句说俗了的话;一定要“舍得”,有舍才有得。我经常给学生说,除非是你奶奶,连你妈都没有兴趣和耐心听你唠叨,80%的读者看你的文章都是一掠而过,所以一定要言简意赅。(写博文,可以稍微话篓子一点儿哈。)
 
   另外,你在做的过程中,可能走过弯路,思路设计等等都有过变化。这些弯弯绕绕就不要写了。你自己知道就行,别人没有和你一起走这个过程,也不知道你的心理活动,所以也看不懂。你就直指人心,见性成佛即可。(当然,如果负面结果很重要,那当然要提一下,省得别人也走弯路。这个分寸如何把握,就是考较工夫的时候。把握不好,就去问问前辈)
 
   中国学生,能读到博士这个阶段,脑子一般都是清楚的,在叙述的逻辑方面应该没什么太大的问题。所以我下面主要交流一下如何提高科研写作的英文。讲英文科研写作的文章有很多,大多数都很长,你看了也不一定都能记住。我就讲三个非常简单的要点,保证你能记得住。
 
   第一,提高科研写作水平,最最最最重要的重中之重,我最重要的心得,说出来很简单,就是两个字:“耐心”。 写一篇文章时一定要有耐心。这个可能和我个人有关。我一气呵成的文章,人家往往批的我满地找牙。我精雕细刻,苦苦思索出来的文章,基本上都会得到好评。所以,我个人的经验就是,革命不是请客吃饭,不是绣花做文章,不能。。。哦,不对,我是要说,科研写作不是写诗,不要文不加点,不要李白,科研写作是绣花,要的是贾岛—两句三年得,一吟双泪流。一定要慢慢写,性子要压住(所以陈疯子写不了呢,他不肯屈就读者嘛)。
 
   而且,写完了之后,一定要放一两个星期再投出去。投稿之前一定要再花几天时间改几遍。我敢赌一桌满汉全席—你放几周之后再看,肯定有你自己看不明白的地方,肯定有你觉得不通的地方。你当初写的时候,胸中“自有丘壑”,其实很多时候是主观地自说自话,并没有客观的把意思表达出来。很多句子写出来,靠的是你的意识流,自己写起来痛快,别人却不知你什么意思。你自己在你胸中的“丘壑”里弯弯绕绕,别人不一定能跟上。放个几周,胸中的激情消失了,意识流也都流光了,就可以比较冷静客观地看自己的文章。肯定有诸多需要修改的地方。
 
   我只知道三个人写作是一气呵成的。一个是哈佛大学的伟大的生态学家爱德华威尔逊。威尔逊写作几乎不用改第二遍,写完就可以付梓。但是人家能得普利策奖(美国最大的文学奖),你行吗?你不多改几遍?还有一个是清华的施一公教授,据他说,他有一次一晚上就把一篇投Nature的文章写完了。这个我是极其佩服。第三个是中科院的邢老人家,科学网的网友都很熟悉了,他LRJ是江湖有名的快手,写文章也经常是下笔千言,立马可待。但是我想,他搞的理论物理,公式是主要内容,描述性的文字是次要的,而且学物理的人自有一套语言和逻辑体系,有别常人,所以我们学不了。 我博后老板是学物理的英国人,大家都说他写作极佳,但是他写一篇生物的Nature/Science也得一个月。
 
   好几年前看过一个加州大学的大牛讲如何写基金标书的视频,给我印象很深。此大牛(美国的母语写作者)多少年来NIH的基金没有断过,而且经常同时拿两个(美国的基金基本是看本子,不看人的。我认识两个院士很多年都拿不到钱)。大牛说,他写一个本子,要改几十遍甚至上百遍(你能想象得到吗)。最后一版和第一版比起来可以面目皆非。所以他建议写作一定要早早的开始。写完了之后,每隔一段时间就拿出来改一遍。他说他改写的稿子能有一尺高。在那个视频里,我记得他说,then  rewrite it, then rewrite it, then rewrite it,一口气说了能有六七遍强调这个事儿。
 
   我觉得,写文章和写标书虽然有不同,但是道理差不多,一定要提早开始,改个几遍再投。
 
   第二,一定要秉持我最不齿的西方还原论思维。中国人的思想有时候很深,很复杂,想表达的意思很精妙。所以中国人讲存乎一心的哲学,讲究只可意会不可言传,讲究尽在不言中。这种哲学思想已经深入到中国人的基因里了。但是,你去给编辑和审稿人“拈花微笑”吗?人家回过头来给你一棒子。禅宗有个词叫棒喝。什么是棒喝?这就叫棒喝!(此处应该有幅漫画。)
 
   西方人的思维特点就是“肢解”“还原”。从小学开始,他们就是这样教育的,一直到生意场的经营管理测略,什么“和乔布斯学管理”之 类的烂书,都强调先把一个复杂的事物肢解还原成多个简单的事物,再一一处理。(其实我隐隐觉得,西方很多东西,包括人工智能,都是对真实世界的还原模拟。东方人用整体的思维方式从上而下的去认识宇宙自然,西方人先还原了之后,自下而上的,通过无限接近的模拟方式去认识事物。所以他们认识到的都是近似的假东西。但是在大多数情况下,比如苹果手机诸多的实体模拟程序,因为太像了,人们都感觉不出来。)
 
   兰若寺的燕赤霞大侠说,道可道,非常道。这话可以反过来说,若要道,弃常道。什么意思呢?意思就是,既然你要发论文,就别学陈疯子故作高深了,一定要繁杂化清淡。你既然想写论文,那就是要和人交流,要和人交流,那你的思想只要“道”出来,就必然要失真。你要做的就是,通过细细的分割,把失真率降低。
 
   好像说的太玄了哈。高大上的玄谈背后是一个非常非常非常简单的规则:重写所有的长句子!
 
   就这么简单。
 
   中国人写英文,喜欢从句套从句,从句再套从句。你把所有三个以上的从句都改成两个以下,你文章的接受率就大大提高了。
 
   欧洲人我不清楚。美国人,一个句子里若出现三个以上互相有逻辑关系的事物,普通人的耳朵就关上了(没有统计数据,是我自己的感觉)。科研人员能好一些。但是如果你把一个含两个关系的句子,分割成两个各含一个关系的句子,一般的科研人员就会觉得读起来轻松的多。当然,我就是举这么个例子,很多时候是不可能分割 开的。但是,宗旨就是把一个复杂的句子分割成多个简单的句子。这样做,读起来会稍微显得罗嗦一些,但是别人肯定能读得懂。(这也要有个度,也不能太罗嗦了,从句还是有用的。但是总而言之就是要化繁就简。)
 
   从操作层面来讲,如果一个句子超过了Word文件的四行,你就一定要想办法或者缩减或者分割,三行其实都嫌长了。很多美国人的大脑不足以处理这么长的信息。(除非你英文特别好,用词捏拿都很有分寸,那就没有这个限制了。但是,如果你的英文这么好,看我这篇博文就是浪费时间了。)
 
   第三,用小词儿,不用大词(jargon,自己去查这个词的意思)。除了专业词汇,不要显示你的词汇量。马克吐温说,要想写好,很简单,杀掉所有的形容词。科研写作是一样的,尤其是动词,能用use,就不要用utilize。形容词,凡是太花的,全部删掉。平平实实。这不是文学创作,不是写小说,不需要词汇量。我博后的大老板有时候会在讨论部分加点儿比较感性一点的词,不多,两三个而已。即就是这样,博后的二老板还是很反对,能删的都删。
 
   儒林外史里有个秦翰林。秦翰林很看不起放浪形骸不求功名的天长县的杜少卿,他让自己家族里入了学的子弟们在书桌上都贴一个小纸条—“不可学天长杜仪”(杜仪,字少卿)。如今我说,要发SCI文章的青年博士生博士后,你们听我的,在电脑的桌面上都贴一句话:”不可学泉州陈楷翰“。
 
   你们去看看陈楷翰正儿八经的专业博文(散文不算)。。。那是一个大词接一个大词,专有名词不解释不说,把些动词都搞得那么复杂。除了他师兄戎可和我,不知道谁有耐心去一个一个请教他都是什么意思。至于看懂看不懂?那是谁也看不懂(我请教之后,能看懂七八成),比相对论还玄。所以,同学们,如果要发SCI论文,就去看陈楷翰的博文,他怎么做,你们就反着做,庶几能发。
 
   差不多就这些了。还有一些细节的技巧,比如动词不要放在很长的词组后面(美国人读句子的时候下意识地在找动词在哪里 ,如果你放在句子的后半段,他一下子看不到,心里就会着急)等等。那些我说了你也记不住,我也就不说了。
 
   词汇量,不重要。你的目标不是诺贝尔文学奖,也不是写小说。日常的词汇加专业词汇足够了。何况还有我刚才说的第三条。
 
   语法,重要,但是简单。真的很简单。其实常见的语法错误就是那么多。找一本新东方的托福的语法书。认真看,认真做所有的练习,两个星期,保证你能杜绝90%的语法错误。剩下的10%不好解决,但是不会让审稿人产生厌恶的情绪。到现在,什么地方加the,什么地方不加the,我还是搞不清楚(其实美国人有些时候对加不加the也有争议),但这些都对你发文章影响不大了。
 
   对于英文程度比较高的科研工作者,想写的精致文雅一点的,我100次强烈推荐这篇文章:“The science of scientific writing”。网上一搜就有,免费下载的。这文章我看了好几遍了,每隔几年看一遍。现在能达到文章要求的50-60%,还在继续努力中。内容是什么我就不废话了,总之就是科研写作的九阳真经,绝对的宝典。
 
   总结一下:英文的科研写作的宗旨就是化繁就简,屈尊俯就读者,让人读着不累。如何才能不累呢:整篇文章要有逻辑;每句话要短,尽量一句话一个意思,不要把复杂的意思挤到一句话里;每个词要简单,用小词儿,不用大词儿。为了保证这几点,一定要慢慢的多改几遍。
 
   最后叮嘱一句,怕你们忘了,科研文章,不要李白,不要陈楷翰,不要那高,不要那多雪。
 
 
 
又及:我上面说的完全是八股实用文。而且是初级和中级层次的,单纯就是为了博士生青椒们发学术文章而已。而真正的科研论文也可以写的很有文采,爱德华威尔逊,还有很多有文化底蕴的科学家,都可以写得很漂亮。 只不过在现代社会(全民民主,什么事情都是全民参与),做什么事情的趋势都是向下看齐,实用即可。所以那种科学家/作家也越来越少。这其中如何做到平衡,如何做到通俗而不低俗,是一件很考较功夫的事情。
 
又又及: 好像有朋友误会了。我和陈疯子是铁哥们。我对他那是明贬暗褒,这么夸他,正是赞他特立独行,矫矫不群,他高兴着呢。  我是真心贬他,但是换个角度看,实际上就是真心褒他。我是“何时忘却营营”,他是已经“小舟从此逝“了。
 

爱情故事

鸡尾酒“玛格丽特”的爱情故事

没有一杯鸡尾酒能象“玛格丽特”这样独特。不只是它的味道,更是在它背后那催人泪下的爱情故事…… 
 天阴沉沉的,朔风北吹,玛格丽特趴在阁楼的窗户上,嘴里哈着热气把窗户玻璃上霜花化掉,她瞪着明亮的大眼睛看着镇子尽头的那条小 
马路上,风雪交夹的路上一个人也没有,她在等简的到来。 
 他答应过她今天要来带她到城里的酒吧去看看,那可是洛杉矶呀,从墨西哥移民来到这个大城市边上的小镇里,玛格丽特一次都没有走出这个 
小镇子的管辖范围,只有一次玛格丽特到镇子西南的林子里,追她养的那只从老家带来的那只可爱的牧羊犬,算是她离家最远的一次了。 
 也就是那次,玛格丽特认识了简·雷得沙--一个和她一样有着棕红色头发的英俊的小伙子。简在洛杉矶一个叫Old friend的小酒吧里做侍应生。 
简和玛格丽特初识的时候还是因为绿蒂--那只凶悍的牧羊犬。当简帮玛格丽特逮住了那只要冲向森林的牧羊犬时,被绿蒂狠狠地咬住了手臂。要不是玛格丽特及时赶到一声娇叱,他想也许这只胳膊一定会被咬掉了。而也就在这时,简才发现玛格丽特的美,那是城里的姑娘们无法比的。简和玛格丽特成了一见衷情的恋人。 
  每每和玛格丽特在一起时,简都要问候那只牧羊犬,他总会笑着对玛格丽特说:“我的小玛蕊,我要绿蒂做我们结婚时的见证犬,它是那样的可爱,它会给我们生一窝漂亮的小baby,就象你,我要你给我生三个女儿,都象你一样的漂亮。”。这时,玛格丽特会靠在简的肩头,用她纤长的手臂揽住简的腰,闭上她那长长睫毛的绿眼睛,幸福地憧憬着未来。那可是她小的时候就有的梦啊。  
 
简的父亲是洛杉矶大学的一位知名的教授,是个老保守党。他从知道了儿子在和一个墨西哥的女孩子的交往后,就怒发冲冠地冲简怒吼着要把他赶出这个家门,除非他断绝和那个女孩的来往。简就是不明白一向在家中和蔼可亲的父亲为何动了这么大的肝火,他去问他的母亲,他母亲也叹了口气并不正面回答他。于是简一气之下就搬到了离酒吧不远的老菲尔德大街的一处公寓房里。他想也许过段时间就会好的吧。就这样僵持了有一年了。 
简准备挣完这个星期的钱,交完学费与房租,剩下的钱就可以带玛格丽特到城里来看看。他答应过玛格丽特带她到酒吧里坐坐,并亲手给她调杯鸡尾酒喝。看到玛格丽特美丽的绿眼睛里流露出渴望的神色,简知道这是他第一次答应她带她到城里看看,对于一个不谙世事的乡下姑娘来说,该是不小的诱惑。想到这简浑身就充满了兴奋的喜悦,就连在酒吧干活时都哼着小曲。酒吧的老板--亨利·杜维就喜欢看简干活时朝气活力的样子,那是他这个酒吧里的一个景色,许多的老客人都愿意和简打着招呼,而简的热情好象永远也使不完一样。 
  简告诉亨利,后天也就是他休息的日子,他要带他的女朋友玛格丽特到酒吧来喝酒。亨利咧开大嘴,笑着把这消息瞬间传到了酒吧的每一个角落。杰瑞·卡休大声地问简:“嘿!小伙子,把你的玛丽带来吗?那可是个大美人呀,她有着地中海一样的绿色的眼睛,她的红棕色的头发象墨西哥城外的野马驹,那就是在墨西哥人里也不多见的啊。还有她的鼻子,她的鼻子,高傲的象个公主,她的红红的嘴唇,噢!那嘴唇呦性感的象玛丽莲·梦露。多想吻她一下呀!可她是你的了!”简开怀笑着,他曾对这里每一个老顾客描绘着他的玛丽的动人的美丽。直到上一次大家去小镇打猎,很多人都见到了玛格丽特的美貌。杰瑞当时就对玛格丽特跪下说道:“简的玛丽,我心中的女神,当简这个可爱的小伙子无情地抛弃你时,可别忘了我老杰瑞,我会痛打他一顿,再把你关在我的象牙笼子里,你会快活地歌唱。”老杰瑞的这番表演曾惹得大家轰堂大笑。但美丽善良的玛格丽特用她纤细修长的双手拉起老杰瑞,在他卸顶的光头上轻轻地吻了一下说:“杰瑞大叔,会有个漂亮的姑娘爱上你的,上帝在看着你,他会为你选个美丽的新娘子的。”杰瑞·卡休象喝醉了一样站了起来,他满脸通红地有些不好意思了,但旋即他高高举起酒杯对四周的人大声叫道:“我说绅士们,来吧,为这对儿年轻人举杯吧,祝福他们,象祝福我们的儿子与女儿!” 老杰瑞的话音还没落,那边角落里正打着“休斯顿国王”纸牌的加利安奴·得比律斯也高声叫道:“简,后天我会带来我烤熟的整只的羊腿,那是上次你的玛丽--我们这里的小天使最爱吃的了,我还会让我家里的人都来看看天使一样美的玛格丽特。对吗,我的亨利大叔?” 
  亨利哈哈笑着应和着:“是呀是呀,小玛丽是这里的小天使,老伙记们有什么好吃的就都拿来吧,简,明天放你个假,去理理发,买件新的衬衫,还有你那双旧的皮鞋也要换掉,这些钱我会从你三个月的工资里慢慢扣除的。去吧,小伙子!” 
  让万·德西这个老法国骑师也举起手中的酒杯对简说道:“简,去吧,接你的天使去吧,还是骑上我的那匹银王子去吧,用它拉回你的小玛丽,你就是她的白马王子。多可爱的玛格丽特,就象我的女儿……”让万喝了口酒又开始在嘴里嘟囔着谁也听不清的法语了。 
   夜已很深了,客人们陆陆续续地走出了酒吧。打烊的时候,简问亨利:“亨利大叔,我想在后天为玛格丽特调一杯酒,可用什么做基酒好呢?” 
  亨利想了想说道:“就用Tequila酒吧,那是墨西哥的特产,也是很有意义的啊。” 
  简边擦着地,边琢磨着亨利的话。回到家里,简开始精心地计划后天的安排了。他想明天回趟家,想让父亲与母亲都来看看他们未来的儿媳妇是多么地受人欢迎,是多么的可爱。简不知道成不成,但他觉得都过去有一年了,父亲该气消了。 
  当简睁开眼睛的时候,已是快到中午了,简匆匆地吃了块面包煎鸡蛋,喝了杯牛奶,就向回家的路上走去。家里静悄悄的,只有保姆薇薇安在,她在厨房里烤着奶油梨馅饼,那是简最爱吃的甜点了。浓郁的法式甜点香味在薇薇安的身旁飘了过来。 
   简走了过去,从后面抱住了这个胖胖的黑人老太太,薇薇安装做被一大跳。因为在这个家里只有简象自己的儿子一样,从小就和她做这样的把戏。 
  简回来了,是薇薇安最高兴不过的事了。她愉快地说:“你回来了,我的小安迪,瞧,这是我为你烤的馅饼,本来还要给你送去,这回可好了,省得让我这老骨头再跑两条街了,就快烤好了。” 
  简笑着对薇薇安说道:“薇薇安,我是回来请父亲的,明天我要带玛格丽特到我工作的酒吧,那有很多的人要为我们俩庆祝,我也要在那里向我的玛丽正式地求婚,请她嫁给我,我想请你们都去。” 
  薇薇安听后快活地笑了:“老让万早就对我说了,玛格丽特是个好姑娘,是个漂亮的美人,我的小安迪,你是个幸福的人,明天我对你父亲说,他今天要很晚才回来,你母亲去医院了,刚才有个病人需要她去动手术,我的小安迪,你今天在家住吗?” 
  简摇摇头说:“不了,我还要去买衬衣与皮鞋,亨利放了我一天的假,我要好好准备准备,明天一大早就要出发去接她。我走了薇薇安,明天你们一定要来呀!” 
  薇薇安边送着简,边保证着:“会的,我们都会去的,老爷就是不去我也会拖着他去的,我的小安迪求婚的日子,我们都会去的,哦!我的上帝,主啊!快让我见到我的小安迪的天使吧。那是您怎样的仁慈呀!” 
天有不测风云,就在第二天的黎明时,简发现天阴得厉害,但他还是骑上银王子向城外奔去。路上冷风在飕飕地吹着,夹杂着雪粒打得脸生疼。银王子埋怨着踏着蹄子向前奔去,它不明白在这样的天气里,为什么简还要带它出来。但从简执着地驱赶它向前走着,它也好象明白今天对于简一定是个重要的日子。 
  适应了这鬼天气,一路上简与银王子也就不再忽急忽慢地跑了。雪渐渐地大了起来,天地间开始变得一片银白。放亮的天光被浓浓的雾霭阻隔着,虽隐约还看得见路面,但已没了往日里的色彩。简此时的心早已飞到了玛格丽特的身边,风雪中的急驰让简和银王子的头上身上都冒起了蒸气。 
  玛格丽特在窗前趴着,她微微皱了一下好看的眉毛,天气这样,简还会来吗?他要是这时来……。 
  玛格丽特胡思乱想着,就听见楼下妈妈在叫着:“绿蒂!绿蒂……回来,回来……” 
  她跑下楼去,听见妈妈在说:“玛丽,快快,绿蒂跑了出去,快去追呀。” 
  玛格丽特想也没想就冲出屋门,她一边呼唤着绿蒂的名字,一边四处寻看着。到了镇子口的路上,玛格丽特才发现一串绿蒂的脚印向西南的林子里跑去。犹豫了一下,玛格丽特不知该不该到林子里去。这几天镇子里的人们都在传说林子里常有只黑熊出没,她不知道该不该去找绿蒂。 
  忽然一阵马蹄声从远处的风雪里传了过来。玛格丽特转过头,一匹白马隐隐约约的出现在大路的尽头。那是银王子,玛格丽特激动地高声呼喊了起来:“简……简……我在这儿!” 
  听到玛格丽特的叫声,银王子脚下加速了奔跑,简也兴奋地高声回应道:“玛丽!……玛丽!……” 
  跳下马,简和玛格丽特紧紧地抱在了一起,简热切地吻着他心爱的姑娘,玛格丽特依偎在简的怀里轻声地问道:“简,下这样大的雪还赶来呀,瞧你脸上的汗……” 
  简揽过玛格丽特的腰,他笑着在问:“我的玛丽,下这样大的雪你还站在这里等我呀?” 
  玛格丽特这才想起绿蒂来,她连忙对简说:“简,是绿蒂,绿蒂跑了出来,它跑到林子里去了,可我不敢去追了。” 
  简疑惑地问道:“为什么?” 
  玛格丽特便把黑熊的事对简说了一遍。简笑了笑,拍了拍银王子鞍子上的枪囊,对玛格丽特说:“玛丽,你在这等着我,我不只把绿蒂给你带回来,还会给你带回来一只大笨熊,明年的圣诞夜你会和我们的孩子躺在温暖的熊皮上听我给你们讲故事的!”说完,简上马冲向了树林。 
 等了好久,玛格丽特也不见树林子里有什么动静。不由得着急了起来,简该不会出什么事吧?玛格丽特跺着渐渐发冷的脚,双手哈着气在原地来回来去地徘徊着,这时身后传来父亲的呼唤,回头望去,父亲和镇上的一些人跑了过来。 
  原来他们发现玛格丽特向镇外的树林走去,又听她的妈妈说绿蒂跑了,估计玛格丽特是准备进林子找绿蒂,众人这才着急了起来,当看到她一个人在这里徘徊,众人才舒了一口气。可听到玛格丽特说到简单人匹马的闯进林子找绿蒂去了,不由得为这个小伙子的勇敢所感动。玛格丽特听见众人在夸奖她的心上人,心里充满了甜蜜。 
正当众人商量着要去林子里接应简的时候,突然听见林子里传来一声枪响。接着银王子飞也似地冲了出来。玛格丽特连忙过去拉住缰绳,只见银王子的后臀上有四道血印,在雪白的马身上,格外的显眼。 
玛格丽特象疯了一样失声大叫着:“简……简……”,她翻身上马,驳转马头,双腿一夹,嘴里狂呼着向林子飞奔而去。 
  银王子虽很惧怕,但蹄下也丝毫没放松地向林中急弛。众人一下子被甩在了后面,大家高声呼叫着玛格丽特也发足急奔过去。 
 
  玛格丽特的耳边风声呼呼地在响,她根本听不见后面众人的呼唤。一阵急奔就到了林子的边上,银王子说什么也不肯再向前跑了。玛格丽特只好翻身下马,下马时她发现在马鞍的枪囊里还有一枝转轮手枪斜插着,她拔了出来,双手提起拖地的长裙向林子里跑去。 
  这时,她听见林子深处又传来一声枪声。玛格丽特急了,她大声地呼唤着:”简!……简!……你在哪?绿蒂!……绿蒂!……快帮帮我!……” 
  突然身边的草丛里一动,绿蒂嘴里叼着一只小兔钻了出来,玛格丽特连忙蹲下身,用手去抱绿蒂的头。 
  就在这时,绿蒂一声低吼,它呲出两排锋利的牙齿,从玛格丽特身边蹿了出去,与此同时玛格丽特听见简在大声地叫着:“玛蕊!……玛蕊!……,你别过来,我来救你!”   玛格丽特抬头向前方看去,她惊呆了,一只身材巨大的黑熊,恶狠狠地扑向正向她奔来的简。她惊得忘记了一切。突然绿蒂的身影一跃而起,它矫健地冲黑熊就是一口,绿蒂咬住了黑熊的左前爪,不住地甩动着头。简举起枪又放了下来,他冲绿蒂喊到:“绿蒂,闪开!” 
  绿蒂咬住黑熊不放,黑熊也发起疯来。玛格丽特看到这,举起手中的枪,一扣板机,一颗子弹正中黑熊的右臂。黑熊这才转过身发现向它开枪的玛格丽特,它狂呼一声一掌打在绿蒂的身上,绿蒂象一块布一样地飞了出去,当场晕了过去。 
  黑熊疯一样地冲玛格丽特大声叫着,它直起身,象一座黑塔一样地扑向玛格丽特,两只血淋淋的黑爪在所过的灌木上留下了无数的血红的点子。玛格丽特一下子惊得不知所措了,她往后退着,长长的裙子被灌木刮住了,撕开了一条大口子,也绊倒了她。就在这时,黑熊已到了她的眼前。 
就在黑熊直起身准备扑向玛格丽特的时候,简手中的枪响了,一颗颗准确的子弹打在了黑熊的后头部。黑熊一声撕人心脾的哀嚎,转过身又向已到了它面前的简扑去。简万万没想到黑熊会倒转过来扑向他,再退已来不及了,说时迟,那时快,简冷静地举起枪,瞄准黑熊的心窝处的那点白毛,扣动了板机。黑熊倒了下去,简在黑熊倒下的一瞬间,再次地扣动了板机,而这第三次的枪响,却没能打中黑熊,简看见黑熊身后的玛格丽特再次地倒了下去。 
  原来玛格丽特听见简在叫她和一声枪响,她看见黑熊转身扑向了身后的简,玛格丽特也不知哪来的气力,一下子爬了起来。她举起手中还握着的枪,对着黑熊的后脑就是一枪,与此同时简的第三枪也响了。 
  简没看见玛格丽特站了起来,不幸的事发生了。简的第二枪打中了玛格丽特,而此刻跑进林子里的众人正好看到了这悲惨的一幕。简惊呆了!他张大了嘴,不敢相信自己的眼睛,他心爱的姑娘倒在了自己的枪下。 
   “噢!不!不!不!……”简惊得说不出话来。他扑了过去。 
  简抱起玛格丽特的身子,哭着在喊:“噢!不!不!我的上帝!玛蕊!玛蕊!我做了什么?玛蕊!玛蕊!……噢!主啊!醒醒!醒醒!我的玛蕊!……” 
  玛格丽特艰难地睁开了她那美丽的绿眼睛,嘴角流出了鲜红的血。她缓缓抬起手,艰难地用手摸摸简蓬乱带血的棕红色的头发,简连忙伏下头。 
  众人围了上来,玛格丽特的父亲蹲了下来,他亲眼目睹了这一惨剧的发生。玛格丽特看看父亲,又看看简,用尽了全身最后的一丝力气,断断续续地说道:“简。我……的爱……人,我……走了,我……爱……你!不后……悔!我好…………冷……” 
  还没说完,玛格丽特的头一歪,倒在了简的臂弯里。简吓得紧紧地抱住了玛格丽特的头,放声大哭了起来。他用尽了力气在呼唤着玛格丽特的名字,一遍一遍,又一遍的,林子的上空回旋着简那凄惨的哀嚎……  
四个月后的洛杉矶,大街小巷都在议论一件事,本市大学的简因误击恋人玛格丽特,被洛杉矶警局起诉。但因死者的家属提出不让简承担任何责任,最后法庭在听取了证人及陪审团的意见后,宣布判简做社区义工半年,当庭释放。 
  简成了大街小巷的公众人物。可他的内心无法排解那份失去玛格丽特的痛苦。他来到了小镇住下,在西南的林子里,在玛格丽特倒下的地方,玛格丽特被埋在了那里,绿蒂也被埋在她的旁边。玛格丽特的墓志铭是这样写道:“勇敢的天使,为爱而与上帝同在。” 
  简每天都带上一瓶墨西哥的Tequila酒和一只与玛格丽特绿色的眼睛一样的青柠檬到那里和玛格丽特在一起,眼泪和痛苦已深深地把他和他心爱的姑娘埋在了一起。简从早喝到晚,直到醉在玛格丽特的墓前被人抬回家中,就这样过了一年。 
当人们渐渐忘了这件事后的一天,简还象往日一样来到了玛格丽特的墓前,他发现一个熟悉的背影,原来是他的父亲。他苍老了许多,他老泪纵横地给简讲了一件事,关于他的母亲。 
  原来简的亲生母亲也是个墨西哥人,在生下简没多久就抛弃了简和丈夫,改嫁他人了。所以,他恨死了墨西哥人,听到简也交了一个墨西哥女孩,他简直要气疯了。 
  但他万万没有想到玛格丽特会为救自己的儿子,死在了自己儿子的枪口下,而且玛格丽特的家人并没有因此而把简投入监狱。还在简来到小镇上时照顾他,他被这真诚的感情所感动。老雷得沙教授托人从墨西哥带回一丛郁郁葱葱的蓝色的龙舌兰花,带到了玛格丽特的墓前,祭典这位美丽的墨西哥姑娘。他默默地请求玛格丽特原谅他的过错,也请求儿子的原谅。简听到这些,心中那无法解开的郁闷与痛苦被释然了。

 

德芙背后的故事:初恋如冰淇淋般甜美 
1919年的春天,卢森堡王室的夏洛特公主继承了王位,同时她又嫁给了波旁家庭的后裔费利克斯王子。
  双喜临门,整个卢森堡王室热闹非凡。为了迎接那些贵客,御厨们更是通宵达旦地忙碌着。18岁的男孩莱昂已经在这个厨房工作了4个年头。14岁那年,他跟着做厨师的亲戚从希腊辗转来到卢森堡,后来进了王宫。
  这几天莱昂忙坏了,他的双手在水里泡得太久,几乎每根手指都裂开了口子。
  好不容易有点空闲,莱昂坐在门口用盐水擦洗伤口。“这样太不卫生了,伤口容易发炎。”一个细弱的声音轻轻地飘进了莱昂的耳朵,他抬头看到,阳光下站着一个女孩。女孩大方地坐在了他的身边,说:“要用药水擦洗,这样一定很疼吧?”她盯着莱昂的手指,心疼地微微皱起了眉头。就在莱昂不知道如何回答时,一个女佣跑进来说:“芭莎公主,快走,夫人在找你!”女孩回头冲­莱昂笑了一下,急忙跟着女佣跑了出去。原来她是公主!在王宫中,除了带他进来的亲戚,从来没有人关心过他,更何况是公主。她那几句简短的问候,让他产生了温暖的错觉。
  此后,莱昂得知,15岁的芭莎是波旁家族的远亲。因为无依无靠,所以被费利克斯王子带了过来。
  有一次,莱昂正在忙着手里的活计,转头时突然发现厨房门口有一个脑袋探来探去。当对方与莱昂的目光对上时,高兴地冲他招了招手。那正是芭莎公主。她塞给莱昂一个布包,便慌慌张张地走了。莱昂打开布包,里面竟然有一只疗伤的药膏。那个晚上,莱昂躺在床上,脑海中总是浮现出芭莎因为心疼而皱眉的样子,多善解人意的姑娘啊,他的心里既温暖又甜蜜。
  几天后,一位伯爵过生日,要在宫中举办一个小型宴会。宴会上的蛋卷冰激凌是当时刚刚流行的,它成了年轻的公主、王子们最喜欢的甜点。其实芭莎也很喜欢冰激凌,但这种还是稀罕物的美食是轮不上她的。莱昂开始设法为她做冰激凌。
  那天晚上,莱昂悄悄地潜入厨房。过了一会儿,一个橙子味的冰激凌就被他做好了。芭莎品尝着香甜爽滑的冰激凌,神情陶醉,仿佛陷入了某种美好的回忆。随后,她轻声告诉莱昂,她的母亲是个富有想象力的女人,在世时喜欢给她调制各种口味的冰激凌。莱昂恍然大悟,原来冰激凌里有芭莎对母亲的回忆。此外,由于母亲是英国人,芭莎也精通英文,她经常教莱昂简单的英语,似乎这样也能让她重温对母亲的回忆。
  从此以后,莱昂常常为芭莎调制各种口味的冰激凌。很多个繁星点点的夜晚,他们品尝着美味的冰激凌,­也让爱情的甜蜜萦绕在心头。不过,由于身份和处境的差异,他们谁都没有说出心里的爱意,只是默默地将这份感情埋在心底。
  悲伤的热克力难留爱情
  有一回,芭莎突发奇想地说:“­莱昂,冰激凌里加上巧克力会不会更好吃:”芭莎的愿望就是莱昂的动力,他又有了新的目标:巧克力冰激凌。如何让巧克力融入冰激凌并有最佳的口感,让他很费神。就在他苦苦琢磨时,一个消息像阴风一样吹进了卢森堡的王宫。
  20世纪初,小小的卢森堡在整个欧洲的地位很低,不时有人提出废除王室特权。为了找到一个靠山和同盟国,1921年,卢森堡和邻国比利时确立了经济同盟关系。为巩固两国之间的关系,王室联姻成了最好的办法,而被选中的人就是芭莎公主。这个新闻在御厨房里炸开了锅,正在埋头调制巧克力冰激淋的莱昂感到自己的心猛烈地抽搐着。
  一连3天,芭莎公主都没有出现在餐桌旁。心急如焚的莱昂盼着周三的晚上能快些到来,因为那是他们约定一起调制巧克力冰激凌的日子。可是那天晚上,芭莎失约了,直到莱昂盘中的冰激凌完全化掉,她也没有出现。莱昂感到有种撕心裂肺的疼痛。
  芭莎出现在莱昂的视线里已经是在一个月后,那天下午,他意外地在餐桌前看到了芭莎。她瘦了一圈,整个人看上去憔悴了许多。只是在看到莱昂的那一瞬间,她的眼中迸发出两道强烈的亮光,那光像剑一样刺痛了莱昂的心。他很想冲过去,抓住芭莎的手质问她,希望她告诉自己一切都是假的,她不会嫁人,因为她真心爱的人是他。可他是仆人,她是高贵的公主,莱昂无法开口确认她的爱情。
  这天,莱昂给公主和王子们准备的甜点依然是冰激凌,由于真正的巧克力冰激凌还没有调制成功,他急中生智,在芭莎的那份冰激凌上直接用热巧克力写了几个英文字母“DOVE” ,正是“DO YOU LOVE ME”的缩写。他相信如果芭莎心有灵犀,一定会读懂他的心声。莱昂紧张地盯着芭莎,看着那份写着字母的冰激凌转到了她的面前,可是直到上面的热巧克力融化,芭莎也没有仔细看那几个字母,她只是发了很长时间的呆,然后含泪吃下他为她做的最后一份冰激凌。
  几天之后,芭莎出嫁了。莱昂坐在高高的山坡上,看着载着芭莎的车驶向远方。他手里的冰激凌融化了,心爱的姑娘也远去了,他流下了伤心的眼泪。
  伤感情话刻在每一块巧克力上
  芭莎出嫁的第二年,莱昂离开了卢森堡,来到美国,在一家高级餐厅里找到了工作。他踏实肯干、虚心老实,老板很赏识他,便将女儿许配给他。几年后,莱昂随老板一家迁往芝加哥。在芝加哥,莱昂又成了一名糖果商。由于莱昂始终无法忘记芭莎,妻子只好与他离了婚。
  此后莱昂一直独自带着儿子,经营着他的糖果店。1946年的一天,莱昂看到儿子在追一辆贩卖冰激凌的车,当他拦下儿子后,儿子失望地告诉他,那辆车上有好吃的巧克力冰激凌。莱昂紧闭的心门顿时被撞开了。芭莎离开之后,他没有做过一次冰激凌。这次莱昂决定继续过去没有为芭莎完成的工作。­
  经过几个月的精心研制,一款富含奶油、同时被香醇的巧克力包裹的冰激凌问世了,上面刻着四个字母。儿子天真的问莱昂,冰激凌上“DOVE”这几个字母是什么意思。莱昂轻轻地说:这是冰激凌的名字。
  就在此时,莱昂意外地收到了一封来自卢森堡的信件,莱昂从信中得知,芭莎公主曾派人回国四处打听他的消息,希望他能够去探望她,却被告知他去了美国。芭莎到底怎么样了?她还好吗?莱昂的心仿佛又回到了当年,依然那么急迫而热切。
  历经千辛万苦,莱昂终于来到了比利时。芭莎并不在王宫,而是住在郊外一处破败的别墅里。迎接他的佣人神色悲戚,莱昂有了一种不祥的预感。芭莎老了,她虚弱地躺在床上,曾经如清波荡漾的眼睛变得灰蒙蒙的。莱昂扑在她的床边,眼泪无法自抑地滴落在她苍白的手背上。芭莎伸出手来轻轻地抚摸着莱昂的头发,用近乎微弱到听不清的声音叫着莱昂的名字。­随后,她艰难地讲述了整个故事。
  原来当年在卢森堡时,芭莎也深深地爱着莱昂,曾以绝食拒绝联姻,但是被送到宫外严密看守了一个月后,她深知自己不可能逃脱联姻的命运,何况莱昂从未说过爱她,更没有任何承诺。在那个年代,一个女子要同整个家庭决裂是要付出沉重代价的,何况她也无处可去。她最终只能向命运妥协,但希望离开卢森堡前能回王宫喝一次下午茶,因为她想在那里与莱昂作最后的告别。她吃到了他送给她的巧克力冰激凌,却没有看到那些融化的字母。
  听到这里,莱昂泣不成声,过去的误解终于有了答案。但一切都晚了。三天之后,芭莎离开了人世。莱昂听佣人说,自从芭莎嫁过来之后,终日郁郁寡欢,导致疾病缠身。她曾派人回去找过莱昂,得知他离开卢森堡并已经在美国结婚后,就一病不起。
  听到这里,莱昂泣不成声,如果当年那冰激凌上的热巧克力没有化掉,如果芭莎明白他的心思,她会改变主意与他私奔吗?他觉得一定会的。他开始悔恨自己的愚蠢和疏忽,为什么要在冰激凌上面用热巧克力写字。如果那巧克力是固体的,那些字就不会融化了,他就不会失去最后的机会。莱昂决定制造一种固体的巧克力,使其可以保存更久。­
  经过精心调制,香醇独特的德芙巧克力终于制成了,“DOVE”这四个字母被牢牢地刻在了每一块巧克力上,莱昂以此来纪念他和芭莎那错过的爱情。它虽然苦涩而甜蜜,悲伤而动人,如同德芙的味道。
  ­
  如今,德芙巧克力已经有了数十种口味,每一种爱情都能在这巧克力王国中被诠释和寄托。全世界越来越多的人爱上因爱而生、从冰淇淋演变而来的德芙。当情人们送出了德芙,就意味着送出了那轻声的爱情之问:DO YOU LOVE ME?那也是创始人在提醒天下有情人,如果你爱他(她),请及时让爱的人知道,并记得深深地爱,不要放弃。
 

名酒百利甜酒的传奇色彩和凄美爱情故事

【中文名称】:百利甜酒 
【英文名称】:Bailys
【材    料】:爱尔兰威士忌、奶油等
【联系方式】:爱尔兰 
调酒师的凄美爱情: 
  百利甜的发祥地在英国。那里有一位著名的调酒师,调酒师的太太是一名出色的女性,他们彼此深爱着对方。有一天,很不幸,调酒师的太太死于一次意外。调酒师从此悲伤,过着孤单的生活。直到一次出行的飞机上,调酒师遇到了一位像极了他前妻的空姐。他仿佛重获新生,一切生命的希望再一次点燃。那以后,调酒师疯狂的追求着那位空姐。
  但是空姐并不能接受调酒师的爱,空姐对调酒师说,有时候人的心会被蒙住。你对你前妻的思念和对我的爱完全是不同的情感,就像是奶和威士忌永远无法混在一起。调酒师听完空姐的话,默默的走开。他用了一年的时间,终于将奶和威士忌相溶,而且加了蜂密使味道也混为一体,并起了一个好听的名字(Baileys Rock),以此证明他对空姐的爱。当他知道空姐终于肯品尝这第一杯Baileys Rock时,忍不住在杯里加上了一滴眼泪。后来百利甜被空姐带上飞机,传播到世界各地,她对每一个喜欢喝百利甜的人说,“这杯酒,我等了一年”。 
典雅的甲丁坊酒吧:
  在景色秀丽、波光闪烁、绿荫铺地、花树倒影、斜阳染红水面的北京什刹海西岸,有一处别致的中式酒吧 — 甲丁坊。百利甜(baileys)全球排名第一的甜酒等你和你的她来鉴赏品尝,甲丁坊酒吧她古朴典雅,在荷花市场酒吧一条街,格外耀眼璀璨。
神奇的百利甜:
  百利甜不像酒,有人说它是漂亮女人的最爱,更像是巧克力奶昔。如果她不会或者不喜欢喝酒,百利甜会令她改变主意,放下矜持。百利甜是由新鲜的爱尔兰奶油和上等的爱尔兰whiskey调混而成,带有芳香的巧克力味道,香滑细腻。也有人说它像爱情,冰凉的巧克力奶油、甜蜜、诱人、却一点都不会觉得腻。并且还有点威士忌的刺激。
百利甜的品牌历史:
  1974年问世于爱尔兰;
  全球排名第一的力娇酒;
  全世界销量排名第六的高档酒,在近30年问世的最成功的高档酒品牌;
  在130个国家销售;
  纯天然的,是它是新鲜奶油、爱尔兰威士忌和上等烈酒及天然香料的完美结合,演绎爱尔兰的历史与农业传统;
  每年有40,000头奶牛为百利甜酒产奶;
最佳饮用方式:
  女人喝酒是很冒险的事情,其形象要不就是特别坠落,要不就是特别美丽,而百利甜酒的所有组合都是很适合时尚女性,让她们在品位百利酒芳香与细腻的同时,更能完美的展现出自身的时尚浪漫和优雅品位!无论在何时何地以百利甜酒做休闲饮品,都给饮家带来独特的感受。
  最风靡的方式就是将它倾倒在冰块上,使冰和酒相互融合交织,啜一口,就这样的一口,就让人情不自禁的爱上它。不过每一种喝法都能演绎出不同美妙味觉,适合在各种不同的场合饮用。

卡农(canon),是一种曲式的名称,是复调音乐的一种。原意为 "规律"。这种曲式的特征是间隔数音节不停重复同一段乐曲, 一个声部的曲调自始至终为另一声部所摹仿,即严格的摹仿对位。卡农出现于13、14世纪,后人常采用古典曲调作为卡农主题。帕赫贝尔的"D大调卡农",是作者在意大利威尼斯时写的音乐,带有一丝意大利式的,甜蜜宁静的忧伤。在不到五分钟演奏长度里,所谓的"顽固音型主题" 反复出现28次,可是听者却能陶醉在这旋律之中,丝毫不会感到单调。
   故事:
  Pachelbel在他10几岁的时候,流浪到英国被英国一个小村庄的琴师收养,之后他天天听那个他弹琴,也学会了钢琴。
  在他们旁边的镇上上有一个女孩子叫Barbara Gabler,家里有钱有势,Barbara Gabler也是镇上最漂亮的女孩,自从到教堂听Pachelbel弹的曲子,就爱上了他。很多有钱人上门向Barbara Gabler提亲都被拒绝了,因为Barbara Gabler心里只喜欢Pachelbel。但女孩比较害羞,从小被宠到大的Barbara Gabler一直不敢向Pachelbel表白,后来Barbara Gabler就找了个理由,说要去Pachelbel那里学钢琴,她对Pachelbel说自己热爱音乐,希望可以拜师学艺。Pachelbel很高兴的收下了这个徒弟。
  但Barbara Gabler她的目的并不是弹琴,所以几乎不把精力花费在钢琴上,遭到了Pachelbel一再的责骂。Barbara Gabler心里委屈,但还是一直跟Pachelbel,希望Pachelbel能明白自己的心意。终于有一天Pachelbel对Barbara Gabler说:“你走吧,你真的不适合弹钢琴。而且你也不喜欢钢琴。”Barbara Gabler听后,对自己说:“不要说我不行!Pachelbel。我回去一定要好好弹琴,半年后我要拿到本地的钢琴第一名的!” 半年里,Barbara Gabler天天练习,饿了就叫家里的用人送些吃的,困了就趴着睡一会。半年一转眼就过去了,Barbara Gabler参加了比赛,果真的拿了奖。
  Barbara Gabler想拿这个奖杯去Pachelbel向他表达自己的爱意Pachelbel已经走了。当时正值战乱,Pachelbel被征去打仗,Barbara Gabler说:“好,我等他回来。”就这样Barbara Gabler等了Pachelbel3年多。
  在这期间村长的儿子看上Barbara Gabler ,村长的儿子很清楚Barbara Gabler已经心有所属,就叫人从前线运回来一具碎尸体,说那就是Pachelbel,但没有人可以对证,Barbara Gabler相信Pachelbel真的已经死了,趴在"Pachelbel"的尸体上哭了3天3夜,那时,村长的儿子买了很多的礼物去找Barbara Gabler提亲,Barbara Gabler没有理睬。
  在3天后晚上, Barbara Gabler割腕自杀了。而Pachelbel在Barbara Gabler离开的半年里,他发现没有Barbara Gabler在身边,自己少了很多很多的快乐。Pachelbel在Barbara Gabler离开后才发现原来自己已经不知不觉的爱Barbara Gabler,只是因为她学琴不努力所以就埋没了对她的喜欢。当时他准备写一首歌,做为向Barbara Gabler求婚的礼物,当他完成了卡农的1/3的时候。他被招去打仗了,在战乱中,自己的生命多次都是九死一生,每当心中不舒服的时候都会想到 Barbara Gabler,想到教她弹琴的日子......那段时光真是值得怀念啊。之后他完成了卡农剩下的2/3。
  在Barbara Gabler自杀后了第2个月,Pachelbel回到了村里。他从村民的口中知道了Barbara Gabler的故事和她为自己做的所有事后,他咆哮着,放声大哭。他找到了Barbara Gabler的家人,问她现在葬在哪。她家里人都不肯告诉Pachelbel,随后的一次礼拜,Pachelbel招集他们村和Barbara Gabler村上所有的人,他坐在钢琴前强忍着泪水,弹出了卡农"Canon(D Major)”。

品牌故事:劳斯莱斯标识背后的爱情故事

爱情可超越一切
劳斯莱斯汽车以贵族气质享誉全球,征服了包括英国王室在内的各国元首、贵族和富豪的心,它被称为“帝王之车”,是地位和身份的象征。英国女王无论走到哪里,座驾一定是劳斯莱斯。劳斯莱斯车身上的“飞翔女神”经典车标更被称为汽车史上最有名最动人的汽车标志。不过,它并不是自劳斯莱斯车诞生之日就出现在车身的,而是因一段真实却不为人知的悲情之恋而生。
七幅画像与一份跨越世俗的爱
1902年夏季,对埃莉诺·桑顿来说,是极不寻常的一个夏天。她是一位美丽的姑娘,但命运却并没有因为她拥有过人的容貌和聪颖的头脑而对她格外眷顾。年幼时的桑顿遭到父亲的遗弃,与靠擦皮鞋为生的母亲相依为命。为了养家和给重病卧床的母亲治病,她只能到夜总会当交际花。
因为容貌倾城,又能歌善舞,桑顿很快成了夜总会里的红人,被许多贵族公子哥追崇着。可桑顿虽地位卑微,却生了一身傲骨。她知道那些声称说爱她的男人,不过是爱她的美貌与身体,一旦得到,终有一天他们会像厌弃一块抹布一样将她抛弃。
这天晚上,桑顿像往常一样在伦敦最富有人气的夜总会开始了表演。演出结束后,桑顿照旧收到一些男人们送来的礼物,她在那堆东西中无意发现了一件特别的礼物。那是一张人物速描像,主角正是跳舞的她。这幅画或许只是哪个无聊的贵族少爷来消遣她的吧,桑顿并没特别放在心上。但令她意外的是,第二天又收到了一幅肖像,这次画像里的她像一位月光下的公主,高贵而忧郁。接下来一连三天,桑顿都从侍应生的手中收到了画像,而且每张画像中的她各有风情,或妩媚,或忧郁,或纯真。
第七天的时候,桑顿演出后直接等在了后台的入口,她终于见到了那个送画来的神秘人。那是个长像俊朗的男人,棱角分明的轮廓,深邃的眼睛,他便是约翰·蒙塔古。
出身贵族世家的英国保守党议员蒙塔古是当时名噪一时的人物,他不仅是一个飞车迷,还是个绘画迷,刚刚担任了《汽车画报》的主编。蒙塔古和朋友第一次在夜总会见到舞台上的桑顿时,她正像一只孤独而骄傲的天鹅在起舞。他被她身上特殊的气质吸引了,这个有着褐色卷发、忧郁眼神的女孩仿佛是一幅古典而美丽的油画。蒙塔古一时看得发呆,忍不住随手将她的肖像画了下来。从此他疯狂地迷恋上了她的一颦一笑,直到她真实地站在他的面前,他还觉得如坠梦中。
桑顿和蒙塔古就这样相识了,并电闪雷鸣般撞击出爱的火花。蒙塔古成了桑顿最忠实、也最特殊的观众。不过,蒙塔古为了避免不必要的麻烦,也怕桑顿会因身份悬殊而有所顾虑,只告诉她自己是《汽车画报》的主编。
强烈的爱情让蒙塔古将贵族的戒律抛到了一边。为了每时每刻都能见到桑顿,他为她还清了夜总会的所有债务,让她担任自己的秘书,并无所顾忌地带着她去见他的那些朋友,不顾任何人的拦阻和桑顿同居了。
爱情在兜兜转转里错过和失落
但是上帝似乎总是嫉妒太过完美的爱情,一个好事的朋友不想看到蒙塔古为了一个舞女“如此堕落”,便悄悄地将他与桑顿的事告诉了他的家人。蒙塔古的家人知道他在外面“包养舞女”后惊骇和愤怒,他们觉得这是对家族的侮辱。
于是怒火中烧的老蒙塔古爵士立即派人将蒙塔古找了回去。他们召开家族会议,将蒙塔古像审犯人一样审讯。蒙塔古与父亲挣执起来,说自己和桑顿是真心相爱的,谁也阻止不了他们的爱情,甚至宣称为了桑顿情愿放弃继承爵位。老爵士勃然大怒,他对蒙塔古说,你将一无所有。
果然,老爵士一气之下中断了对蒙塔古的经济支援。蒙塔古感到靠报社那点微薄的薪水根本无法继续给予桑顿富足幸福的生活,但又不想让心爱的女孩重新过困窘的生活。然而他们不仅要吃饭穿衣,还要支付公寓的昂贵租金,蒙塔古渐渐力不从心,他开始向朋友们借钱维持体面,然而那些人多是蒙塔古家族的世交,已经受过嘱咐,谁都不肯伸出手来帮他一下。老爵士觉得只要将儿子逼到无路可走,他就会乖乖回家的。蒙塔古终于陷入了从来没有过的绝境。
一无所知的桑顿依然沉浸于心醉神迷的爱情中,但有时看到蒙塔古因为一些鸡毛蒜皮的事冲着自己发火,她感到困惑和无所适从,甚至怀疑蒙塔古是不是有些厌倦她,想离开她。正在她胡思乱想的时候,老蒙塔古爵士找到了她。
原来,在蒙塔古离家出走半年之久后,老爵士见儿子如此桀骜不逊,一点没有回头的迹象,非常恼火。于是,他让人发信给蒙塔古,告诉他母亲病危。蒙塔古接到这封信后,忧心忡忡地对桑顿说,他的母亲病重,他必须要回家一趟。
随后,骄傲的老爵士上门了,从他嘴里,桑顿知道了一切真相:原来她深爱的蒙塔古不仅有显赫的身份和地位,还即将与一位贵族小姐联姻。因此,他的父亲称她以这么卑贱的身份,想跨进上流社会简直是无稽之谈,如果她与他的儿子结合,生出的孩子将是对蒙塔古家族高贵血统的侮辱。
最后,老爵士不由分说扔下一笔钱,让桑顿拿着这些钱走得远远的,不要再纠缠他的儿子。老爵士走后,她终于撑不住了,泪留满面。她感到自己真的是太低贱了,怎么能让蒙塔古这样的贵族爱上她?再联想到蒙塔古这段时间的反常表现,还有对她隐瞒的身份,她几乎相信了老爵士的话。然而桑顿仍抱着一线希望,希望蒙塔古回来亲自告诉她那一切都是假的,他是真心地爱着她。
可在公寓中苦苦等待了一个月,桑顿没有得到蒙塔古的任何消息。桑顿伤心欲绝,终于相信蒙塔古不仅放弃了她的感情,连金钱也不愿为她再多付出一分。这个时候,她又意外地发现自己怀了身孕,便不得不动用老爵士留下的那笔钱结清房租,另找了处便宜的房子住下。
而蒙塔古匆匆回到家后,母亲安然无恙地出来迎接他,他这才知道是个骗局。但是一切已经来不及了,他被直接关进了城堡。当他被放出来是三个月之后,已经到了冬天,伦敦下了一场很大的雪,老爵士用比天气还要冰冷的语气告诉他,我们去调查过了,你爱的那个女人早就跟着有钱人跑掉了。
蒙塔古自然不会轻信父亲的话,他火急火燎地赶回公寓,却发现那里的确如父亲所说人去楼空,贵重物品和珠宝连同桑顿都不见了。蒙塔古当时感到天昏地暗,有一种坠入深渊的感觉。
从那之后,蒙塔古关闭了自己的心房,他再也不相信忠贞不渝的爱情,过着放浪形骸的生活。 1905年,老爵士去世后,他成为伯利欧地区的第二代蒙塔古爵士,并与贵族小姐塞西尔成婚。
情归深海爱留风中
蒙塔古的婚姻生活并不幸福,他将妻子冷落在家中,终日与朋党去花天酒地。1906年春天,新开张的一家酒吧为招览生意,特意请了一批脱衣舞女郎来表演。蒙塔古与朋友们听说后,立即相邀同去玩乐。那些露着性感身材大跳艳舞的女郎们引起了一阵阵的轰动和尖叫,蒙塔古和朋友们也沉浸在刺激和快乐的气氛中。最后一个紫衣女郎出来时,还处在兴奋中的蒙塔古突然地呆住了,那张化着浓妆的冷艳面孔是那么熟悉,实在太像桑顿了。虽然一直刻意想忘记她,可她容貌却无时无刻不在他的脑海中萦绕。
当紫衣女郎将最后一件衣服脱掉时,舞台下几乎沸腾了。蒙塔古看见她胸口有颗醒目的红痣,与桑顿身上的那颗在同一个位置。蒙塔古不敢相信自己的眼睛,他立即告诉自己,那也许只是喝醉了酒的幻觉,现在的桑顿或许早就和哪个有钱男人在一起过着锦衣美食的日子。他几乎是冲到了后台,在那些女人中寻找着紫衣舞女的身影,终于在一个角落中,他看到那个女人正在一点点整理自己的物品。
这么忧郁的眼神,这么美丽的面孔,除了是桑顿还能是别人吗?桑顿和蒙塔古隔了三年,却在这样尴尬的情景中重逢了。她不知道如何面对这个自己深深爱过的男人,只能仓惶而逃。蒙塔古跟着桑顿跑过了几条街,在低矮贫民区的一间破房子中他看到了桑顿和那个孩子。看到当年他为桑顿画的那七幅肖像被一张张地贴在墙上后,蒙塔古的眼眶红了。
他们终于弄清楚了事情的真相,两个人之间的误解和怨恨就在这一刻烟消云散。
这几年里,桑顿为了养活自己和孩子,只能重操旧业出来卖笑挣钱。蒙塔古望着满脸憔悴的她,不知不觉中眼泪便流了出来。他发誓要弥补给她一切。
蒙塔古回家的当晚,便向整个家族宣布他要与妻子塞西尔离婚,娶回桑顿。他的母亲被气得晕了过去。这时,深爱丈夫的塞西尔试图为他解围,她表示可以接受桑顿,甚至请她搬进他们家里共同生活。
塞西尔的话让蒙塔古冷静了下来。想想这几年来,他一直冷漠对待塞西尔,可她到关键时刻还能设身处地地为他和他的家族着想,并且心甘情愿地成全他的爱情,他感到心酸。最后,蒙塔古将桑顿和孩子安置在了伦敦一处公寓中。为顾全家族声誉,蒙塔古与桑顿的恋情只能在地下继续,他感到压抑和痛苦。
这时,蒙塔古的艺术家朋友查尔斯·塞克斯被他们曲折的爱情深深打动了,应蒙塔古之请,他以桑顿为原形创作了一个生动的雕塑:一个年轻的女孩,迎风而立,任凭裙裾在风中飞扬,却把食指轻轻地点在自己的嘴唇上,这正象征着桑顿和蒙塔古之间不能言说的秘密情感。这个雕塑被命名为“私语”。蒙塔古将第一个“私语”放在他的劳斯莱斯“银色幽灵”轿车的车头,含蓄地以此宣示自己的爱情,很快,这独特的车标竟被不明就里的上流社会劳斯莱斯车主们引为风尚。
这股风潮很快被劳斯莱斯创始人之一的劳斯先生获悉,他找到好友蒙塔古,委托他请塞克斯在“私语”的基础上为劳斯莱斯汽车设计一个专门的车标。这一次塞克斯征求了桑顿的想法,将“私语”女孩的双手改为如羽翼般向后伸展,其它不变,汽车史上最经典的“飞翔女神”车标就此诞生了。1911年2月6日,镀银的“飞翔女神”正式飞上了华贵的劳斯莱斯车头,成为劳斯莱斯车的车标。“飞翔女神”启用典礼的隆重程度不亚于第一辆劳斯莱斯轿车下线。就这样,蒙塔古以一种特别的方式表达了对情人桑顿的深情。每当他用车载着自己的女神桑顿飞驰在风中,那美丽的车标在阳光里闪烁着无限光芒时,桑顿就觉得空气都洋溢着爱的气息,而她成了世界上最幸福的女人。
天妒红颜,1915年12月,蒙塔古被派往印度负责考察机动化部队,为了能陪伴在爱人的身边给他安慰,桑顿与蒙塔古一起登上了开往印度海的轮船。12月13日,当蒙塔古和桑顿乘坐的轮船驶入地中海海域时,被德军的鱼雷击中。冰冷刺骨的海水中,蒙塔古紧紧地抓住桑顿,可桑顿的身体很快就被冻僵了,在即将完全失去知觉的那一刻,她拼劲全身力气最后一次对他说:“我爱你!”随后,蒙塔古眼睁睁看着挚爱一生的女人被无情的海浪吞噬。
蒙塔古在海上漂了36个小时,最终获救。回到英格兰后,他很长一段时间都无法从失去桑顿的悲痛中恢复过来,整日精神恍惚,抚摸着“飞翔女神”车标流泪。
劳斯先生决定用一种特殊的方式抚慰朋友受创的心。从此,劳斯莱斯将以桑顿为原型制造的“飞翔女神”车标从银白色改为了镍合金制作,因为只有金灿灿的女神像才能代表劳斯莱斯汽车无与伦比的品质,也只有永不褪色的光彩才能映射桑顿与蒙塔古长达13年的刻骨悲情。
斯情已归大海,但永恒的爱却被每一辆伫立在劳斯莱斯车头的“飞翔女神”诉说,永远呢喃在风中……

1650年,斯德哥尔摩的街头,52岁的笛卡尔邂逅了18岁的瑞典公主克里斯汀。 
  那时,落魄、一文不名的笛卡尔过着乞讨的生活,全部的财产只有身上穿的破破烂烂的衣服和随身所带的几本数学书籍。生性清高的笛卡尔从来不开口请求路人施舍,他只是默默地低头在纸上写写画画,潜心于他的数学世界。 
  一个宁静的午后,笛卡尔照例坐在街头,沐浴在阳光中研究数学问题。他如此沉溺于数学世界,身边过往的人群,喧闹的车马队伍。都无法对他造成干扰。 
  突然,有人来到他旁边,拍了拍他的肩膀,“你在干什么呢?”扭过头,笛卡尔看到一张年轻秀丽的睑庞,一双清澈的眼睛如湛蓝的湖水,楚楚动人,长长的睫毛一眨一眨的,期待着他的回应。她就是瑞典的小公主,国王最宠爱的女儿克里斯汀。 
  她蹲下身,拿过笛卡尔的数学书和草稿纸,和他交谈起来。言谈中,他发现,这个小女孩思维敏捷,对数学有着浓厚的兴趣。 
  和女孩道别后,笛卡尔渐渐忘却了这件事,依旧每天坐在街头写写画画。 
  几天后,他意外地接到通知,国王聘请他做小公主的数学老师。满心疑惑的笛卡尔跟随前来通知的侍卫一起来到皇宫,在会客厅等候的时候,他听到了从远处传来的银铃般的笑声。转过身,他看到了前儿天在街头偶遇的女孩子。慌忙中,他赶紧低头行礼。 
  从此,他当上了公主的数学老师。 
  公主的数学在笛卡尔的悉心指导下突飞猛进,他们之间也开始变得亲密起来。笛卡尔向她介绍了他研究的新领域——直角坐标系。通过它,代数与几何可以结合起来,也就是日后笛卡尔创立的解析几何学的雏形。 
  在笛卡尔的带领下,克里斯汀走进了奇妙的坐标世界,她对曲线着了迷。每天的形影不离也使他们彼此产生了爱慕之心。 
  在瑞典这个浪漫的国度里,一段纯粹、美好的爱情悄然萌发。 
  然而,没过多久,他们的恋情传到了国王的耳朵里。国王大怒,下令马上将笛卡尔处死。在克里斯汀的苦苦哀求下,国王将他放逐回国,公主被软禁在宫中。 
  当时,欧洲大陆正在流行黑死病。身体孱弱的笛卡尔回到法国后不久,便染上重病。在生命进入倒计时的那段日子,他日夜思念的还是街头偶遇的那张温暖的笑脸。他每天坚持给她写信,盼望着她的回音。然而,这些信都被国王拦截下来,公主一直没有收到他的任何消息。 
  在笛卡尔给克里斯汀寄出第十三封信后,他永远地离开了这个世界。此时,被软禁在宫中的小公主依然徘徊在皇宫的走廊里,思念着远方的情人。 
  这最后一封信上没有写一句话,只有一个方程:r=a(1-sinθ)。 
  国王看不懂,以为这个方程里隐藏着两个人不可告人的秘密,便把全城的数学家召集到皇宫,但是没有人能解开这个函数式。他不忍看着心爱的女儿每天闷闷不 乐,便把这封信给了她。拿到信的克里斯汀欣喜若狂,她立即明白了恋人的意图,找来纸和笔,着手把方程图形画了出来,一颗心形图案出现在眼前,克里斯汀不禁 流下感动的泪水,这条曲线就是著名的“心形线”。 
  国王去世后,克里斯汀继承王位,登基后,她便立刻派人去法国寻找心上人的下落,收到的却是笛卡尔去世的消息,留下了一个永远的遗憾…… 
  这封享誉世界的另类情书,至今,还保存在欧洲笛卡尔的纪念馆里。